Sie sind auf Seite 1von 68

1 / 68

BOARD QUESTIONS IN PHARMACOLOGY

CHOOSE THE BEST ANSWER: MPL = 44.5

1. A 40 year old female who underwent total abdominal hysterectomy with bilateral salpingo-oophorectomy. To prevent osteoporosis what HRT regimen
will be best for her. She claims to have no history of breast cancer in the family:
A. Estrogen only preparation
B. Estrogen and medroxyprogesterone acetate
C. Medroxyprogesterone acetate
D. Raloxifene
MPL- 0.25 REFERENCE: PHARMACOLOGY 4th Ed. Rang, pp. 440
2. The Patient at the delivery room is having uterine atony after giving birth to an 8-lb baby boy.
Her BP at present is 150/90 mmHg. What agent should you give her:
A. Methylergonovine maleate C. Carboprost
B. Oxytocin D. Ergometrine
MPL: 1 REFERENCE: PHARMACOLOGY 4th Ed. Rang, et al pp.450
3. An old female was treated for endometriosis for 6 months. She claims to have been given a drug with androgenic effects such as hirsutism deepening
of the voice and acne. Which of the following drugs could have been given:
A. GnRH agonist C. Danazol
B. Combined oral contraceptive pills D. Medroxyprogesterone acetate
MPL: 0.25 REFERENCE: PHARMACOLOGY 4th Ed. Rang, pp.447
4. A patient underwent hip replacement and is being given morphine for pain relief. At present her RR was noted to be at 8 cycles/min with prolonged
episodes of apnea. What will you give in this patient?
A. Atropine sulfate C. Naloxone
B. Flumazenil D. Protamine sulfate
MPL; 1 REFERENCE: PHARMACOLOGY 4th Ed. Rang, pp.598
5. If you have a patient with seizure disorder which anesthetic agent will not be appropriate: A. Enflurane
C. Halothane
B. Nitrous oxide D. Procaine
MPL: 0.25 REFERENCE: PHARMACOLOGY 4th Ed. Rang, pp.523
6. A 50-year-old male with COPD is complaining of difficulty of sleeping for almost one month. Which of the following sedative hypnotics would be best
for him:
A. Diazepam C. Zolpidem
B. Pentobarbital D. Thiopental
MP: O.25 REFERENCE: PHARMACOLOGY 4th Ed. Rang, pp.534
7. On waking up a patient who just underwent knee surgery cannot remember what happened while he is at the Operating Room. The anxiolytic agent
that was probably given to him was _______.
A. Buspirone C. Hydroxyzine
B. Lorazepam D. Thiopental
MPL: 0.25 REFERENCE: PHARMACOLOGY 4th Ed. Rang, pp.536
8. A 22-year-old female was found unconscious in her room. She presently broke up with her boyfriend. By her bedside a number of sleeping pills and
whiskey was noted. What could be the possible effect and interaction that occurred?
A. Additive C. Potentiation
B. Antagonism D. Synergism
MPL: 0.33
9. If the patient will be given antidepressants. What adverse effect could be anticipated?
A. Insomnia C. Sedation
B. Diarrhea D.Tachycardia
MPL: 0.33 REFERENCE: PHARMACOLOGY 4th Ed. Rang, pp.556
10. Antipsychotic potency generally runs parallel to the activity on which of the following receptors:
A. D1 B. D2 C. D3 D. D4
MPL: 0.25 REFERENCE: PHARMACOLOGY 4th Ed. Rang, pp.487
11. The WHO classification of amphetamines:
A. Cognition enhancer C. Neuroleptic
B. Hallucinogen D. Psychostimulants
MPL: 0.33 REFERENCE: PHARMACOLOGY 4th Ed. Rang, pp.469
12. If the patient claims to have visual hallucinations, what could be the possible psychotropic drug class that was taken?
A. Antidepressant C. Psychotomimetic
B. antipsychotic D. Psychomotor stimulant
MPL: 0.25 REFERENCE: PHARMACOLOGY 4th Ed. Rang, pp.469
13. A student would like to improve her memory and cognitive performance. What class of psychotropic drug will she choose:
A. Cognitive enhancers C. Psychostimulants
B. Psychotomimetics D. Thymoleptics
MPL: 1 REFERENCE: PHARMACOLOGY 4th Ed. Rang, pp.469
14. A bone cancer patient took high doses of morphine to relieve pain. What agent can be used to reverse its toxic effects?
A. Nalbuphine C. Nalorphine
B. Naloxone D. Methadone
MPL: 1 REFERENCE: PHARMACOLOGY 4th Ed. Rang,pp.598
15. If respiratory depression would be the marked effect of the opioid analgesic, it is probably more selective to what receptor subtype?
A. Delta C. Kappa
B. Gamma D. Mu
MPL: 0.25 REFERENCE: PHARMACOLOGY 4th Ed. Rang,pp.592
16. Which of the following is not a manifestation of a patient taking an opioid analgesic. Selectively interacting with mu receptors?
A. Analgesia C. Physical dependence
B. Dysphoria D. Respiratory depression

Pharmacology
2 / 68
17. If a child was brought to the clinic with skin abscess. Wound cotton done revealed the presence of Staph aureus. What antibiotic will be appropriate
for this child:
A. Amoxicillin C. Cloxacillin
B. Cephalexin D. Vancomycin
MPL: 1 REFERENCE: PHARMACOLOGY 4th Ed. Rang, pp.693
18. The second generation cephalosporin which crosses the blood brain barrier:
A. Cefoxime C. Cefoxitin
B. Cefuroxime D. Cefoperazone
MPL: 1 REFERENCE: PHARMACOLOGY 4th Ed. Rang, p.694
19. A G2P1 term, will be given prophylactic antibiotic prior to CS. Which of the following should be recommended:
A. Cephalexin C. Cefazolin
B. Cefoxitin D. Ceftriaxone
MPL: 0.33
20. After prolonged treatment with penicillin, the patient developed pseudomembranous colitis. What antibiotic will you give for this case:
A. Aztreonam C. Imipenem
B. Ceftriaxone D. Vancomycin
MPL: 0.25 REFERENCE: PHARMACOLOGY 4th Ed. Rang,p.702
21. This agent inhibits topoisomerase II (DNA gyrase)
A. Clindamycin C. Spectinomycin
B. Ciprofloxacin D. Tetracycline
MPL: 0.25 REFERENCE: PHARMACOLOGY 4th Ed. Rang, p.720
22. In case of gram-negative septicemia if Ampicillin will be given with Gentamicin. What will be the expected response?
A There will be better chances of resolution
B B. Similar effect with monotherapy
C. Disease resolution will be delayed
D. No response will be noted
MPL:1 REFERENCE: PHARMACOLOGY 4th Ed. Rang Pp. 698
23.. A neonate was given IV antibiotics for 3 days. However, the baby had hypothermia, diarrhea and other gray color. The baby was probably given:
A. Amikacin C. Chloramphenicol
B. Erythromycin D. Gentamycin
MPL: 1 REFERENCE: PHARMACOLOGY 4th Ed. Rang,p.692
24. The following antibiotics inhibit protein synthesis. Which of the following inhibits the translocation process:
A. Erythromycin C. Netilmicin
B. Chloramphenicol D. Tetracycline
MPL: 0.25 REFERENCE: PHARMACOLOGY 4th Ed. Rang,p.699
25. The purpose of giving compound drug therapy in the treatment of tuberculosis:
A. to rapidly eradicate the strains of tubercle bacilli
B. to shorten the infections phase
C. to decrease the emergence of resistant organisms
D. to prevent complications of tuberculosis
MPL: 0.33 REFERENCE: PHARMACOLOGY 4th Ed. Rang,p.706
26. The patient on the 4th month of anti-TB treatment is having peripheral neuropathy. This may be attributed to which of the following:
A. Isoniazid C. Pyrazinamide
B. Ethambutol D. Streptomycin
MPL: 1 REFERENCE: PHARMACOLOGY 4th Ed. Rang,p.704
27. Which of the following is NOT a first line agent in the treatment of tuberculosis?
A. Ethambutol C. Rifampicin
B. Pyrazinamide D. Streptomycin
MPL: 1 REFERENCE: PHARMACOLOGY 4th Ed. Rang,p.706
28. This agent is NOT included in the management if lepromatous leprosy:
A. Capreomycin C. Dapsone
B. Clofazimine D. Rifampicin
MPL: 0.33 REFERENCE: PHARMACOLOGY 4th Ed. Rang,p.706
29. It is important for the clinician to avoid selection of an antiretroviral regimen that contains agents with similar toxicity profiles. Which regimen below
contains at least 2 agents with similar toxicity profiles?
A. Zidovudine, Lamivudine, Indinavir C. Zidovudine, Didanosine, Saquinavir
B. Didanosine, Zalcitabine, Nelfinavir D. Stavudine, Lamivudine, Indinavir
MPL: 0.25 REFERENCE: PHARMACOLOGY 4th Ed. Rang,p.714
30. A guanosine derivative which selectively inhibits viral DNA polymerase:
A. Aciclovir C. Vidarabine
B. Amantadine D. Zidovudine
MPL: 0.25 REFERENCE: PHARMACOLOGY 4th Ed. Rang,p.713
31. A patient taking Warfarin was also being treated for fungal infection. After 7 days, the patient developed epistaxis, Which of the following antifungal
agents may have this interaction with Warfarin?
A. Nystatin C. Flucytosine
B. Ketoconazole D. Amphotericin B
MPL: O.25 REFERENCE: PHARMACOLOGY 4th Ed. Rang,p.317
32. If the patient became anemia with neutropenia and thrombocytopenia during antifungal treatment. . What is the possible agent she is taking?
A. Griseofulvin C. Flucytosine
B. Fluconazole D. Terbinafine
33. A7 year old patient was admitted because of pneumonia. On routine stool exam E. Histolytic Cyst 5 – 8/hpf was noted:
Anti- amebic treatment not necessary
B. Give diloxanide furoate
C. Give metronidazole
D. Give both diloxanide furoate and metronidazole
34. Which of the following anti-malarial agent may promote radical cure?
Pharmacology
3 / 68
A. Chloroquine C. Primaquine
B. Mefloquine D. Pyrimethamine
MPL: 0.25 REFERENCE: PHARMACOLOGY 4th Ed. Rang,p.728
35. The drug of choice for mixed round worm infection:
A. Pyrantel pamoate C. Niclosamide
B. Mebendazole D. Praziquantel
MPL: 0.50 REFERENCE: PHARMACOLOGY 4th Ed. Rang,p.740
36. On follow-up, after treatment with tapeworm infection, scolex was noted in the stool of the child:
A. Praziquantel C. Mebendazole
B. Niclosamide D. Ivermectin
MPL: 0.25 REFERENCE: PHARMACOLOGY 4th Ed. Rang,p.742
37. Classified as anti-cancer antimetabolite agent:
A. cyclophosphamide C. Methotrexate
B. Doxorubicin D. Paclitaxel
MPL: 0.25 REFERENCE: PHARMACOLOGY 4th Ed. Rang,p.676
38. An agent used for cancer chemotherapy that inhibits purine synthesis:
A. Cytarabine C. Mercaptopurine
B. Fluorouracil D. Vincristine
MPL: 0.50 REFERENCE: PHARMACOLOGY 4th Ed. Rang,p.677
39. Does not cause myelosuppression:
A. Bleomycin C. Etoposide
B. Cisplatin D. Dactinomycin
MPL: 0.25 REFERENCE: PHARMACOLOGY 4th Ed. Rang,p.678
40. In a prescription made by an internist, the generic name was enclosed in a parenthesis and written below the brand name. This will be interpreted by
the drug store as:
A. Violate prescription C. Erroneous prescription
B. Impossible prescription D. Correct prescription
MPL: 0.50 Generics Act of the Philippines
41. If the physician makes a prescription order that utilizes a drug supplied by the pharmaceutical company, what class of prescription order was made:
A. Extemporaneous C. Precompounded
B. Compounded D. Erroneous
MPL: 0.25
42. The response of a patient who has bronchial asthma on the following drugs maybe graded as follows:
A. Epinephrine > Norepinephrine > Isoproterenol
B. Isoproterenol > Epinephrine >> Norepinephrine
C. Isoproterenol > Epinephrine = Norepinephrine
C D. Isoproterenol = Epinephrine >> Norepinephrine
MPL: 0.33 REFERENCE: PHARMACOLOGY 4th Ed. Rang,p.160
43. A patient with septic shock was noted to have absent urine output for the past 12 hours. Which of the following drugs would be most helpful to this
patient?
A. Furosemide C. Dopamine
B. Norepinephrine D. Epinephrine
MPL: 0.33 REFERENCE: PHARMACOLOGY 4th Ed. Rang,p.288
44. A 40 year old male was seen at the ER because of anaphylaxis and was immediately given Epinephrine. Apparently, The patient has taken
prazosin for his hypertension. Which of the following maybe observed in this patient?
A. The patient may develop hypertensive crisis.
a. B. There will be no effect on the patient’s blood pressure.
C. The patient may develop hypotension.
D D. The patient may develop severe difficulty of breathing due to bronchoconstriction.
E MPL: 0.33 REFERENCE: PHARMACOLOGY 4th Ed. Rang,P.152
45. Which of the following actions of norepinephrine is blocked by prazosin?
A. Bronchial smooth muscle relaxation
B. Vascular smooth muscle contraction
B. Renin release
C. Increased heart rate
MPL: 0.25 REFERENCE: PHARMACOLOGY 4th Ed. Rang,p.152
46. Bradycardia maybe noted as reflex response when a patient is given
A. Dobutamine C. Norepinephrine
B. Propranolol D. Methyldopa
MPL: 0.25 REFERENCE: PHARMACOLOGY 4th Ed. Rang,p.127
47. Rifampicin and INH are given together for the treatment of pulmonary tuberculosis because their drug interaction is
A. Additive C. Potentiation
B. Synergism D. Antagonism
MPL: 0.33 REFERENCE: PHARMACOLOGY 4th Ed. Rang,p.703
48. If the plasma concentration of a drug declines with first order kinetic, this means that
A. there is only one metabolic path for drug disposal
B. the rate of elimination is proportional to the remaining concentration
b. C. the drug is largely metabolized in the liver after oral absorption and has low bioavailability
D. the drug is not distributed outside the vascular systems
MPL: 0.50
REFERENCE: PHARMACOLOGY 4th Ed. Rang,p.87
49. The following is NOT phase II drug metabolized reaction:
A. Oxidation C. Methylation
B. Glucoronidation D. Acetylation
MPL: 0.25 REFERENCE: PHARMACOLOGY 4th Ed. Rang,p.79

Pharmacology
4 / 68
50. Mr. Jose Vida was admitted due to severe pneumonia. He was given a drug with a volume of distribution of 40L and clearance of 80ml/min. What
should be the loading dose in order to achieve a therapeutic plasma concentration of 40mg/L?
A. 0.1 mg/L C. 115.2 mg
B. 10 mg D. 160 mg
MPL: 0.25 REF. BASIC & CLINICAL PHARMACOLOGY 9th Ed. KATZUNGp.46
51. Two drugs A and B have the same mechanism of action. Drug A at a dose of 5 mg produce the same magnitude of effect as drug B at a dose of
500 mg. This means that
A. Drug B is less efficacious than drug A
B. Drug A is 100x more potent than drug B
C. toxicity of drug A is less than that of drug B
F D. Drug A is more effective than drug B.
MPL: 0.50 REFERENCE: PHARMACOLOGY 4th Ed. Rang,p.29
A 67 year old male was seen at the OPD due to epistaxis which occurred thrice for the past week and easy bruising. He also
complained of abdominal discomfort and lack of appetite.
PMH: Atrial fibrillation, Nocturnal heartburn
Social history: Chronic alcohol abuse
Medications: Warfarin, Digoxin, Cimetidine, Procainamide
Pertinent laboratory exam: INR – 4.5
52. Which of the following could have contributed to the INR result of the patient?
A. Cimetidine C. Chronic alcoholism
B. Procainamide D. Digoxin
MPL: 0.50 REFERENCE: PHARMACOLOGY 4th Ed. Rang,p.374
53. The INR result is brought about by:
A. Increased thrombin activity
B. Decreased platelet activation
C. Diminished levels of vitamin K
c. D. Decreased levels of factors II, VII. IX and X
MPL: 1 REFERENCE: PHARMACOLOGY 4th Ed. Rang,p.316
54. In the event this patient develop massive bleeding, which of the following is the best thing to be done?
A. Discontinue the drug and give vitamin K
B. Discontinue the drug and change to LMWH.
d. C. Discontinue the drug and give protamine sulfate
e. D. Discontinue the drug and transfuse platelets.
MPL: 0.33 REFERENCE: PHARMACOLOGY 4th Ed. Rang,p.319
55. If this patient’s ventricular rate cannot be controlled with digoxin, Which of the following maybe used as an alternative for ventricular rate control?
A. Amlodipine C. Quinidine
B. Atenolol D. Ibutilide
MPL: 0.25 REFERENCE: PHARMACOLOGY 4th Ed. Rang,p.269
56. Which of the following is an adverse effect of procainamide?
A. Hypertension C. drug induced Lupus erythematosus
B. Bradycardia D. Corneal microdeposits
MPL: 0.25 REFERENCE: PHARMACOLOGY 4th Ed. Rang p.229
A 26 year old medical student was brought to the ER for an apparent suicide poisoning due to broken heart. The maid could not recall the
medication but claimed that the bottle contained medicine for headache. On PE, the physician noted cyanosis of the oral mucosa, tongue and nail beds.
57. Which of the following is the most likely drug ingested?
A. Aspirin C. Flurbiprofen
B. Acetaminophen D. Meloxicam
MPL: 0.25
58. This DMARD agent is also an immunosuppressant.
A. Gold C. Chloroquine
B. Penicillamine D. Cyclosporine
MPL: 0.50 REFERENCE: PHARMACOLOGY 4th Ed. Rang p.237
59. A 52 year old woman was suffering from severe joint pains and was diagnosed to have RA. However, she also appears to be suffering from acid
peptic disease. Among the following, which is the safest to give her?
A. Aspirin C. Celecoxib
B. Ibuprofen D. Phenylbutazone
MPL: 0.50
60. Which of the following NSAID has the longest half-life and therefore should be given at a longer dosage interval?
A. meloxicam (20h) C. Tenoxicam (72h)
B. Rofecoxib (17h) D. Piroxicam (50-60h)
MPL: 1 REFERENCE: PHARMACOLOGY 4th Ed. Rang p.230
A 21 year old male after quarrelling with his girlfriend took a bottle of insecticide from their garden approximately 30 –40 cc. He went back to his
room and talked with his girlfriend over the phone. While talking to his girlfriend, he complained of dizziness, blurring of vision, generalized body
weakness, numbness, severe epigastric pain and shortness of breath. He was able to call the attention of his older sister before falling on the floor. He
was found lying on the floor and was noted to be pale, drowsy, with stiffening of the trunk and extremities and soaked with sweat. He was then brought
to the ER. On arrival, he has drooling of saliva with 3 episodes of white viscid material, non-projectile non-bile stained about 2 tbsp. per bout.
61. As an ER physician, which of the following are you going to give the patient?
A. Pilocarpine C. Atropine
B. Physostigmine D. Carbachol
MPL: 0.33 REFERENCE: PHARMACOLOGY 4th Ed. Rang p.122
62. A diabetic patient develops hyperlipedemia (elevated LDL, and triglycerides, normal HDL). Which of the following is NOT appropriate for the
patient?
A. Colestipol C. lovastatin
B. Gemfibrozil D. niacin
MPL: 0.25 REFERENCE: PHARMACOLOGY 4th Ed. Rang p.308

Pharmacology
5 / 68
63 . The patient was noted to have an elevated triglycerides level after treatment with a hypolipidemic agent. This side effect could be secondary to
which of the following?
A. Cholestyramine C. cerivastatin
B. Clofibrate D. niacin
MPL: 0.25 REFERENCE: PHARMACOLOGY 4th Ed. Rang p.307
64. Inhibits de novo synthesis of cholesterol by inhibiting HMG CoA reductase:
A. Atorvastatin C. gemfibrozil
B. Colestipol D. niacin
MPL: 0.50 REFERENCE: PHARMACOLOGY 4th Ed. Rang p.305
65. The patient has been having intermittent episodes of dry cough. The agent that may have probably caused this is ______________:
A. Captopril C. losartan
f. B. candesartan D. telmisartan
MPL: 1 REFERENCE: PHARMACOLOGY 4th Ed. Rang p.292
66. A patient with tachyarrhythmia and hypertension would be given an antihypertensive. What agent would be the choice for this patient?
A. Felodipine C. nifedipine
B. isradipine D. verapamil
MPL: 0.25 REFERENCE: PHARMACOLOGY 4th Ed. Rang p.275
67. β -blockers are given with vasodilators due to the following reasons:
A. reduce renal sodium excretion
B. reduce renin release
C. decrease systemic vascular resistance
g. D. increase cardiac contractility
MPL: 0.50 REFERENCE: PHARMACOLOGY 4th Ed. Rang p.287
68. Which of the following conditions will NOT benefit from the use of VASODILATORS:
A. hypertension C. angina
B. migraine D. peripheral vascular disease
MPL: 0.50 REFERENCE: PHARMACOLOGY 4th Ed. Rang p.290
69. .An asthmatic patient came to the ER because of generalized wheezing in the lungs. Urgent management was done. However after awhile
hypertension was noted. Which of the following medications was probably given?
A. salbutamol C. terbutaline
B. isoproterenol D. salmeterol
MPL: 0.50 REFERENCE: PHARMACOLOGY 4th Ed. Rang P.160
70. .A flight attendant has been having allergic rhinitis. She comes to your clinic asking for medication prescription 2 hours prior her to flight. What will be
appropriate for her?
A. diphenhydramine C. loratadine
B. Chlorpheniramine D. promethazine
MPL: 0.50 REFERENCE: PHARMACOLOGY 4th Ed. Rang p.345
71. It acts by inhibiting the immediate phase of asthma as it prevents mediator release from mast cells
A. cromolyn sodium C. montelukast
B. beclomethasone D. zileuton
MPL: 0.33 REFERENCE: PHARMACOLOGY 4th Ed. Rang p.347
72. An asthmatic patient maintained on oral salbutamol for 8 months, apparently had infrequent asthma attacks for the first 6 months. However, at
present she had increased frequency of asthma attacks. This could be secondary to:
A. inadequate dose C. hypoxemia
B. poor compliance D. tachyphylaxis
MPL: 0.33 REFERENCE: PHARMACOLOGY 4th Ed. Rang p.16
73. Which of the following is NOT a pharmacologic action of Cortisone:
A. anti-inflammatory C. promotes fetal lung maturation
B. immunosuppressant D. adrenal suppression
MPL: 0.33 REFERENCE: PHARMACOLOGY 4th Ed. Rang p.416
74. A patient with severe persistent asthma was treated for 6 months now with prednisone. Which of the following would be the expected side effect?
A. Cushingoid facie C. hyperkalemia
B. hypoglycemia D. hypotension
MPL: 0.50 REFERENCE: PHARMACOLOGY 4th Ed. Rang p.423
75. If a patient after prolonged steroid treatment developed depression. What synthetic corticosteroid agent is he taking?
A. cortisone C. fludrocortisone
B. dexamethasone D. prednisone
MPL: 0.50 REFERENCE: PHARMACOLOGY 4th Ed. Rang p.552
FM, 40 year old male was seen at the clinic for check-up. He has bradykinesia, muscle rigidity, pill rolling movement, resting tremors and shuffling
gait.
76. The single most effective agent in the treatment og this disorder is
A. Levodopa C. Carbamazepine
B. Baclofen D. Tacrine
MPL: 1 REFERENCE: PHARMACOLOGY 4th Ed. Rang p.509
77. Which of the following is NOT TRUE about this drug?
A. In clinical practice, it is almost always used in combination with peripherally acting inhibitor of aromatic L-amino acid
decarboxylase.
B. This drug has a short half-life about 1 – 3 hr.
C. Administration of this drug with meals delay its absorption.
h. D. This drug has very little side effect and can be withdrawn abruptly.
MPL: 0.33 REFERENCE: PHARMACOLOGY 4th Ed. Rang p.510
78. Which of the following is a selective inhibitor of MAO-B
A. Selegiline C. Tolcapone
B. Amantadine D. Bromocriptine
MPL; 0.50 REFERENCE: PHARMACOLOGY 4th Ed. Rang p.511

Pharmacology
6 / 68
GM, a 53 year old male was seen at the OPD for regular check-up. He had a history of type II DM for 5 years and hypothyroidism for 20 years.
Medications: Levothyroxine, Glyburide
79.The patient had a series of laboratory examinations and was noted to have hyperlipedemia. Which of the following drugs can interfere with the
absorption of levothyroxine?
A. Simvastatin C. Gemfibrozil
B. Cholestyramine D. Niacin
MPL: 0.33 REFERENCE: PHARMACOLOGY 4th Ed. Rang p.70
LC, a 25 year old patient was seen at the clinic because of lump on her throat. She has gained 5 kg for the past 6 months, has cold
intolerance, unusual painful heavy menses and constipation. She complained that the lump on her throat interferes with her swallowing.
PMH: iron deficiency anemia of 4 months duration
Medications: Ferrous sulfate, Ibuprofen, Kelp tablets(contains Iodine), Lo-ovral (Ethinyl estradiol and norgestrel) daily
80.What effect does the use of oral contraceptive have on the patient’s hypothyroidism?
A. Low dose oral contraceptives are unlikely to affect her test results.
B. Estrogen may falsely increase TSH due to alterations in TSH secretion
C. Estrogen may falsely decrease FT4I due to alterations in TBG.
D. Estrogen may falsely elevate total T4 levels due to alterations in TBG.
MPL: 0.33 REF. BASIC & CLINICAL PHARMACOLOGY 9th Ed. KATZUNGp.630
81. If LC becomes pregnant, how might her levothyroxine replacement be affected?
A. She may require a 20-30% increase in dose.
B. She may require a 20-30% decrease in dose.
C. She will probably not require any changes in dose.
D. TT4 levels should also be monitored because of changes in TBG.
MPL: 0.33 REF. BASIC & CLINICAL PHARMACOLOGY 9th Ed. KATZUNG p.631
A 62 year old male was seen at the ER because of lightheadedness, palpitation and shortness of breath. He said that the palpitations were
associated with exercise that usually went away with rest. Two days ago, while washing the dishes, he began to have shortness of breath and felt that
his heart was “racing”.
PMH: hypertension x 20 years; Hyperlipidemia for 5 years, and RHD with MVP as a child.
Medications: Lisinopril, Furosemide, Gemfibrozil
82. Which of the following drugs is likely to be least effective in controlling this patient’s rapid ventricular response?
A Digoxin C. Verapamil
B. Diltiazem D. Atenolol
MPL: 0.25 REFERENCE: PHARMACOLOGY 4th Ed. Rang p.265
83. This patient was initially given verapamil for his AF. Due to the addition of this drug, which of the following is the most appropriate treatment of this
patient’s hypertension?
A.Continue lisinopril and furosemide
B. Continue lisinopril and discontinue furosemide
C. Continue lisinopril
D. Continue lisinopril, discontinue furosemide and add hydrochlorothiazide
MPL: 0.25 REFERENCE: PHARMACOLOGY 4th Ed. Rang p.361
A 37 year old Fil-Am was seen at the clinic because of nausea and vomiting. Five days prior to consultation, he had nausea, vomiting, fever
and chills. He took ibuprofen for generalized ache 3 days ago. Recent blood sugar reading was unavailable because he ran out of test strips . On
admission, he was weak looking, severely dehydrated with weak pulses.
PMH: DM type I for 21 years, one episode of DKA 4 months ago; peripheral neuropathy
Medications: Insulin NPH< Ibuprofen, Amitryptilline
84. Which of the following agents is least likely to aggravate the patient’s condition?
A. Furosemide C. Acetaminophen
B. Amitryptilline D. Insulin
MPL: 0.33 REFERENCE: PHARMACOLOGY 4th Ed. Rang p.363
85. Loop diuretics have their principal diuretic effect on:
A. collecting ducts
B. ascending limb*
C. distal convoluted tubules
D. proximal convoluted tubules
MPL: 0.25 REFERENCE: PHARMACOLOGY 4th Ed. Rang p.361
86. Which of the following are potential side effects of thiazide diuretics?
A. hypokalemia, hyperglycemia, hyperlipidemia
B. hypokalemia, ototoxicity, hyperuricemia
C. hyperkalemia, alkalosis, nausea/vomiting
D. hyperglycemia, hypokalemia, metabolic alkalosis*
MPL: 0.33 REFERENCE: PHARMACOLOGY 4th Ed. Rang p.364
87. Patients receiving insulin therapy generally require which of the following interventions with progression to end-stage renal disease?
A. Decrease in total insulin dose C. Increase in frequency of insulin administration
B. Increase in total insulin dose D. No change in insulin regimen
MPL: 0.25 REF. BASIC & CLINICAL PHARMACOLOGY 9th Ed. KATZUNG p.694
BK a 23 year old female seen at the ER because of nausea and vomiting. Myalgia, polydipsia and polyuria. Three days prior to consult, he
attended a party and drank an excessive amount of alcohol. He woke up sick to his stomach, vomited 6x since then and was unable to eat nor drink.
He stopped taking insulin and currently has headache.
PMH: Type I DM for 11 years. Depression and allergic rhinitis
Medications: Human insulin, Sertraline, Fluticasone, Loratidine, Acetaminophen
88. If Bk.’s blood glucose does not decrease by at least 2.8 mg/dl in the first 2 hours, what should be done?
A. Double insulin infusion rate C. Increase the insulin infusion to 0.5 /kg/hr
B. Give insulin bolus dose of 2U/kg D. Check blood glucose again in 2 hours
MPL: 0.50
89. Which of the following statements about mixing NPH and regular insulins is NOT TRUE?
A. The injection must be taken within 10 mins of mixing
B. Regular insulin is drawn up before the NPH dose
Pharmacology
7 / 68
C. Mixtures of NPH and regular insulin are stable in any ratio
D. Mixing insulins allows greater flexibility versus 70/30 insulin
MPL: 0.25
90. Clostridium botulinum toxin produces respiratory paralysis by
A. blocking nicotinic receptors C. causing circulatory collapse
a. B. blocking the release of Ach from nerve endings D. stimulating vagus
MPL; 0.25 REFERENCE: PHARMACOLOGY 4th Ed. Rang p.130
91. A 60 year old male underwent cataract removal. After surgery Ach chloride was administered intraocularly to:
A. Relax the circular muscle of the iris C. decrease tearing from lachrymal secretion
b. B. ensure complete miosis D. decrease the flow of aqueous humor
MPL; 0.50 REFERENCE: PHARMACOLOGY 4th Ed. Rang p.118
92. An elderly patient was diagnosed to have open angle glaucoma. She was given 0.25% pilocarpine 2 gtts every 6 hours. The anticipated effect
after the administration would be
A. relax ciliary muscle
B. improve accommodation
C. relax sphincter muscle of iris
D. contract ciliary muscle and pull on trabecular network to relieve pressure
MPL: 0.33 REFERENCE: PHARMACOLOGY 4th Ed. Rang p.121
93. Tachyphylaxis maybe observed with the use of the following:
A. Propranolol C. Ephedrine
B. Alpha methyl tyrosine D. Phenylephrine
MPL: 0.50 REFERENCE: PHARMACOLOGY 4th Ed. Rang pp.16, 158
94. A 25 year old patient weighing 180 lbs consulted at the clinic. She tried several methods of losing weight but failed. Her height is 170 cm. Based
on her BMI she is considered:
A. healthy C. Obese
B. overweight D. underweight
MPL: 0.33 REFERENCE: PHARMACOLOGY 4th Ed. Rang p.399
95. This inhibitor of GIT lipase is useful in the treatment of obesity and has minimal side effects:
A. sibutramine C. Orlistat
B. Phentermine D. glucosan
MPL: 0.33 REFERENCE: PHARMACOLOGY 4th Ed. Rang p.407
96. A group of students studied the effects of various GI drugs by measuring the effects on gastric pH and volume. The baseline gastric volume was
2cc and when tested with litmus paper, the paper remained pink. After administering drug A, the gastric volume was 2 cc and the litmus paper
turned blue. This means
A. The drug is effective in lowering gastric Ph
c. B. The drug is effective in increasing gastric pH but no effect on the volume
d. C. The drug is effective in lowering the gastric pH and volume
e. D. The drug has no effect at all on volume and ph of gastric juice.
MPL: 0.50 REFERENCE: PHARMACOLOGY 4th Ed. Rang p.375
97. A patient taking this drug came into the clinic due to visual disturbances, constipation, difficulty in urination and dry mouth. This drug could be:
A. muscarinic antagonist C. dopamine D2 antagonist
B. histamine H2 antagonist D. gastrin antagonist
MPL: 0.50 REFERENCE: PHARMACOLOGY 4th Ed. Rang p.122
98. The onset and duration of action of NPH insulin are extended because:
A. protamine decreases the rate at which insulin is absorbed
B. protamine blocks insulin metabolism in the liver
C. protamine is basic and combines with insulin by charge interactions
f. D. protamine is slowly degraded proteolytically releasing the bound insulin
MPL: 0.25 REF. BASIC & CLINICAL PHARMACOLOGY 9th Ed. KATZUNG p.697
99. Which of the following is NOT TRUE about insulin action?
A. It stimulates glycogen synthesis in muscle fiber
B. It inhibits lipolysis in the adipocyte
C. It stimulates K fatty acid synthesis in the hepatocytes
g. D. It stimulates gluconeogenesis in the hepatocytes
MPL: 0.50 REFERENCE: PHARMACOLOGY 4th Ed. Rang p.387
100. Which of the following drugs is most likely to improve gastric emptying in-patients with diabetic gastroparesis?
A. Loperamide C. magnesium hydroxide
B. Cisapride D. Sucralfate
MPL: 0.50 REFERENCE: PHARMACOLOGY 4th Ed. Rang p.381

I. SIMPLE MULTIPLE CHOICE. Select the single best answer.

01. The action of the body on the drug is known as:


A. clinical pharmacology C. pharmacotherapeutics
B. pharmacodynamics D. pharmacokinetics
Ans. D. MPL = 0.5 (Choices: D or B). Recall.

02. The gingival hyperplasia that results from chronic administration of phenytoin, an antiepileptic drug,
is an example of which type of fundamental drug action?
A. stimulation C. irritation
B. depression
Ans. C. MPL = 0.5 (Choices: C or A). Recall.

Pharmacology
8 / 68
03. Which of these fields of pharmacology is concerned with the development of new drugs and the
rational use of drugs?
A. posology C. toxicology
B. pharmacoeconomics D. clinical pharmacology
Ans. D. MPL = 0.5 (Choices: D or A). Recall/Analysis.

04. It refers to the passage of a drug from its site of administration into the plasma:
A. bioavailability C. bioequivalence
B. absorption D. diffusion
Ans. B. MPL = 0.5 (Choices: B or D). Recall.

05. Which of these factors can lead to enhanced or increased absorption of a drug?
A. massage or local application of heat
B. use of epinephrine with local anesthetics
C. diarrhea
D. the co-administration of antacids with tetracycline
Ans. A. MPL = 0.5 (Choices: A or C). Analysis.

06. What will be the consequence of greater binding of the drug to plasma proteins?
A. Its duration of action is shortened.
B. It may require a higher drug dose.
C. Its onset of drug action is hastened.
D. It promotes rapid drug excretion.
Ans. B. MPL = 0.5 (Choices: B or A). Analysis.

07. The highly acidic metabolites of the drug are mainly excreted from the body by which process?
A. filtration C. passive diffusion
B. tubular secretion D. exocytosis
Ans. A. MPL = 0.5 (Choices: B or A). Recall.

08. Which of the following statements about plasma protein binding of drugs is TRUE?
A. Plasma protein binding is a linear process.
B. Plasma protein binding is a nonsaturable process.
C. Hypoalbuminemia results in an increase in the unbound drug.
D. The extent of plasma protein binding is relatively constant & unaffected by disease.
Ans. C. MPL = 0.3 (Choices: C, B, or A). Understanding/Analysis.

09. Of the following isoforms of cytochrome P450 enzymes, which of these is often involved in human metabolism of drugs and involved in drug-drug
interactions?
A. CYP1A2 C. CYP2D6
B. CYP2C9 D. CYP3A4
Ans. D. MPL = 0.5 (Choices: D or C). Recall

10. Which of the following statements about renal excretion of drugs is TRUE?
A. Unbound drug is filtered by the glomerulus.
B. P-glycoprotein transporters in the apical membrane cause drug reabsorption.
C. Urine acidification will result in better excretion of weak acids.
D. Probenecid will increase urinary drug excretion of penicillin.
Ans. A. MPL = 0.5 (Choices: A or B). Recall.

11. Two alpha adrenergic antagonists ( Drugs A and B) decrease BP by the same degree following IV administration at the following doses:
Drug A: 150 mg Drug B: 25 mg
This information implies that Drug A:
A. has a higher therapeutic index than Drug B
B. has a lower bioavailability than Drug B
C. is less efficacious than Drug B
D. is less potent than Drug B
Ans. D. MPL = 0.5 (Choices: D or C) Analysis.

12. If we say that Drug B has a higher therapeutic index than Drug A, it means that Drug B is:
A. more potent C. more efficacious
B. safer D. toxic
Ans. B. MPL = 0.5 (Choices: B or C). Analysis.

13. Aspirin, ethanol, and phenytoin follow zero-order, rather than first-order kinetics. This means that:
A. the clearance of aspirin, ethanol, and phenytoin is constant
B. these drugs are removed at a constant rate independent of their concentration
C. if given in toxic doses, these drugs are easier to eliminate
D. their clearance is unaffected by poor perfusion of the organ of elimination
Ans. B. MPL = 0.3 (Choices: B, A, or D). Analysis.

14. The sudden withdrawal of the chronic administration of the anthypertensive drug, propanolol, can
lead to hypertensive crisis because:
A. the antagonist can induce increase in the number of receptors
B. propanolol induces down-regulation of adrenoceptors
C. of hyporesponsiveness of adrenoceptors
Pharmacology
9 / 68
D. of the development of antibodies against propanolol
Ans. A. MPL = 0.5 (Choices: B or A). Recall/Analysis
Note: Mechanism of down regulation

15. Which of these effects is NOT a consequence of genetic polymorphism in drug metabolism?
A. facial flushing due to a defect in ethanol oxidation
B. hemolytic anemia in G6PD deficient patients given primaquine
C. prolonged apnea in patients with plasma pseudocholinesterase variant following succinylcholine
administration
D. overdose toxicity in patients with impaired hydroxylation of mephenytoin
Ans. B. MPL = 0.5 (Choices: B or C). Recall/Analysis.

16. Which of these is NOT a genetically determined idiosyncratic reaction?


A. Fanconi syndrome with the use of expired tetracycline
B. hepatic porphyria with the use of barbiturates
C. fatal aplastic anemia with chloramphenicol administration
D. malignant hyperthermia with the use of inhalational anesthetics
Ans. A. MPL = 0.5 (Choices: A or C). Recall.
Note: Inhalational anesthetics like dantrolene and penthotal may cause malignant hyperthermia.

17. What type of drug interaction can explain how the combination of trimethoprim-sulfamethoxazole
can increase potency and spectrum of coverage beyond the individual drug effects?
A. additive effect C. potentiation
B. synergistic effect D. antagonism
Ans. B. MPL = 0.5 (Choices: B or C)

18. At which autonomic sites does acetylcholine act as a primary neurotransmitter?


A. sympathetic ganglia
B. parasympathetic postganglionic nerve terminals
C. somatic neuromuscular junction
D. all of the above
E. B and C only
Ans. D. MPL -= 0.5 (Choices: D or E) Recall

19. Which of these statements is NOT characteristic of the parasympathetic nervous system?
A. Ganglia lie very close or are actually embedded within organs innervated
B. It is designed to function usually under conditions of strenuous muscular activity and stress.
C. It is activated during digestion of a meal.
D. It is involved in the conservation of body resources.
Ans. B. MPL = 0.5. Choices: B or C

20. What is the common effect on the peripheral autonomic effector sites when either the muscarinic
(M2) or alpha2 receptors are activated?
A. inhibition of adenylyl cyclase C. opening of sodium channels
B. formation of IP3 and DAG D. increase in intracellular calcium
Ans. A. MPL = 0.5 Choices: A or B. Understanding

21. Stimulation of which of these receptors is most associated with the production of smooth muscle
relaxation?
A. alpha1 C. beta2
B. beta1 D. beta3
Ans. C. MPL = 0.5 Choices C or B. Understanding

22. Which of these pharmacologic drugs does NOT produce mydriasis?


A. isoproterenol C. phenylephrine
B. atropine D. ephedrine
Ans. A. MPL = 0.5. Choices: A or D. Understanding/Analysis

23. In minute doses, epinephrine will produce:


A. marked increased in systolic pressure C. decrease in total peripheral resistance
B. increase in diastolic pressure D. A and B are correct
Ans. C MPL = 0.5 Choices: C or D. Understanding/Analysis

24. Which of these statements is referable to beta2 receptor stimulants?


A. They are effectively metabolized by COMT and MAO.
B. They produce cardiac stimulation similar in degree as epinephrine
C. A common side effect is tremor.
D. They are effectively administered by the oral route only.
Ans. C. MPL = 0.5. Choices: C or D. Understanding/Recall

25. Which of these drugs may be employed in the prophylaxis of migraine headache?
A. ergotamine C. ergonovine
B. metoprolol D. mannitol
Ans. B. MPL = 0.5. Choices: B or A. Recall/ Understanding

26. Why are certain beta adrenoceptor blockers useful in the treatment of open-angle glaucoma?
Pharmacology
10 / 68
A. They decrease the aqueous secretion from the ciliary epithelium.
B. They enhance the outflow of the aqueous humor.
C. They produce ciliary muscle contraction.
D. They deplete extracellular bicarbonates.
Ans. A. MPL = 0.5. Choices: A or B. Understanding

27. In what way is methyldopa different from clonidine?


A. presence of an active metabolite
B. reduces peripheral vascular resistance
C. may cause withdrawal symptoms if stopped abruptly
D. may produce sedation
Ans. A. MPL = 0.5. Choices: A or C. Understanding

28. Which of the following is a selective alpha1 receptor antagonist that is effective for hypertension?
A. ergotamine C. prazosin
B. phentolamine D. tamsulosin
Ans. C. MPL = 0.5. Choices: C or B

29. Which of the following is the most effective agent for a 50 year old bank manager with BPH and
mild hypertension?
A. ergotamine C. terazosin
B. phentolamine D. tamsulosin
Ans. C. MPL = 0.5 Choices C or D.

30. Which of the following is an expected effect of pilocarpine on the eye?


A. relaxation of the smooth muscles of the iris sphincter
B. increased drainage of aqueous humor
C. paralysis of the ciliary muscle
D. increased activity of the ciliary epithelium/body
Ans. B. MPL = 0.3. Choices: B, C, or D. Understanding.

31. Which of the following is a reversible anti-cholinesterase?


A. edrophonium D. all of the above
B. tacrine E. A and C only
C. carbamate
Ans. E. MPL = 0.5. Choices: E or D. Understanding.

32. Which of the following cholinomimetics may be used for paralytic ileus and urinary atony?
A. physostigmine D. all of the above.
B. betanechol E. B and C only
C. neostigmine
Ans. E. MPL = 0.3. Choices: E, B, or C. Understanding.

33. What is the mechanism of pralidoxime?


A. spontaneous hydrolysis of acetylcholinesterase
B. nucleophilic deactivation of acetylcholinesterase enzyme
C. nucleophilic regeneration of acetylcholinesterase enzyme
D. competitive anticholinergic activity at receptor site
Ans. C. MPL = 0.5. Choices: C or B. Recall/Understanding.

34. A 65-year old male has prostate cancer. Which of the following is most useful in its management?
A. fluoxymesterone C. tamoxifen
B. intermittent infusions of leuprolide D. flutamide
Ans. D.

35. Intractable vomiting induced by cancer chemotherapy is best managed with:


A. GM-CSF C. Leucovorin
B. ondansetron D. dexrazoxane
Ans. B

36. The following antianginal drugs are vasodilators, EXCEPT:


A. felodipine C. metoprolol
B. nitroglycerin D. Diltiazem
Ans. C

37. The nitrate with the longest half-life from among the following is:
A. oral nitroglycerin C. isosorbide mononitrate
B. sublingual nitroglycerin D. isosorbide dinitrate
Ans. C.

38. Captopril and enalapril do all of the following, EXCEPT:


A. inhibit an enzyme, peptidyl dipeptidase
B. competitively block angiotensin II at its receptor
C. decrease angiotensin II concentration in the blood
D. increase bradykinin concentration in the blood
Ans. B.
Pharmacology
11 / 68

39. Which of the following groups of antihypertensive drugs is contraindicated in a pregnant hypertensive?
A. beta blockers C. ACE inhibitors
B. calcium channel blockers D. thiazide diuretics
Ans. C.

40. Hyperuricemia and hypercalcemia are side effects of the following diuretics?
A. furosemide C. spironolactone
B. hydrochlorothiazide D. Amiloride
Ans. B.

41. Which of the following drugs have been shown to retard deterioration of left ventricular function and prolong survival of patients with heart failure?
A. enalapril C. dobutamine
B. furosemide D. milrinone
Ans. A.

42. An antiarrhythmic drug with a half-life of 15-20 seconds that necessitates its being given by rapid IV bolus:
A. amiodarone C. esmolol
B. adenosine D. lidocaine
Ans. B.

43. To induce ovulation after stimulating follicular growth in anovulatory women, which of the following is preferred?
A. chorionic gonadotropin C. hMG
B. FSH D. GnRH
Ans. A.

44. The longest-acting glucocorticoid of the following is:


A. prednisone C. hydrocortisone
B. betamethasone D. methylprednisolone
Ans. B.

45. Which of the following steroids has more salt-retaining than anti-inflammatory effects?
A. dexamethasone C. prednisolone
B. fludrocortisone D. triamcinolone
Ans. B.

46. Insulin is the hypoglycemic agent of choice in the treatment of diabetic patients who are:
A. obese and with type 2 DM C. pregnant
B. non obese and with type 2 DM D. newly diagnosed
Ans. C.

47. PTU differs from methimazole in that PTU:


A. is more potent on a weight basis
B. crosses the placental barrier more readily
C. inhibits peripheral conversion T4 to T3
D. all of the above
Ans. C.

48. An antiestrogen used in the treatment of breast CA:


A. flutamide C. ethylestrenol
B. cyproterone D. tamoxifen
Ans. D.

49. Which drug is used as an antiretroviral agent?


A. oseltamivir C. indinavir
B. famciclovir D. ribavirin
Ans. C. MPL = 0.5 (Choices: C or B)

50. A 52 year old patient with a BP of 170/98 and mild wheezing develops increased wheezing after
medication. He was probably given:
A. verapamil C. methyldopa
B. propranolol D. hydrochlorothiazide
Ans. B.

51. A 52 year old female who had been admitted for acute pulmonary edema complained of tinnitus and
hearing loss after one week of treatment, which of the following could have produced this problem?
A. triamterene C. furosemide
B. spironolactone D. ethacrynic acid
Ans. D.

52. An analog of GnRH useful in advanced prostatic carcinoma:


A. adrenocorticosteroids D. leuprolide
B. androgens E. tamoxifen
C. flutamide
Ans. D.

Pharmacology
12 / 68
53. The major danger in the tight control of diabetes is :
A. nephropathy C. ketoacidosis
B. neuropathy D. hypoglycemia
Ans. D.

54. A hyperthyroid patient on carbimazole develops a sore throat and fever. Throat is congested with
pustules. WBC=3000/cu mm. You should:
A. maintain carbimazole and start antibiotics
B. discontinue carbimazole and start antibiotics
C. continue carbimazole alone
D. increase carbimazole and add antibiotics
Ans. B.

55. Following exposure to chicken pox, which of the four household members who never had chicken
pox should receive immunoglobulin?
A. the 7 year old apparently normal sibling
B. the 32 year old breadwinner father who wishes not to be absent from work
C. the 18 year old single “yaya”
D. the 29 year old pregnant housewife mother
Ans. D.

56. Chloroquine as an amebicide is specially useful in:


A. mild amebic colitis C. hepatic amebiasis
B. asymptomatic cyst passers D. extrahepatic amebiasis
Ans. C. MPL = 0.5 (Choices: C or D)

57. The most effective antitussive against which all other antitussives are measured.
A. codeine C. guaifenesin
B. bromhexine D. S-carboxymethylcysteine
Ans. A.

58. Which of the following is a selective T-cell suppressant?


A. azathioprine C. methotrexate
B. interferon D. tacrolimus
Ans. D.

59. The following drugs may stimulate immune response, EXCEPT:


A. levamisole C. interferon
B. BCG D. mycophenolate mofetil
Ans. D.

60. Which of the following statements about digitoxin is TRUE?


A. It is poorly absorbed from the GIT
B. It is mainly excreted through the kidneys.
C. It undergoes significant enterohepatic circulation.
D. It has faster onset of action than digoxin.
Ans. C.

61. The diuretics useful in lowering CSF pressure and intraocular fluid pressure are the:
A. potassium-sparing diuretics C. thiazides
B. osmotic diuretics D. loop diuretics
Ans. B.

62. Which treatment is indicated for the ablation of residual thyroid tissue after thyroidectomy for malignancy?
A. thyroid hormones C. radioiodine
B. stable iodine D. antithyroid drugs

63. For herpes virus encephalitis, the drug of choice is:


A. acyclovir C. ganciclovir
B. valacyclovir D. famciclovir
Ans. A.

64. The effects of neuromuscular blockers may be enhanced by:


A. alcohols C. aminoglycosides
B. thiopental D. all are correct
Ans. C. MPL = 0.5. Choices: C or B. Understanding

65. Double burst stimulation shows the presence of residual neuromuscular blockade. What drug/s can be
given to reverse this remaining residual block and counter some of the side effects?
A. pyridostigmine + neostigmine C. pyrimidine + scopolamine
B. neostigmine + atropine D. edrophonium only
Ans. B. MPL = 0.3. Choices: B, A, or C. Recall

66. Antimicrobials which inhibit nucleic acid synthesis include:


A. aminoglycosides and tetracycline C. penicillins and cephalosporins
B. quinolones and rifampicin D. ziduvidine and acyclovir
Pharmacology
13 / 68
Answer: B. MPL = 0.5 (B or A).

67. Which beta-lactamase inhibitor is rationally combined with ampicillin, producing potentiation of its action?
A. clavulanic acid C. cloxacillin
B. sulbactam D. tazobactam
Answer: B. Choices: B or A. MPL = 0.5. Understanding/Recall.

68. Intravenous amikacin, an aminoglycoside, is NOT useful for meningitis due to Neisseria meningitides because:
A. its spectrum of activity is only intermediate
B. it is ineffective against gram negative bacteria
C. it has poor CNS penetration
D. it is effective only against anaerobes
Answer: C. MPL = 0.5 (C or A)

69. Ron Co, a 9 month old infant developed cough and fever of 5 days. He sought consultation in your clinic and you noted that he is not in respiratory
distress. There were fine rales in the right lower lung. His mother has given him amoxicillin on the second day of illness but there was no relief. The
following are alternative drug choices for Ron EXCEPT:
A. cephalexin C. cefuroxime
B. cefaclor D. chloramphenicol
Answer: A. MPL = 0.5 (A or C).

70. Clinical indication/s for fluoroquinolones include/s:


A. urinary tract infection C. typhoid fever
B. Shigella dysentery D. all of the above
Answer: D/ MPL = 0.5 (Choices: A or D)

71. If a hospitalized patient developed a culture-proven gram negative septicemia due to Pseudomonas
aeruginosa, which of the following drug combinations would be MOST EFFICACIOUS?
A. ceftazidime-amikacin C. ceftazidime-vancomycin
B. ceftazidime-cefuroxime D. ceftazidime-metronidazole
Ans. A. MPL = 0.5. Choices: A or B. Understanding.

72. The following is a semisynthetic penicillin with coverage for gram negative organisms:
A. penicillin G C. cloxacillin
B. benzathine penicillin D. piperacillin
Ans. D. MPL = 0.5 (D or C). Recall.
ITEM ANALYSIS: GOOD QUESTION

73. Which of the following is NOT a quinolone?


A. ciprofloxacin C. clarithromycin
B. ofloxacin D. nalidixic acid
Ans. C. MPL = 0.5 (C or D). Recall.

74. Hyperuricemia and hepatotoxicity are adverse effects of which anti-TB agent?
A. isoniazid C. pyrazinamide
B. rifampicin D. streptomycin
Answer: C. MPL = 0.5 (Choices: C or D)

75. The best agent for slowly multiplying TB bacillin inside macrophages is:
A. PZA C. INH
b. RIF D. STM
Answer: A. MPL = 0.5 (Choices: A or D)

76. The bacterial cell wall enzyme inhibited by INH is:


A. mycolase synthetase C. cytochrome P450
B. RNA polymerase D. glucoronyl transferase
Answer: A. MPL = 0.5 (Choices: A or B)

77. The antituberculous drug that is effective for Hansen’s disease and for prophylaxis for persons
exposed to meningococcemia is:
A. isoniazid C. pyrazinamide
B. rifampicin D. streptomycin
Answer: B, MPL = 1.0

78. On regular follow-up for tuberculosis, you noticed that the visual acuity of Vilma worsened from 20/20 to 20/50 in both eyes after only three weeks of
HRPE. Your approach in management would be to discontinue:
A. all of the 4 drugs and reintroduce one drug at a time
B. ethambutol and refer to an ophthalmologist
C. pyrazinamide and refer to an ophthalmologist
D. ethambutol, reintroduce it after 1 week, and refer to an ophthalmologist
Ans. B. MPL = 0.5 (B or D). Application.

79. The antimalarial agent that is effective in the exoerythrocytic cycle thereby producing radical cure of vivax infection is:
A. primaquine C. artemisinin
B. chloroquine D. quinine
Answer: A. MPL = 0.5 (Choices: A or D)
Pharmacology
14 / 68

80. The following statement is TRUE about diloxanide furoate:


A. it is a effective for extraintestinal amoebiasis
B. it is the agent of choice for hepatic abcess
C. it is effective for asymptomatic cyst passers
D. it is safe for infants and pregnant patients
Ans. C. MPL = 0.5 (A or C). Recall.

81. Which of the following antihelminthics is effective for Schistosoma japonicum?


A. mebendazole C. diethylcarbamazine citrate
B. piperazine D. praziquantel
Answer: D. MPL = 0.5 (Choices: D or C). Recall.

82. If the agents below are available in the nearby drug store, the efficacious agent and affordable agent
for a school teacher with pytiriasis versicolor is:
A. selenium sulfide C. amphothericin b
B. fluoconazole D. griseofulvin
Ans. A. MPL = 0.5 (A or B). Recall.

83. Hepatitis B vaccine is LEAST likely to be recommended in which of the following patients?
A. medical interns C. alcoholic fond of eating raw fish
B. addicts who are mainliners D. neonates
Ans: C. MPL = 0.5 (Choices: C or B)

84. Which of the following is a/are live vaccine/s?


A. measles C. oral polio
B. rubella D. all of the above
Ans. D. MPL = 0.5 (Choices: D or A)

85. The most important mechanism for the prokinetic activity of metoclopromide is through its:
A. action on motilin receptors
B. antagonism of the D2 receptors on the area postrema
C. serotonin 5-HT4 receptor antagonism
D. histamine receptor occupation
Answer: B. MPL = 0.5. (Possible confounder: C)

86. Nocturnal gastric acid secretion is largely stimulated by:


A. histamine C. acetylcholine
B. gastrin D. A and B only.
Answer. A. MPL = 0.5 (Choices: A or D). Recall.
H2 antagonists are especially effective at inhibiting nocturnal acid secretion (which depends largely on histamine) but have a modest impact on meal-
stimulated acid secretion (which is stimulated by gastrin and acetylcholine as well as histamine). Thus they block more than 90% of nocturnal acid but
only 60-80% of daytime acid secretion. (Katzung, 9th edition, p. 1036)

87. The following statement is TRUE of omeprazole:


A. It is a nitroimidazole C. It has a poor safety profile
B. It is administered as a prodrug. D. Its bioavailability is enhanced by food
Answer: B. MPL = 0.5 (Choices: B or C). Analysis.
Since their introduction in the late 1980s, these (proton pump inhibitors) efficacious acid inhibitory agents have rapidly assumed the major role for the
treatment of acid-peptic disorders. They are now among the most widely selling drugs worldwide due to their outstanding efficacy and safety… All
(proton pump inhibitors) are substituted benzimidazoles that resemble H2 antagonists in structure… Proton pump inhibitors are administered as inactive
prodrugs… Their bioavailability is decreased approximately 50% by food; hence the drugs should be administered on an empty stomach. (Katzung, 9th
edition, p. 1039)

88. Proton pump inhibitors have found clinical use in the following conditions EXCEPT:
A. gastroesophageal reflux disease C. H-pylori-associated ulcers
B. NSAID-associated ulcers D. chemotherapy-induced emesis
Answer: D. MPL = 0.5 (Choices: D or B). Recall.
Proton pump inhibitors are the most effective agents for the treatment of non-erosive and erosive reflux disease, esophageal complications of reflux
disease (peptic stricture or Barrett’s esophagus), and extraesophageal manifestations of reflux disease… The most effective regiments of H. pylori
eradiation are combinations of two antibiotics and a proton pump inhibitor… For patients with ulcers caused by aspirin or other NSAIDs, either H2
antagonists or proton pump inhibitors provide rapid ulcer healing so long as the NSAID is discontinued; continued use of the NSAID impairs ulcer
healing. Treatment with once daily proton pump inhibitor promotes ulcer healing despite continued NSAID therapy. Proton pump inhibitors are also
given to prevent ulcer complications from NSAIDs. (Katzung, 9th edition, pages 1040-1041)

89. Which of the following agents for inflammatory bowel disease is activated by the cleavage of an azo
bond?
A. mesalazine C. prednisolone
B. azathioprine D. infliximab
Answer: A. MPL = 0.5 (Choices: A or B). Recall.

90. Which antimicrobial is the most cost-effective for the treatment of pseudomembranous colitis in a patient whom you know is not wealthy?
A. ampicillin C. metronidazole
B. clindamycin D. vancomycin
Answer: C. MPL = 0.5 (Choices: C or D). Analysis.

Pharmacology
15 / 68
91. Which of the following drugs may be given to chronic renal failure patients to stimulate erythroid proliferation and differentiation?
A. cyanocobalamin C. ferrous sulfate
B. folic acid D. recombinant human erythropoeitin
Ans: D. MPL = 0.5 (Choices: D or B).

92. Which of the following is a polymer of MW 2000-6000 that has more effect on factor X than on prothrombin?
A. heparin C. potamine
B. enoxaparin D. warfarin
Ans. B. MPL = 0.5 (Choices: B or A)

93. Which of the following is efficacious for the rapid reversal of warfarin effects?
A. Vitamin K C. prothrombin
B. whole plasma D. protamine
Ans: B. MPL = 0.5 (Choices: B or A)

94. The following drugs may cause folic acid deficiency by interfering with its absorption EXCEPT:
A. phenytoin C. oral contraceptives
B. isoniazid D. trimethoprim
Ans. D, MPL = 0.5 (Choices: D or B)
TMP and pyrimethamine may cause folic acid deficiency because of inhibition of DHFR, not because of interference with absorption.

95. What is the mechanism of streptokinase?


A. blocks the gamma-carboxylation of glutamate residues in Vitamin K-dependent clotting factors
B. catalyzes the formation of the serine protease plasmin
C. indirect thrombin inhibition
D. specific irreversible thrombition inhibition
Ans. B. MPL = 0.5 (Choices: B or C)
A is the mechanism of action of warfarin, C that of heparin, and D of hirudin.

96. Which of the following may cause cutaneous vasodilatation?


A. lovastatin C. cholestyramine
B. gemfibrozil D. niucotinic acid
Ans. D. MPL = 0.5 (Choices: D or C)
Cutaneous vasodilatation is prevented by giving aspirin.

97. Which of the following inhibits lipoprotein lipase?


A. lovastatin C. atorvastatin
B. gemfibrozil D. cholestyramine
Ans. B. MPL = 0.5 (Choices: B or D)

98. The following may be given to a pregnant mother in premature labor to prevent respiratory distress syndrome to the baby upon delivery:
A. dexamethasone C. surfactant
B. isoxsuprine D. all of the above
Answer: A, MPL – 0.5 (choices: A or C), RECALL

99. A 25 year old female was left by her father for a few minutes in their airconditioned car while the engine was running. She was found apparently
confused but complaining of headache. and visual problems. She was tachycardic. Her condition is most probably due to:
A. nitrogen oxide poisoning C. carbon monoxide poisoning
B. diabetic ketoacidosis D. hypoglycemia
Ans. C. MPL = 0.5 (Choices: C or A)

100. Which of the following is NOT a clinical sign associated with chemical warfare agents such as sarin
or tabun?
A. bronchosconstriction C. diarrhea
B. urinary incontinence D. pinpoint pupils
Ans. A. MPL = 0.5 (Choices: A or C)
The signs and symptoms are due to excess acetylcholine, with the pnemonics DUMBELS.

BOARD REVIEW QUESTIONS

1. The action of a drug on the body, including receptor interactions, dose receptors phenomena and mechanism of therapeutic and toxic
action is:
A. toxicology C. pharmacogenetics
B. pharmacokinetics D. pharmacodynamics***

2.True of metabolism:
A. aims to make the drug more polar and readily excretable***
B. aims to make the drug more effective
C. aims to make the drug stay in the body longer
D. aims to make the drug less toxic

3. If the plasma concentration of drug declines with “first order kinetics” this means that:
A. there is only one metabolic path for drug disposition
B. the half life is the same regardless of the plasma concentration***
C. the drug is largely metabolized in the liver after oral administration and has low bioavailability
Pharmacology
16 / 68
D. the drug is not distributed outside the vascular system

4.Distribution of drugs in specific tissues:


A. is independent of blood flow to the organ
B. is independent of the solubility of the drug in that tissue
C. depends on the unbound drug concentration gradient between blood and the tissue***
D. has no effect on the half life of the drug

5.A drug that blocks the action of epinephrine at its receptors by occupying those receptors without activating them
A. pharmacologic antagonism***
B. physiological antagonism
C. partial antagonist
D. chemical antagonist

6. A drug that binds a reception to produce an effect is called:


A. Agonist*** B. Antagonist C. pro-drug D. metabolite

7. Phase of clinical trial that involves “many doctors & many patients”:
A. PHASE I B.PHASE II C.PHASE III*** D.PHASE IV

8. The main excitatory transmitter in the CNS s:


A. Glutamate*** B. aspartate C. glycine D. GABA

9. A benzodiazepine which is the drug of choice for status epilepticus:


A. Flurazepam B. triazolam C. diazepam*** D. midazolam

10. Benzodiazepines binding to the GABA receptors leads to:


A. Increased frequency of GABA chloride channel opening***
B. Prolonged duration of GABA chloride channel opening
C. Decreased frequency of GABA chloride channel opening
D. Shortens duration of GABA chloride channel opening

11.Effects of hypnotic sedatives on sleep, EXCEPT:


A. decreased sleep latency
B. decrease duration of slow wave sleep
C. increase duration of NREM stage 2 sleep
D. increase duration of REM sleep***

12. True regarding Ethosuximide:


A. inhibits sodium channel
B. drug of choice in absence seizures***
C. causes neural tube defects
D. all of the above

13. Given orally it causes sedation and also ‘resets the biological clock
being used for the purpose to counter jet lag:
A. nitric oxide B. melatonin*** C. purines D. arachidonic acid

14. Useful for manic-depressive patients; it has its effect by altering intracellular concentration of inositol triphosphate:
A.imipramine B. fluoxetine C. isocarboxacid D. lithium salts***

15.The general mechanism of action of neuroleptic drug is:


A. inhibition of GABA receptors
B. increase dopamine activity
C. decrease dopamine activity***
D. Increase lipid solubility of the drugs

16. This nonselective adrenergic agonist is an effective bronchodilator agent, it is now considered as the drug of choice for anaphylactic shock:
A. epinephrine*** C. tertabulline
B. isoproterenol D. metaproterenol

17. This mucolytic agent is also used as an antidote for paracetamol poisoning:
A. Carbocysteine B. acetylcysteine**** C. bromhexine D. ambroxoll

18.Isoniazid is recommended as a “prophylactic agent for the following, EXCEPT:


A. Who are close contacts of an active case of tuberculosis
B. in children whose PPD test is positive within 2 years after a documented negative skin test(recent converters)
C. who are immunocompromised esp HIV infected and AIDS patients
D. children who are malnourished or with no vitamin supplement****

19.This is the most significant major side effects of isoniazid


A. Hepatitis*** C. deafness
B. retro bulbar neuritis D. hemolysis

Pharmacology
17 / 68
20. Drug A causes a decrease of blood pressure from 10 –20 mm Hg
Drug B causes a decrease of blood pressure from 5 –10 mm Hg
Drug C at 5 mg can lower the blood pressure from 10- 20 mm Hg
Drug D at 10 mg can lower the blood pressure from 5 –10 mm Hg
Therefore:
A. Drug A is more effective than drug B
B. Drug C is more potent than drug D
C. Drug B and D are equally effective***
D. all of the above

21.Which one of the following is NOT an example of drug-drug interactions?


A. drowsiness caused by diphenhydramine***
B. enhanced toxicity of cardiac glycosides when given with thiazide diuretics
C. an increase in the metabolism of oral anticoagulants after phenobarbital administration
D. increased central nervous system depression on administration of both a barbiturate and narcotic analgesic

22.Given prior to stage performance, this relieves the symptoms of anxiety such as tremors & tachycardia:
A. buspirone C. diazepam
B. propranolol*** D. phenobarbital

23. Displacement of this drug from plasma protein binding sites by anticoagulants increases its adverse effects, which include nystagmus, diplopia and
ataxia & gum hyperplasia:
A. valproic acid C. Phenytoin***
B. Carbamazepine D. lamotrigine

24. Phenytoin toxic level is above:


A. 8 mg/dl C. 40 mg/dl***
B. 20 mg/dl D. 100 mg/d

25 Effects of dopamine on the nigrostriatal system is associated with:


A. emotions C. endocrine effects
B. motor control*** D.. eating control

26. This causes the highest incidence of Extra pyramidal effects:


A. chlorpromazine C. haloperidol***
B. thiothexine D. clozapine

27. Indicated for osteomyelitis because of its well bone penetration:


A. Chloramphenicol C. Spectinomycin
B. Clindamycin*** D. gentamycinl

28.Toxic effects of Chloramphenicol except:


A. GIT disturbances C. bone marrow depression
B. Vaginal candidiasis*** D. Gray baby syndrome

29.The following agents primarily act on the 30 S ribosomal subunit except:


A. Chlortetracycline C. amikacin
B. Netilmicin D. chloramphenicol***

30. The parent compound of all semi synthetic penicillin:


A. 6 – aminopenicillanic acid*** C. beta lactam ring
B. 7-aminocephalosporanic acid D. penicilloic acid

31.A fourth generation cephalosporin:


A. Cefpirome *** B. Cephalexin C. Cefuroxime D. Trimethoprim

32.The following inhibit cell wall synthesis except:


A. Vancomycin C. Amocycillin
B. Clindamycin*** D. Cefuroxime

33.The cephalosporins containing methyl thiotetrazole group include the following, except:
A. Cefuroxime*** B. Cefotetan C. Cefamandole D. Cefoperazone

34.The following are penicillinase resistant except:


A. Cloxacillin B Nafcillin C. Amoxicillin*** D. Methicillin

35. The following are the actions of beta lactams except:


A. Inhibits transpeptidation
B. activation of PBPs
C. activation of autolysins
D. inhibits translocation***

36.The only agent in the 2nd generation cephalosporins which can produce sufficient CSF level:
A. Cephalexin C. Cefprom
B. Cefoperazone D. Cefuroxime***

Pharmacology
18 / 68
37.Toxicity that is more pronounced with doxorubicin than with other agents include this potentially irreversible cumulative, dose related :
A. cardio toxicity*** C. neurotoxicity
B. nephrotoxicity D. pulmotoxicity

38.An estrogen receptor antagonist which is useful for post-menopausal breast cancer is:
A. megastrol C. flutamide
B. tamoxifen *** D. leuprolide

39.The major site of alkylation within DNA of alkylating agents is the ____ position of guanine:
A. N1 C. N5
B. N 3 D. N7***

40.TRUE regarding vincristine, EXCEPT


A. a spindle poison
B. useful in acute leukemia in children
C. arrest mitotic cycle
D. causes marked bone marrow suppression***

41.This plant alkaloid is popularly used in ovarian and advanced breast cancer; comesfrom Western yew and is a mitotic spindle poison:
A. etoposide C. Paclitaxel***
B. vincristine D. vinblastine

42. Androgens, estrogens and adrenorecortical hormones all can produce:


A. Cardio toxicity C. bone marrow suppression
B. fluid retention *** D. nausea and vomiting

43.This antileprosy drug is useful for both multibacillary and paucibacillary type; may cause hemolysis n G6PD deficient patient:
A. clofazimine C. Sulfones***
B. Rifampin D. Ofloxaci

44.An adverse effect of sulfonamides in newborns due displacement of bilirubin from binding sites on albumin:
A. Photosensitivity C. Hemolytic anemia
B. Crystalluria D. Kernicterus***

45.This sulfonamide penetrates ocular tissues and fluids at high concentrations thus is useful in chlamydia trachomatis infection:
A. Sulfacetamide*** C. Sulfathiazole
B. Sulfadoxine D. Sulfamethoxazole

46.This alkylating related drug is useful in Hodgkin’s lymphoma. It has leukemogenic, teratogenic and mutagenic activity. It has a disulfiram like effect
and has a MAO inhibitor as its metabolite:
A. nitrogen mustard C. Busulfan
B. procarbazine*** D. melphalan

47.This anticancer is also useful in the treatment of rheumatoid arthritis; its adverse effects can be revered by administration of leucoverin:
A. cyclophosphamide C. methotrexate***
B. cytarabine D. Melphalan

48.Parkinson’s Disease can be managed with these dopamine agonist except:


A. bromocriptine C. pergolide
B. ropinirole D. amantadine***

49.This cholesterase inhibitor is useful for minizing violent behaviors in Alzheimer’s Disease:
A. physostigmine C. arecoline
B. neomycin D. donezepil***

50.The following statements are true in the use of carbidopa in combination with levodopa; EXCEPT:
A. reduces the dose needed by about ten fold
B. diminishes the peripheral side effects
C. conversion to dopamine is largely prevented
D. carbidopa rapidly penetrates the brain ***

51. The risk of ischemic heart disease is directly proportional to the concentration of:
A. LDL cholesterol*** C. triglycerides
B. HDL cholesterol D. VLDL cholesterol

52.MG has an elevated serum cholesterol but with normal triglycerides level. You would prescribed the following:
A. clofibrate*** C. Simvastatin
B. gemfibrozil D. all of the above

53.ZZ has an elevated serum cholesterol and triglycerides. Your choice of hypo lipid agent for ZZ include:
A. clofibrate C. niacin
B. atorvastatin*** D. all of the above

54. You have prescribed atorvastatin to Mr. Park for his hyperlipedemia. You should advise him to take the drug:
A. first thing in the morning C. after breakfast
B. last thing before going to bed*** D. any time of the day
Pharmacology
19 / 68

55.Atorvastatin causes the following effects EXCEPT:


A. decrease hepatic cholesterol synthesis
B. increase synthesis of LDL receptors
C. decrease clearance of LDL***
D. decrease concentration of LDL cholesterol in plasma

56.Drugs that can be used for the prevention of malaria in travelers, EXCEPT:
A. Chloroquine C. mefloquine
B. quinine*** D. doxycyline

57.This is an effective luminal amebecidal agent but is not active against tissue trophozoite:
A. emetine HCl C. chloroquine
B. metronidazole D. diloxanide furoate***

58.This is a broad-spectrum oral antihelminthis which is the drug of choice for the treatment of hydatid disease and cysticercosis:
A. Mebendazole C. thiabendazole
B. albendazole*** D. metronidazole

59.Which of the following antimalarial drugs causes a dose-dependent toxic state that includes: flushed and sweaty skin, dizziness, nausea,
diarrhea, tinnitus, blurred vision, and impaired hearing?
A. Amodiaquine C. pyrimethamine
B. Primaquine D. quinine***

60. Antihypertensive agent of choice in a patient with diabetes mellitus and protenuria:
A. beta blocker C. diuretics
B. ACE inhibitors*** D. calcium channel blockers

61. OF the insulin secretagogues, this is least that cause hypoglycemia as its adverse effect:
A. nateglinide*** C. glifizide
B. miglitide D. glibenclamide

62. A short acting insulin that can be given intravenously:


A. insulin lispro C. insulin ultralente
B. regular insulin*** D. insulin glargine

63. Mechanism of action of nitrates in smooth muscles:


A. activated adenyl cyclase
B. inactivates adenyl cyclase
C. activates guanylyl cyclase***
D. inactivated guanylyl cyclase

64. The following are drugs with positive inotropic effect, EXCEPT:
A digoxin B. hydrochlorothiazide*** C. amrinone D. dobutamine

65. A 21 year old male sustained a severe head in a car accident and found to have an elevated intracranial pressure. CT scanning
showed no intracranial hemorrhage. You would start on:
A. amiloride*** B. mannitol C. furosemideD. benzthiazide

66. Most common adverse effect associated with diuretics:


A. hypotension C. anorexia
B. fluid and electrolyte imbalance*** D. nausea and vomiting

67.Hypokalemia is an adverse effect with the following, Except:


A. spironolactone*** C. furosemide
B. acetazolamide D. hydrochlorothiazide

68. This binds to a proton pump, H+/K+ATPase located in the luminal membrane of the parietal cells and inhibits it irreversibly:
A. ranitidine C. omeprazole***
B. Sucralfate D. misoprostol

69. This nonselective cholinergic blocking agent is an antidote for organophosphate pesticides poisoning:
A. trimethaphan C. tubocurarine
B. nadolol D. atropine***

70.This is the metabolic precursors of noradrenaline and adrenaline which functions as a neurotransmitter in its own right in the brain and possibly also
in the periphery:
A. norepinephrine C. dopamine***
B. epinephrine D. isoprenaline

71 The cholinesterase inhibitor that is used in the diagnosis of myasthenia gravis is:
A. edrophonium chloride*** C. physostigmine salicylate
B. malathion D. ambinonium hydrochlorid

72. The cholinomimetic that is useful for treating postoperative abdominal distention and gastric atony is:
A. Acetylcholine C. Carbachol
Pharmacology
20 / 68
B. Methacholine D. bethanechol***

73. This drug has a very high affinity for the phosphorus atom in parathion and is often used to treat insecticide toxicity:
A. Tubocurarine C. scopolamine
B. Trimethaphan D. pralidoxime***

74. Important characteristics of non-depolarizing blocking agents:


A. It is reversible by anticholinesterase drugs***
B. Produces initial fasciculation and often postoperative muscle pain
C. It is hydrolyzed by plasma cholinesterase and is normally very short-acting
D. All of the above

75. This IV anesthetic is moderately rapid in its onset and recovery; causes cardiovascular stimulation and increased cerebral blood flow. Emergence
reactions impair recovery:
A. Etomadate C. midazolam
B. ketamine *** D. propolol

76. This inhalational anesthetics has a low blood: gas partition coefficient but is not used for induction of anesthesia because of its
pungency, it causes patient to hold their breath?
A. sevoflurane C. Halothane
B. Isoflurane D. Desflurane***

77. Most serious toxic reactions to local anesthetics are due to:
A. convulsions from excessive blood levels ***
B. conversion of hemoglobin to methemoglobin
C. its direct effect on the cardiac muscles
D. its toxicity to nerve muscles

78.This is not an action of morphine:


A. analgesic effect due to stimulation of mu receptors
B. Sedations and euphoria due to the stimulation of mu receptors
C. antitussive effects due to depression o the medullary cough center
D. decrease in circular muscle tone***

79. The clinical use of disulfiram depends on its ability to inhibit thisenzyme:
A. adenylyl cyclase
B. cytochrome P450
C. alcohol dehydrogenase
D. aldehyde dehydrogenase***

80. This vasoactive peptide dilates arterioles, contracts veins, intestinal, and bronchial smooth muscle, causes diuresis, and is a transmitter in sensory
pain neurons:
A. endothelins C. substance P***
B. bradykinin D. vasoactive intestinal peptide

81. TRUE regarding heparin, EXCEPT:


A. primarily given by parenteral route
B. used in pregnancy
C. liver is site of action***
D. activates anththrombin III

82. This thrombolytic agent is obtained from bacterial cultures; it forms a complex with endogenous plasminogen and provides the conversion
of plasminogen to plasmin:
A. streptokinase*** C. warfarin
B. urokinase D. t-PA

83.The effects of aspirin does not include:


A. reduction of fever
B. reduction of prostaglandin synthesis in inflamed tissues
C. reduction of bleeding tendency***
D. impaired auto regulation of kidney function

84. Actions of thyroxine do not include:


A. acceleration of cardiac rate
B. increased appetite
C. decreased glomerular filtration rate***
D. stimulation of oxygen consumption

85. The following are effects of the glucocorticoid, EXCEPT:


A. altered fat deposition
B. inhibition of leukotriene synthesis
C. increased blood glucose
D. increased skin protein synthesis***

86. All of the following are recognized effects of combined oral contraceptives, except:
A. breakthrough bleeding
Pharmacology
21 / 68
B. increased risk of ovarian cancer***
C. decreased risk of endometrial cancer
D. increased risk of ischemic stroke

87.This guanosine analog is used for treatment of mucocutaneous and genital herpes lesion and for prophylaxis in AIDS:
A. acyclovir*** C. Zidovudine
B. Amantadine D. Interferon

88. Successful therapy of congestive heart failure with digoxin will result in which one of the following?
A. decreased heart rate***
B. increased after load
C. increased renin secretion\increased after load
D. increased sympathetic outflow to the heart

89. Succinylcholine are peripherally acting skeletal muscle relaxants that are sometimes used to:
A. treat trauma to the spinal cord
B. treat neuromuscular diseases
C. act as adjuncts to general anesthesia***
D. relieve spasms from traumatic injuries

90. Which of the following statements is true with regards to the therapeutic uses of iron:
A. enteric coated tablets are better absorbed than the immediately released tablet
B. ferrous and ferric salts differ in their bioavailability
C. oral ferrous sulfate is the treatment of choice for iron deficiency***
D. medicinal iron preparations are relatively non-toxic to children

91. Anti-arrhythmic agent that can be given intravenously in the treatment of malaria:
A. phenytoin B. procainanide C. quinidine*** D. chloroquine

92. What is the mechanism of action of cyclosporine as an immunosuppressive agent used in human organ transplantation?
A. interferes with DNA synthesis and function
B. inhibits T cell generation in response to transplant antigens***
C. intensifies the production of T lymphocytes
D. inhibits B cell imunoglobulin production

93 Patients with photophobia , excessive lacrimation, stomatitis, cheilosis and keratitis are usually deficient in:
A. ascorbic acid B. cyanocobalamine C. riboflavin*** D. thiamine

94. An anti-thyroid agent that is used days before surgery in order to diminish the size and vascularity the hyperplastic thyroid gland:
A. Methimazole C. propylthiouracil
B. iodides*** D. liothyronine

95.The drug of choice for the treatment of chronic mucocutaneous candidiasis is:
A. amphotericin B C. metronidazole
B. Ketoconazole D. nystatin***

96. The most commonly used antibiotic in topical preparation with a very low incidence of hypersensitivity reaction:
A. clotrimazole C. griseofulvin
B. bacitracin*** D. cephalexin

97. Which of the following drugs is LEAST likely to be effective in the treatment of an infection caused by S. aureus?
A. amoxycillin *** C. cefazolin
B. nafocillin D. oxacillin

98. The physiologic functions of thyroid hormones are characterized by the following, EXCEPT:
A. insulin like effects
B. increased oxygen consumption***
C. hypolipoproteinemic effects
D. enhanced basal metabolic rate

99. Malignant hyperthermia is used to be one or the causes of death in anesthesia. This is associated with widespread rigidity and enhanced
heat production by muscle. Which of the following drugs is used to counter act this effect:
A. neostigmine B. atropine C. dantrolene*** D. succinylcholine

100. What do we call drugs that mimic the effects of sympathetic nervous system stimulation?
A. antiadrenergics C. adrenergics***
B. cholinergics D. anticholinergics

1. A patient came in because of severe headache. Before giving a drug, the physician must know what the drug will do to the body. This is
known as:
A. Pharmacodynamic C. Drug development
B. Pharmacokinetic D. Adverse drug reaction

2. Distribution of drugs to tissue:


Pharmacology
22 / 68
A. Is independent on blood flow to the organ
B. Is independent of the solubility of the drug to the tissue
C. Depend on the unbound drug concentration gradient between blood and the tissue
D. Is increased for drugs that are strongly bound to plasma proteins

3. Giving a second drug in order to enhance the action of a primary drug is a kind of drug interaction known as:
A. potentiation C. additive
B. synergism D. antagonism

4. Two drugs have said to have this property when they reached the same plasma concentration at the same rate after oral administration:
A. Potency C. Bioavailability
B. Efficacy D. Bioequivalence

5. When the rate of elimination is proportionate to the concentration it is:


A. Zero order kinetic C. Maechelis Menten kinetics
B. First order kinetics D. none of the above

6. Entero-hepatic recycling of drugs is:


A. Volume of distribution C. First pass effect
B. Half-life D. First order kinetics

7. Which of the following reactions is most effective for Phase II elimination of hydrophobic drugs?
A. glucoronidation C. oxidation
B. hydrolysis D. reduction

8. Drug oxidation frequently involve all of the following EXCEPT:


A. Cytochrome p450 enzymes C. Esterases
B. NADH or NADPH cofactors D. Molecular oxygen

9. Jovita after recurrent seizure became comatose. Blood gas analysis showed severe metabolic acidosis. The antidote for Jovita is:
A. Endophonium C. Pyridoxine
B. Atropine sulfate D. Thiamine

10. A 14 year old was rushed to the E.R. because of poisoning. P.E. showed increased sweating and salivation with constricted pupils. The most
likely substance ingested is:
A. Salicylates C.Opioids
B. Anticholinesterase drugs D.Antidepressants

11. Which of the following will produce the most rapid induction of anesthesia?
A. Halothane C. Methoxyflurane
B. Nitrous oxide D. Isoflurane

12. Of the following drugs, this tranquilizer has muscle relaxant and anticonvulsant properties:
A. Diazepam C. Ethanol
B. Sodium Luminal D. Meprobamate

13. An anticonvulsant which is metabolized to Phenobarbital, hence, their clinical effects are very similar:
A. Carbamazepine C. Diazepam
B. Primidone D. Oxazepam

14. The first synthetic local anesthetics widely used for infiltration anesthesia:
A. Procaine C. Etidocaine
B. Bupivacaine D. Benzocaine

15. Repeated exposure to which of the following anesthetic agents is most associated with liver damage:
A. Isoflurane C. Nitrous oxide
B. Halothane D. Enflurane

16. Which of the following inhalation anesthetics is most likely to produce diffusional hypoxia?
A. Cyclopropane C. Nitrous oxide
B. Halothane D. Methoxyflurane

17. Secobarbital and ethanol when given together show:


A. Additive sedative action
B. Reduced sedative action
C. Competitive antagonism
D. Chemical antagonism

18. Nitrous oxide has a:


A. potent action on the medullary center
B. significant effect on renal function
C. reliable surgical anesthesia only under hyperbaric condition
D. potent relaxation effect

19. Of the Phenothiazines, which is the most popularly used as an antipsychotic drug?
Pharmacology
23 / 68
A. Chlorpromazine C. Chlordiazepoxide
B. Meprobamate D. Diphenhydramine

20. Extrapyramidal adverse effect is commonly seen with the use of this anti-psychotic agent:
A. Haloperidol C. Chlorpromazine
B. Thioridazine D. Fluoxetine

21. An agent that is used in the treatment of organo-phosphate poisoning:


A. Atropine C. Physostigmine
B. Neostigmine D. Naloxone

22. The major action involve in the antiseizure activity of Phenytoin is:
A. Block of sodium ion channel
B. Block of calcium ion channel
C. Facilitation of GABA action on chloride ion
D. Inhibition of GABA transaminase

23. A 3 years old was brought to the ER because of recurrent attacks of blank stares lasting for few minutes. This child is best manage with this
anti-seizure agent:
A. Phenobarbital C. Lamotrigine
B. Ethosuximide D. Vigabantrine

24. Prolonged treatment with this anti-seizure agent is associated with gingival hyperplasia :
A. Ethosuximide C. Valproate
B. Phenobarbital D. Phenytoin

25. This is an inhibitory neurotransmitter:


A. Acetylcholine C. Glycine
B. Glutamic acid D. GABA

26. The best treatment for Parkinson’s disease is:


A. Levodopa C. Amantadine
B. Selegiline D. Pergolide

27. This drug competitively inhibits the binding of aldosterone to mineralocorticoid receptors:
A. amiloride C. spirinolactone
B. triamterene D. hydrochlorothiazide

28. This drug potentiates the ototoxic effect of loop diuretics:


A. coumadin C. digitalis
B. lithium D. streptomycin

29. A diuretic agent that inhibit NaCl symport:


A. spirinolactone C. triamterene
B. ethacrynic acid D. hydrochlorothiazide

30. A symphatomimetic agent that also acts as beta adrenergic agonist:


A. dopamine C. isoproterenol
B. dobutamine D. norepinephrine

31. This drug acts by inhibiting renal systemic and tissue generation of angiotensin II:
A. losartan C. amrinone
B. captopril D. hydralazine

32. Dopamine is used in the treatment of advanced left heart failure because it:
A. causes vasoconstriction
B. produces positive inotropic effect
C. reduces left ventricular afterload
D. decreases cardiac output

33. The following are effects of cardiac glycosides except:


A. inhibits NaKATPase
B. decreased calcium entry to myocardium
C. reduced reuptake of norepinephrine from nerve terminals
D. direct inotropic effects on cardiac muscle

34. Of the vasodilators listed below, the drug of choice for the treatment of acute anginal attack is:
A. Nitroglycerin C. Nifedipine
B. Hydralazine D. Propranolol

35. This angiotensin converting enzyme inhibitor (ACEI) is a prodrug and must be hydrolyzed by the esterases in the liver:
A. captoril C. lisinopril
B. enalapril D. trondolapril

36. A contraindication to the use of beta blockers


Pharmacology
24 / 68
A. Bronchial asthma C. Thyrotoxicosis
B. Hypertension D. Congestive heart failure

37. An acute attack of re-entrant supraventricular tachycardia will respond to:


A. Digitalis C. Magnesium sulfate
B. Verapamil D. Quinidine

38. A 60 years old female patient complained of nystagmus after taking this antiarrythmic drug. The drug could be:
A. Disopyramide C. Procainamide
B. Lidocaine D. Quinidine

39. The mechanism of action of gembfibrozil is:


A. Increase excretion of bile acid salts
B. Increase expression of high affinity LDL- receptor
C. Increase hydrolysis by lipoprotein lipase
D. Inhibition of secretion of VLDL by the liver

40. This antihyperlipidemic agents reduces circulating cholesterol levels by binding to cholesterol in the GIT?
A. niacin C. clofibrate
B. cholestyramine D. gemfibrozil

41. Which of the following drugs is used as an emergency agent for hypertensive crisis?
A. Methyldopa C. Hydralazine
B. Diltiazem D. Nitroprusside

42. A Second generation Sulfonylurea that can inhibit peripheral deiodination of thyroid hormone
A. Tolbutamide C. Glyburide
B. Chlorpropamide D. Glimepiride

43. A member of the newer oral hypoglycemic agents classified as insulin sensitizers
A. Rosiglitazone C. Glimepiride
B. Voglibose D. Repaglinide

44. A thionamide that can block peripheral deiodination of thyroid hormone


A. Propranolol C. Prophylthiouracil
B. Carbimazole D. Methimazole

45. Which among these antithombotic drugs inhibit the synthesis of thomboxane by irreversible acetylation of the enzyme cyclooxygenase?
A. Abciximab C. Integrilin
B. Aspirin D. Ticlodipine

46. An isoxazole derivative of ethinyltestosterone that is used to suppress ovarian function:


A. Danazol C. Raloxifene
B. Clomiphene D. Mifepristone

47. A gnRH analog which produces gonadal suppression:


A. Flutamide C. Finasteride
B. Leuprolide D. Spironolactone

48. A principal regulator of bone mineral homeostasis which needs to be metabolized to gain biologic activity:
A. Vitamin D C. Glucocorticoids
B. Parathyroid hormone D. Adrenal androgens

49. Which among the following is a long-acting glucocorticoids?


A. Betamethasone C. Prednisone
B. Cortisone D. Triamcinolone

50. Antineoplastic agent usually cause the following side effects EXCEPT:
A. loss of hair C. nausea and vomiting
B. liver damage D. bone marrow suppression

51. Busulfan, a useful drug in chronic myelocytic leukemia is:


A. An antimetabolite C. an alkylating agent
B. An antibiotic D. a hormonal agent

52. An immunopotentiating agent with an antihelminthic property:


A. Herceptin C. Interferon
B. Rituximab D. Levamisole

53. The most effective drug to use during acute attacks of gouty arthritis is:

Pharmacology
25 / 68
A. Allopurinol C. Colchicine
B. Probenecid D. Phenylbutazone

54. One approach in the treatment of bronchial asthma is the use of mediator release inhibitors which would include:
A. Cromolyn sodium C. Terbutaline
B. Theophylline D. Ketotifen

55. Which anti-asthmatic would be your choice if you want one with a less cardiac activity?
A. Epinephrine C. Terbutaline
B. Ephedrine D. Isoproterenol

56. A long acting Beta-2 selective agonist used for asthma prophylaxis:
A. Cromolyn C. Metaproterenol
B. Ipatropium D. Salmeterol

57. The H2 antagonist with the highest bioavailability:


A. Cimetidine C. Ranitidine
B. Famotidine D. Nizatidine

58. An irreversible proton pump inhibitor:


A. Lanzoprazole C. Omeprazole
B. Pantoprazole D. Famotidine

59. This drug is used in the treatment of PUD as a mucosal protective agent
A. Sucralfate C. magnesium OH
B. metronidazole D. ocreotide

60. Ethambutol is administered concurrently with other antituberculous drugs in the treatment of TB inorder to:
A. Reduce the pain of injection
B. Facilitate penetration of the BBB
C. Retard the development of organism resistance
D. Delay excretion of other anti TB drugs by the kidney

61. The mainstay drug in the treatment of all types of leprosy is:
A. Clofazimine C. Isoniazid
B. Dapsone D. Rifampicin

62. Vestibular and auditory dysfunction can follow the administration of:
A. Tetracycline C. Clindamycin
B. Amikacin D. Carbenicillin

63. A beta-lactamase inhibitor:


A. Cefotaxime C. Sulbactam
B. Amoxicillin D. Ticarcillin

64. A drug of choice in Rickettsial infections:


A. Chloramphenicol C. Sulfonamide
B. Tetracycline D. Polymyxin B

65. An antimicrobial agent which inhibits bacterial cell wall synthesis:


A. Neomycin C. Erythromycin
B. Cephalexin D. Trimethoprim

66. Gray baby syndrome, a form of cardio-respiratory collapse, is an age specific toxicity of which antimicrobial in the newborn?
A. Sulfonamides C. Chloramphenicol
B. Nalidixic acid D. Nitrofurantoin

67. Exerts a synergistic effect with Sulfonamide:


A. Trimethoprim C. Atropine
B. Prostaglandin D. Metronidazole

68. An anti-malarial agent which also exerts an amebicidal effect:


A. Chloroquine C. Primaquine
B. Chloroguanide D. Mefloquine

69. Antimicrobial agent with greatest CSF penetration:


A. Chloramphenicol C. Tetracycline
B. Penicillin D. Erythromycin

70. The most absorbable of all the oral erythromycin salt is:
A. Lactobromate C. Estolate
B. Succinate D. Stearate

71. This drug, with less side effects, is effective against ascaris and hookworms
A. Mebendazole C. Pyrantel pamoate
Pharmacology
26 / 68
B. Piperazine D. Niclosamide

72. An effective amebicidal agent and at the same time displays antibacterial activity against anaerobes:
A. Metronidazole C. Paromomycin
B. Quinacrine D. Chloroquine

73. The drug of choice for monilial infections of skin and mucous membrane:
A. Griseofulvin C. Amphotericin B
B. Nystatin D. Selenium sulfide

74. Antiviral drug which is effective on the treatment of herpes simplex encephalitis is:
A. Idoxuridine C. Acyclovir
B. Vidarabine D. Amantadine

75. Primaquine should be used with caution in G6PD deficient individuals because of the possibility of production of:
A. Aplastic anemia C. Granulocytopenia
B. Hemolytic anemia D. Megaloblastic anemia

76. Imipenem is combined with this drug:


A. Clindamycin C. Cephalosporin
B. Cilastatin D. Carbenicillin

77. Which of the chemical structure of Beta-lactams is the site of action of Penicillinase?
A. Beta-lactam ring C. R-side chain
B. Thiazolidine ring D. Dihydrothiazine ring

78. The Penicillin with the longest half life (24-28 days ) is:
A. Phenoxymethylpenicillin C. Benzathine penicillin
B. Procaine penicillin D. Benzyl penicillin

79. Antibiotic that is NOT recommended for use in children or during pregnancy because of its potential to cause arthropathy:
A. Sulfonamides C. Aminoglycosides
B. Macrolides D. Quinolones

80. Most nephrotoxic of the aminoglycosides:


A. Streptomycin C. Gentamicin
B. Neomycin D. Amikacin

81. The ratio of sulfamethoxazole to Trimethoprim for it to be effective is:


A. 40:1 C. 200:1
B. 20:1 D. 10:1

82. Doxycycline is an appropriate therapy in infection caused by:


A. Rickettsiae C. Schistosomiasis
B. Plasmodium vivax D. Histoplasma capsulatum

83. The drug recommended for most heavy metal intoxications is:
A. CaNa2EDTA C. Dimercaprol
B. Penicillamine D. Deferoxamine

84. A 24 year old man working in a company that manufacture “semiconductors”.


Following accident at the plant he manifested with nausea and vomiting, headache, hypotension and chills. Laboratory showed
hemoglobinuria and a plasma hemoglobin of 1,4g/dl. This individual is probably exposed to:
A. Arsine C. Mercury vapor
B. Inorganic arsenic D. Methylmercury

85. Serotonin antagonist not derived from a fungus that is useful for the treatment of migraine headache is:
A.Sumapitran C. Bromocriptine
B.Ergotamine D. Ketanserin

86. Clarithromycin and erythromycin have very similar spectrum of antimicrobial activity. The major advantage of Clarithromycin is that:
A. Eradicate mycoplasmal infection in a single dose
B. Active against strains of streptococci that are resistant to erythromycin
C. More active against Mycobacterium avium complex
D. Acts on methicillin resistant staphylococci

87. The primary mechanism underlying the resistance of gram positive organism to macrolide antibiotics is:
A. Methylation of binding site on the 50S ribosomal subunits
B. Formation of esterases that hydrolyzed the lactone ring
C. Formation of drug- inactivating acetyltransferase
D. Decrease drug permeability of the cytoplasmic membrane

88. Weeping lesion is best treated with:


A, Wet dressing C. Ointment
B. Gels D. Aerosol

Pharmacology
27 / 68

89. Host factors affecting the use of antimicrobial agents is/are:


A. Age of patient C. Genetic factor
B. Site of infection D. All of the above

90. The mechanism of action of miconazole is best described as:


A. Binding to 80 S ribosomes
B. Inhibition of sterol synthesis
C. Blockade of tetrahydrofolate reductase
D. DNA intercalation

91. Leukoverin rescue is used to reverse toxicity of which of the following anti-cancer agents?
A. Vinblastine C. Adriamycin
B. Cyclophosphamide D. Methotrexate

92. Antiarrhythmic drug that predominantly block the slow calcium channels :
A. Verapamil C. Lidocaine
B. Amiodarone D. Metoprolol

93. Antiarrythmic drug that predominantly block potassium channels and prolongs repolarization:
A. Class I C. Class II
B. Class III D. Class IV

94. Recommended drug therapy for angina in patients with hyperthyroidism:


A. Dihydroperidine Calcium Channel blockers
B. Beta-Adrinergic Receptor Blocker
C. Non-Dihydroperidine Calcium Channel blockers
D. Organic Nitrates

95. Anti anginal drug which should be avoided in patients with Hypertrophic cardiomyopathy:
A. Metoprolol C. Isosorbide-5-mono nitrate
B. Diltiazem D. Verapamil

96. The least antigenic source of Insulin is


A. Beef C. Pork
B. Human D. Horse

97. Preparation of choice for replacement and suppression therapy of the thyroid hormone
A. levothyroxine C. liothyronine
B. triiodothyronine D. tyrosine

98. Anemia with neurologic abnormalities occur most often in deficiencies of:
A. vitamin B12 C. iron
h. B. folic acid D. biotin

99. Which of the following has the highest elemental iron?


A. Ferrous gluconate C. Ferrous fumarate
B. Hydrated ferrous sulfate D. Desiccated ferrous sulfate

100. A direct acting cholinomimetic that is lipid soluble and often used in the treatment of glaucoma?
A. Acetycholine C. Pilocarpine
B. Bethanecol D. Physostigmine

1. According to the Philippine National Drug Formulary (PNDF), complementary


drugs are:
A. those drugs underlined in the guide
B. drugs that satisfy the health care needs of majority of population
C. drugs with special pharmacological properties
D. not to be used as an alternative to core drugs

2. Which of the following is considered a regulated drug?


A. pseudoephedrine C. lysergic acid diethylamide (LSD)
B. meperidine D. cocaine

3. Which of the following sequences of dosage forms is arranged correctly from lowest
to highest rate of disintegration and dissolution after oral administration:
A. capsule – compressed tablet – suspension
B. compressed tablet – enteric-coated tablet – capsule
C. suspension – capsule – compressed tablet
D. enteric-coated tablet – compressed tablet – capsule

4. Which among the following alkaloid drugs is used as a potent analgesic?


A. ergotamine C. theophylline
B. morphine D. vincristine
Pharmacology
28 / 68

5. Differences in the rate of acetylation of isoniazid among groups of individuals exemplifies.


A. pharmacodynamics C. pharmacogenetics
B. posology D. biochemorphology

6. Absorption of most orally administered drugs occurs in the small intestine. This may be enhanced by:
A. taking agents that accelerate gastric emptying time
B. the presence of food
C. stimulation of hepatic microsomal CYP3A4 enzymes
D. increasing intestinal motility

7. Which of these statements characterize a Phase 2 clinical trial?


A. it constitutes monitoring safety of the new drug under actual conditions after marketing begins
B. it studies the drug for the first time in small number of patients with the disease to determine its efficacy
C. it evaluates the drug in much larger number of patients to further establish safety and efficacy
D. it establishes probable limits of the safe clinical dosage range in healthy human volunteers

8. A 10-year-old girl is on theophylline, an anti-asthma drug with a narrow therapeutic window and which is metabolized by CYP450. High amounts
of the following in her diet may result in higher blood levels of theophylline:
A. char-grilled meat C. grapefruit juice
B. cabbage D. A and B

9. The pharmacokinetic parameter which determines the speed of drug input that must balance the speed of drug elimination to achieve a steady
state concentration is:
A. clearance C. dosing rate
B. bioavailability D. volume of distribution

10. The following are idiosyncratic genetic abnormalities of affected individuals to drugs EXCEPT:
A. glucose 6 phosphate dehydrogenase deficiency causing hemolytic anemia
B. low plasma cholinesterase activity causing local anesthetic toxicity
C. abnormal calcium release from the sarcoplasmic reticulum causing malignant hypertension
D. spontaneous bleeding due to heparin
11. A 10 mg parenteral dose of morphine has the same efficacy as a 100 mg parenteral dose of pethidine. This means that:
A. morphine is less potent than pethidine.
B. both drugs produce sedation and respiratory depression to almost similar degree.
C. morphine and pethidine are bioequivalents
D. only A and C are correct

12. The nicotinic acetylcholine receptors are categorized as to which type?


A. G protein coupled C. kinase-linked
B. ion channel-linked D. nuclear

13. The incorporation of sulfonamides into the cellular components leads to:
A. counterfeit product formation
B. chelation
C. ion channel blockade thru saturation of cell membrane
D. competitive antagonism

14. The most important pharmacokinetic parameter to be considered in optimizing


dosing of patients to maintain a steady state drug concentration is:
A. distribution volume C. bioavailability
B. clearance D. half-life

15. Pharmacokinetic type of tolerance may be due to which of the following?


A. increased synthesis of hepatic microsomal enzymes
B. substrate depletion
C. decreased induction of P glycoproteins (PgP)
D. neuronal adaptation

16. The following statements refer to the loading dose of the drug EXCEPT:
A. It is aimed to achieve target concentration rapidly
B. It is for drugs with long half-lives
C. It may be used to saturate tissue sites
D. It will take 4 half lives to reach steady state concentration

17. If a drug has a high distribution volume, it indicates:


A. it is highly bound to plasma proteins
B. the drug is highly retained in the vascular compartment
C. the plasma drug concentration will increase
D. there is extensive tissue uptake

18. The index of safety of a drug decreases as the median toxic dose :
A. decreases and the median effective dose increases
B. increases and the median effective dose decreases

Pharmacology
29 / 68
C. and the median effective dose decrease
D. and the median effective dose increase

19. The administration of penicillamine in the presence of high level of copper iron will lead to this type of antagonism:
A. competitive C. physiologic
B. chemical D. dispositional

20. L.E., a 20 year old patient from Panay Island, developed hemolytic anemia upon exposure to primaquine. This atypical response is attributed to:
A. deficiency of glucose-6-phosphate dehydrogenase
B. low level of acetyl transferase
C. less effective aldehyde dehydrogenase
D. defective plasma pseudocholinesterase

21. The abrupt cessation of drug therapy, such as beta blockers, should be avoided to prevent:
A. rebound phenomenon C. tachyphylaxis
B. tolerance D. hyperreactivity

22. The occurrence of tolerance to a drug can be explained by the following mechanism:
A. increased availability or activation of the drug receptor
B. up-regulation of receptors
C. down-regulation of receptors
D. A and B only

23. The Latin abbreviation prn means:


A. at bedtime C. as needed
B. per rectum D. by mouth

24. What is the major second messenger of beta receptor activation that participates in signal transduction?
A. inositol triphosphates C. calcium
B. cAMP D. adenylyl cyclase

25. What pharmacologic effect will result if a cholinomimetic agonist is administered in recommended doses?
A. reflex bradycardia C. urinary frequency
B. cycloplegia D. hyperglycemia

26. What is the mechanism of action of saxitoxin in blocking axon potential propagation?
A. depletion of Ach at nerve terminals
B. inhibition of Ach release at nerve terminals
C. blockade of calcium channels
D. blockade of sodium channels

27. Which of these pharmacologic actions of epinephrine is mediated by alpha receptor activation?
A. nasal congestion C. pupillary dilatation
B. bronchodilatation D. relaxation of vascular smooth muscles

28. In what condition are the amphetamine-related drugs therapeutically useful?


A. attention-deficit hyperkinetic disorder
B. diabetic neuropathy
C. chronic obstructive pulmonary disease
D. essential hypertension

29. In what condition are the drugs phenoxybenzamine and phentolamine useful?
A. pheochromocytoma C. essential hypertension
B. migraine attacks D. benign prostatic hyperplasia

30. Which of the following is the mechanism of action of pralidoxime, an appropriate antidote for organophosphate poisoning?
A. competitive binding at acetylcholine receptor sites
B. competitive inhibition at acetylcholine receptor sites
C. acetylcholinesterase inhibition
D. acetylcholinesterase regeneration

31. What is the mechanism for atropine’s reversal of muscarinic toxicity?


A. regeneration of acetylcholinesterase inhibition
B. activation of acetylcholinesterase
C. competitive binding of acetylcholine receptor sites
D. all of the above

32. Acetylcholine acts by:


A. increasing adenylyl cyclase activity
B. decreasing cGMP concentrations
C. activating the IP3, diacylglycerol pathway
D. all of the above

33. Atropine is contraindicated in patients with narrow-angle glaucoma because:


A. it reduces lacrimal secretion
B. of its effect on the ciliary epithelium
Pharmacology
30 / 68
C. of its effect on the ciliary muscle
D. it increases aqueous humor production

34. Which of the following antihypertensive agents has the best documented evidence for prophylaxis against migraine attacks?
A. pindolol C. clonidine
B. propranolol D. prazosin

35. Which of the following beta blockers exhibits membrane stabilizing action?
A. timolol C. propranolol
B. atenolol D. nadolol

36. The effect of tamsulosin on prostatic smooth muscles reflects its higher affinity for which of the following receptors?
A. α 1A C. α 1C
B. α 1B D. A, B, C only
.
37. Bradycardia is a side effect more likely to be expected which vasodilator?
A. verapamil C. nifedipine
B. prazosin D. hydralazine

38. The following antianginal drugs are vasodilators, EXCEPT:


A. felodipine C. metoprolol
B. nitroglycerin D. diltiazem

39. Which is a direct effect of nitrates when given in the usual doses for the management of angina?
A. increased preload C. increased cardiac contractility
B. increased afterload D. smooth muscle relaxation

40. Captopril and enalapril do all of the following, EXCEPT:


A. inhibit an enzyme, peptidyl dipeptidase
B. competitively block angiotensin II at its receptor
C. decrease angiotensin II concentration in the blood
D. increase bradykinin concentration in the blood

41. A college friend consults you regarding the suitability of the therapy his doctor has prescribed for severe hypertension. He complains of postural
and exercise hypotension ("dizziness"), some diarrhea, and problems with ejaculation during sex. Which of the following is most likely to produce
the effects that your friend has described?
A. propranolol C. hydralazine
B. guanethidine D. captopril

42. The most important pharmacologic action of digitalis in the treatment of heart failure is its ability to:
A. reduce venous pressure C. produce diuresis in edematous patients
B. increase heart rate D. increase myocardial contractile force

43. Which of the following drugs have been shown to retard deterioration of left ventricular function and prolong survival of patients with heart failure?
A. enalapril C. dobutamine
B. furosemide D. milrinone

44. Which of the following bronchodilators is a xanthine derivative?


A. terbutaline C. theophylline
B. ipratropium bromide D. salmeterol

45. Which of the following is an inhaled corticosteroid for use in asthma?


A. prednisone C. cromolyn sodium
B. hydrocortisone D. beclomethasone

46. A leukotriene antagonist used in the prevention of asthma:


A. ketotifen C. cromolyn sodium
B. budesonide D. montelukast

47. A 22 year old asthmatic patient who experiences symptoms less than once a week which last a few hours to a few days and has nighttime
symptoms twice a month is best managed with an:
A. inhaled long-acting β 2 agonist at h.s.
B. inhaled short-acting β 2 agonist p.r.n.
C. inhaled steroids o.d.
D. oral β 2 agonist t.i.d.

48. The most effective antitussive is:


A. codeine C. guaifenesin
B. bromhexine D. S-carboxymethylcysteine

49. To induce ovulation after stimulating follicular growth in anovulatory women, which of the following is preferred?
A. chorionic gonadotropin C. hMG
B. FSH D. GnRH

Pharmacology
31 / 68
50. Side effects of oxytocin use in a woman in labor includes the following, EXCEPT:
A. increase in BP C. fetal death
B. uterine rupture D. diuresis

51. To minimize hypothalamic and pituitary suppression, glucocorticoids are usually given:
A. before meals C. in the morning
B. after meals D. in the evening

52. Which of the following insulin preparations is the only one which can be administered intravenously?
A. lente insulin C. regular insulin
B. ultralente insulin D. NPH insulin

53. Rapid relief from the symptoms of hyperthyroidism is best accomplished with:
A. beta blockers C. methimazole
B. propylthiouracil (PTU) D. radioactive iodine (RAI)

54. An antithyroid drug which may be given to reduce the vascularity of the thyroid gland prior to thyroidectomy:
A. RAI C. methimazole
B. PTU D. Lugol's iodine

55. Drugs that decrease bone resorption include the following, EXCEPT:
A. calcitonin C. alendronate
B. estrogen D. fluoride

56. Myelosuppression is a common side effect associated with the use of the following anticancer drugs EXCEPT:
A. vincristine C. methotrexate
B. cyclophosphamide D. chlorambucil

57. The following anticancer drug, with its toxic effect, is correctly matched with the appropriate antidote:
A. cyclophosphamide-induced hemorrhagic cystitis -- ondansetron
B. methotrexate-induced myelosuppression -- MESNA
C. cisplatin-induced nausea and vomiting -- leucovorin
D. doxorubicin-induced arrhythmia -- dexrazoxane

58. Which of the following anticancer drugs acts as a mitotic inhibitor?


A. methotrexate C. bleomycin
B. vinblastine D. carmustine

59. Which drug is used as an antiretroviral agent?


A. oseltamivir C. indinavir
B. famciclovir D. ribavirin

60. Which of the following statements is related to the pharmacokinetics of isoniazid?


A. It is readily absorbed from the GIT.
B. It is given with niacin to prevent neuropathy.
C. There is poor diffusion of isoniazid into body tissues.
D. Excretion of isoniazid is primarily through the bile.

61. Which of the following refer to amikacin?


A. It is effective for Bacteroides, Peptostreptococcus, and Clostridium spp..
B. It exhibits cross-resistance with gentamicin and tobramycin.
C. Once daily dosing of amikacin is ineffective.
D. It is more toxic to the cochlear division of cranial nerve VIII.

62. Which of the following enzymes is inhibited by the binding of a beta lactam antimicrobial with penicillin-binding proteins (PBPs)?
A. beta-lactamase C. peptidyl tranferase
B. transpeptidase D. acetylase

63. A poor farm helper from Palawan was diagnosed to have falciparum malaria by
blood smear. The safe, cost-effective, and readily available agent to prescribe if this
farmer has an allergy to sulfonamides is:
A. Fansidar C. chloroquine
B. quinine D. mefloquine

64. Radical cure for malaria is achieved by giving:


A. chloroquine C. mefloquine
B. quinine D. primaquine

65. Which step of viral replication is inhibited by Zidovudine.?


A. adsorption to and penetration into susceptible host cells
B. uncoating of viral nucleic acid
C. synthesis of early regulatory proteins
D. synthesis of nucleic acid (DNA and RNA)

66. Which of the following anti-Koch’s preparations has the greatest activity as an inducer of hepatic enzymes?
A. isoniazid C. pyrazinamide
Pharmacology
32 / 68
B. rifampicin D. streptomycin

67. There is a sudden increase of atypical pneumonia secondary to Mycoplasma, Chlamydia, and Rickettsia in the Pneumonia Ward of a tertiary
hospital. Majority of were treated with doxycycline. All of the following ADRs are to be watched out for in these patients EXCEPT:
A. photosensitivity C. superinfection
B. fetal bone defects D. renal failure

68. The pharmacology of polyene antimicrobials is best described by the following satement:
A. Amphothericin and nystatin are both polyene macrolides.
B. There is good absorption from the GIT of the clinically available preparations.
C. They are generally fungistatic agents.
D. They act by reducing ergosterol synthesis by inhibition of fungal cytochrome P450 enzymes.

69. Treatment with the combination of vancomycin and an aminoglycoside necessitates a close observation for which of the following:
A. low pitched tinnitus
B. moderately intense headache followed by an acute stage of nausea, vomiting, and difficulty with equilibrium
C. decreased urine output
D. neuromuscular blockade and apnea

70. Which of the following agents may predispose to the development of pseudomembranous colitis?
A. erythromycin C. vancomycin
B. metronidazole D. clidamycin

71. A patient with persistent chronic cough and fever had no response to adequate doses of oral penicillin. The PE showed normal chest findings.
Further PE showed bullous myringitis. A chest Xray showed infiltrates in the right lower lobe. Which of the following antimicrobials would be effective?
A. parenteral penicillin G C. vancomycin
B. chlarithromycin D. gentamicin

72. A 24 year old woman admitted to the emergency room complains of severe, acute, generalized headache. Her temperature is 40 C and exhibits
nuchal rigidity. Her CSF revealed gram-Negative, diplococci and 200 leucocytes/mm3 (98% of which are polymorphonuclear neutrophils). The most
important management would be the administration of:
A. IV sulfonamides C. IV penicillin
B. intrathecal penicillin D. intrathecal chloramphenicol

73. A 4 year old boy that has diarrhea and malnutrition was noted to have a mixed parasitic infection of Ascaris, Capillaria philippinensis, and Trichiuris
trichiura. The single agent that could be effective for this patient is:
A. pyrantel pamoate C. diethylcarbamazine
B. mebendazole D. niclosamide

74 Lovastatin exhibits all of the following effects on lipid profile EXCEPT:


A. decreased serum cholesterol
B. decreased LDL
C. increased triglycerides
D. increased HDL

75. Fibrinolytics affect what specific biological reaction?


A. inactivation of cyclooxygenase
B. inactivation of thrombin
C. formation of gamma-carboxyglutamyl side chain
D. conversion of plasminogen to plasmin

76. The laboratory test/s for heparin effect is/are:


A. plasma thromboplastin time C. WBC count
B. protime D. phytonadione

77. The following are mucosal protective agents EXCEPT:


A. sucralfate C. misoprostol
B. bismuth subsalicylate D. pirenzepine

78. Which of the following drug group for PUD has a better inhibition of nocturnal gastric secretion compared to their 24-hour gastric acid inhibition?
A. antacids C. proton pump inhibitors
B. H2 receptor blockers D. mucosal protective agents

79. Multivitamin-mineral doses for nutritional support in pregnancy must contain higher doses of:
A. folate C. thiamin
B. iron D. A and B only

80. From among the following, ingestion of gasoline or kerosene may cause the most severe reaction to which of the following organs:
A. esophagus C. lungs
B. stomach D. skin

81. In a patient who is not comatose, emesis is indicated after acute poisoning following oral ingestion of all of the following EXCEPT:
A. strong alkali or acid C. digitalis
B. antihistamine D. morphine

82. Activated charcoal may be used to treat poisoning due to the following EXCEPT:
Pharmacology
33 / 68
A. primaquine C. propoxyphene
B. aspirin D. iron overdose

83. This herbal plant endorsed by the DOH is useful for the relief of muscle pain:
A. Yerba Buena C. Tsaang gubat
B. Akapulko D. Sambong

Akapulko is used as an anti-fungal. Tsaang gubal is used for stomach pain. Sambong is a diuretic and an anti-urolithiasis agent.

84. The necessary enzyme for folic acid synthesis that is inhibited by diaminopyrimidines is:
A. dihydrofolate reductase C. mycolase synthetase
B. dihydropteroate synthetase D. reverse transcriptase

85. Which of the following refers to erythromycin base?


A. it is an azalide antimicrobial
B. it inhibits protein synthesis at the 23s ribosomal subunit
C. it remains stable in an acidic environment
D. it is excreted in the bile and feces

86. The poor-healing wound in the hand of a non-diabetic mechanical engineer did not respond to oral penicillin V. The single rational beta-lactam
antimicrobial that could be given for this patient on the basis of efficacy, affordability, and availability from among the following is:
A. benzathine penicillin C. ticarcillin
B. methicillin D. cloxacillin

87. From among the following, the rational antimicrobial for a 7 month old Filipino with community-acquired pneumonia (if a laboratory is not available)
is:
A. cephalexin C. chloramphenicol
B. ciprofloxacin D. vancomycin

88. If the agents below are available in the nearby drug store, the efficacious and affordable agent for a school teacher with Tinea (pytiriasis) versicolor
on large areas on her chest and her back:
A. selenium sulfide C. amphothericin b
B. fluoconazole D. terbinafine

89. Which of the following statements is TRUE about diloxanide furoate?


A. it is a effective for extraintestinal amoebiasis
B. it is the agent of choice for hepatic abcess
C. it is effective for asymptomatic cyst passers
D. it is safe for infants and pregnant patients

90. On regular follow-up for tuberculosis, you noticed that the visual acuity of Vilma from 20/20 to 20/50 in both eyes after only three weeks of HRPE.
Your in management would be to discontinue:
A. all of the 4 drugs and reintroduce one drug at a time
B. ethambutol and refer to an ophthalmologist
C. pyrazinamide and refer to an ophthalmologist
D. ethambutol, reintroduce it after 1 week, and refer to an ophthalmologist

91. Drug toxicity in patients with renal failure may be avoided by:
A. maintaining the normal therapeutic dose
B. peritoneal dialysis
C. interval prolongation
D. hemodialysis

92. Fetal alcohol syndrome is characterized by:


A. intrauterine growth retardation C. microcephaly
B. indistinct philtrum D. all of the above.

93. Which of the following adverse reactions is correctly matched with its offending drug?
A. Gray baby syndrome and aplastic anemia - quinine
B. Red man syndrome and phlebitis - vancomycin
C. Blackwater fever and cinchonism - tetracycline
D. enamel dysplasia and candidiasis - chloramphenicol

94. Phenytoin is administered intravenously for acute management of seizure, but when given at a rapid rated can cause
A. generalized seizure C. cardiac arrhythmia
B. gingival hyperplasia D. gastrointestinal disturbances

95. Which anticonvulsant is safe in pregnancy?


A. hydantoin C. valproic acid
B. trimethadione D. none of the above

96. Which of the following is a common mechanism of action shared by all NSAIDs?
A. inhibition of arachidonate release from the plasma membrane by ihibiting phospholipase
B. Inhibition of histamine release by inhibiting mast cell degranulation
C. Inhibition of the production of prostaglandin and thromboxane from arachidonic acid through inhibition of prostaglandin H
synthesis
Pharmacology
34 / 68
D. Inhibition of the the formation of leukotrienes by inhibiting lipoxygenase

97. The primary reason why aspirin has been displaced by NSAIDs as first choice drug of pediatric rheumatologists is because:
A. poor inflammatory effect and gastric irritation
B. poor inflammatory effect and Reye’s syndrome
C. lack of liquid preparation and high toxicity effect
D. need for frequent administration and association with Reye’s syndrome

98. The intravenous anesthetic producing the so-called "dissociative anesthesia" is:
A. fentanyl C. ketamine
B. thiopental D. propofol

99. Your patient requests you to give her a sedative-hypnotic that not will not only calm her down or put her to sleep but will make her forget the whole
operating room experience as well. To facilitate this request your drug regimen should include:
A. midazolam C. phenobarbital
B. succinylcholine D. etomidate

100. The side effect/s of antidepressants include/s:


A. sexual dysfunction C. hypertension
B. weight loss D. all of the above

1. Potency of a drug is:


A. the concentration of a drug in the plasma to produce a given effect
B. very important in the clinical use of a drug
C. independent of drug usage
D. directly proportional to drug effect
(Answer: A / Ref: page 50 / MPL: 60)

2. Which of the following statements regarding half-life of a drug is not true?


A. half-life of a drug may increase as drug clearance decreases due to disease
B. half-life provides a good indication of the time for a drug to be removed from the body
C. it provides a means to estimate the amount of drug to be administered
D. it is a useful guide to estimate the appropriate dosing interval
(answer: C /reference p. 22-23 / MPL 60)

3. The route of drug administration that will most likely subject a drug to first pass metabolism:
A. sublingual
B. oral
C. topical application to mucous membranes
D. inhalation
(answer: B /reference p. 6-8 / MPL 90)

4. One of the following is NOT a true statement regarding agonist and/or antagonist drugs:
A. A true agonist interacts directly with its receptor to elicit a pharmacologic response.
B. A partial agonist has affinity for its receptors but with low intrinsic activity
C. A competitive antagonist interacts irreversibly with a receptor
D. A competitive antagonist is devoid of intrinsic activity
(answer: C /reference p. 41 / MPL 60)

5. Which of the following statements is not true concerning plasma-protein binding of drugs?
A. acidic drugs bind to plasma albumin; basic drugs bind to alpha1 acid glycoprotein
B. binding is usually reversible
C. drugs with similar pharmacochemical properties can compete for binding sites
D. plasma-protein binding limits renal tubular secretion and biotransformation of drug
(answer: D /reference p. 9 / MPL 60)

6. Which of the following benzodiazepines is conjugated directly and not metabolized Into an active metabolite?
A. alprazolam C. flurazepam
B. oxazepam D. diazepam
(answer: B /reference p. 406 / MPL 60)

7. An opioid drug with mixed actions can produce this effect:


A. provoke withdrawal symptoms in a person addicted to Heroin.
B. replace Naloxone as antidote to neonates from addicted mothers
C. be used as substitute drug to control opioid addiction
Pharmacology
35 / 68
D. not produce sedation, only analgesia
(answer: A /reference p. 580 / MPL 60)

8. To rehabilitate opioid addicts, one must start with:


A. small doses of naloxone by IM injections
B. psychotherapy
C. methadone, a pure mu agonist with longer duration of action to replace the addicting drug
D. immediate discontinuance of the opioid and control withdrawal symptoms with diazepam
(answer: C /reference p. 633 / MPL 90)

9. Morphine and aspirin both can produce this pharmacologic effect:


A. uricosuria C. analgesia
B. respiratory depression D.physical dependence
(answer: C /reference p. 580 / MPL 90)

10. The hormone of choice for thyroid hormone replacement therapy:


A. levothyroxine sodium C. desiccated thyroid
B. liothyronine sodium D. liotrix
(answer: A /reference p. 1577 / MPL 90)

11. Drug of choice in the management of gestational diabetes:


A. Chlorpropamide C. Isophane insulin
B. Propylthiouracil D. Regular insulin
(answer: D /reference p. 1695 / MPL 90)

12. Antianxiety agent which possess clinically significant antiseizure activity:


A. Haloperidol C. Diazepam
B. Chlorpromazine D. Phenytoin
(answer: C /reference p. 410 / MPL 60)

13. Which of the following cytotoxic agent is used for the treatment of severe disabling psoriasis and refractory rheumatoid arthritis?:
A. cyclosphosphamide C. 5- fluorouracil
B. methotrexate D. cytarabine
(answer: B /reference p. 1403 / MPL 60)

14. Drug of choice for the treatment of amebiasis, giardiasis & trichomoniasis:
A. metronidazole C. cotrimoxazole
B. pentamidine D. melarsoprol
(answer: A /reference p. 1107 / MPL 90)

15. The following are antibiotic-anticancer agents except:


A. Actinomycin D C. Doxycycline
B. Doxorubicin D. Mitomycin
(answer: c /reference p. 1425-1431 / MPL 90)

16. Antiprotozoal agent used clinically for both prophylaxis & treatment of Pneumocystis carinii infection:
A. co-trimoxazole C. Metronidazole
B. pentamidine D. Eflornithine
(answer: B /reference p. 1111 / MPL 60)

17. A more common hematological result of chronic lead intoxication:


A. hypochromic microcytic anemia C. pancytopenia
B. megaloblastic anemia D. leukopenia
(answer: A /reference p. 1854 / MPL 90)

18. One of the following agents prevents the conversion of testosterone to dihydrotestosterone and has been used in the treatment of BPH :
A. GnRH agonists C. Finasteride
B. Ketoconazole D. Cyproterone acetate
(answer: C /reference p. 1646 / MPL 60)

19. Pyridoxine deficiency leads to this condition in the anterior segment of the eye:
A. Bitot’s spots C. macular degeneration
B. keratomalacia D. corneal neovascularization
(answer: D /reference p. 1843 / MPL 60)

20. The oxidant found in the highest concentration in polluted atmosphere:


Pharmacology
36 / 68
A. sulfur dioxide C. ozone
B. nitrogen dioxide D. carbon monoxide
(answer: C /reference p. 1879 / MPL 90)

21. One of the following is a broad spectrum antihelminthic drug:


A. piperazine C. mebendazole
B. pyrantel pamoate D. diethylcarbamazine
(answer: C /reference p. 1127 / MPL 90)

22. The following are second line drugs for tuberculosis, except:
A. pyrazinamide C. capreomycin
B. ethionamide D. kanamycin
(answer: A /reference p. 1284 / MPL 90)

23. A first line anti-TB drug that is used for TB prophylaxis:


A. rifampin C. ethambutol
B. streptomycin D. Isoniazid
(answer: D /reference p. 1275 / MPL 90)

24. One of the following is not a mechanism of action of statins:


A. inhibit cholesterogenesis in liver
B. enhance the removal of LDL precursors by decreasing hepatic VLDL production
C. stabilize endothelial cell nitric oxide synthase mRNA
D. enhance the proliferation of smooth muscle cells
(answer: D /reference p. 984 / MPL 60)

25. The major adverse effect of clinical significance associated with the use of statin:
A. bloating & dyspepsia C. myositis flu-like syndrome
B. myopathy D. peripheral neuritis
(answer: B /reference p. 988 / MPL 60)

26. Which of the following drugs used in the treatment of patients with heart failure is a phosphodiesterase inhibitor:
A. Milrinone C. Nitroglycerin
B. Sodium nitroprusside D. Dobutamine
(answer: A /reference p. 927 / MPL 90)

27. Indications for radioactive iodine in the treatment of Grave’s disease include the following, EXCEPT:
A. hyperthyroidism in older patients
B. hyperthyroidism in those with heart disease
C. graves disease persisting after subtotal thyroidectomy
D. as maintenance therapy for those on remission
(answer: D /reference p. 1588 / MPL 90)

28. This drug is recommended for angina patients with insulin-dependent diabetes mellitus:
A. calcium channel antagonists C. verapamil
B. beta adrenergic antagonists D. nitrates
(answer: B /reference p. 862 / MPL 60)

29. Which of the following is a calcium channel antagonist:


A. verapamil C. captopril
B. methyldopa D. losartan
(answer: A /reference p. 891 / MPL 90)

30. This anti-TB drug inhibits the excretion of urate, resulting in hyperuricemia:
A. ethionamide C. ethambutol
B. streptomycin D. pyrazinamide
(answer: D /reference p. 1281 / MPL 90)

31. A primary effect of ACh on the cardiovascular system is:


A. increase in the rate of conduction in SAN/AVN.
B. increase in heart rate
C. increase in the force of cardiac contraction
D. vasodilatation
(answer: D /reference p. 157 / MPL 60)

Pharmacology
37 / 68

32. The standard cholinergic agent for initial treatment of open-angle glaucoma:
A. bethanecol C. pilocarpine
B. methacholine D. muscarine
(answer: C /reference p. 161/ MPL 60)

33. Atropine, as primary antidote to poisoning by organophosphates acts by:


A. competitively blocking muscarinic receptors
B. irreversibly blocking muscarinic receptors
C. directly inactivating cholinesterases.
D. preventing “aging” of phosphorylated AChE.
(answer: A /reference p. 164 GG / MPL 90)

34. Neuromuscular blocking agents are used as adjuvants in surgical anesthesia to:
A. induce hypnosis
B. provide adequate skeletal muscle relaxation.
C. lower the dose of the general anesthetic agent
D. potentiate depth of anesthesia in the surgical stage.
(answer: B /reference p. 205 / MPL 90)

35. Which of the following drugs are used to reactivate phosphorylated AChE:
A. Pyridostigmine C. Methscopolamine
B. Pralidoxime D. Tacrine
(answer: B /reference p. 185 / MPL 90)

36. Measures the potency of general anesthetics :


A. minimum alveolar concentration. C. degree of muscle relaxation
B. partition coefficient of the drug D. emergence from anesthesia
(answer: A /reference p. 328 / MPL 90)

37. The primary therapeutic use of O2 is to:


A. alleviate dyspnea
B. correct hypoxia
C. reduce partial pressure of an inert gas
D. provide hyperbaric bacteriostasis
(answer: B /reference p. 391 GG, 10th ed / MPL 90)

38. The most common untoward effect of tricyclic antidepressants is:


A. postural hypotension C. maniacal behavior
B. anticholinergic effects D. arrhythmias
(answer: B /reference p. 466 / MPL 60)

39. Switching of moods in manic depressive psychotic patients may most likely occur with the use of this antidepressant:
A. serotonin reuptake inhibitors C. tricyclic antidepressants
B. bupropion & other atypical antidepressants D. MAO inhibitors
(answer: C /reference p. 467 / MPL 60)

40. Which of the following H1 receptor antagonists causes the least sedation?
A. clemastine C. chlorpheniramine
B. hydroxyzine D. loratadine
(answer: D /reference p. 655 / MPL 90)

41. This second generation H1 antagonist may induce torsades de pointes when concurrently taken with drugs that inhibit the 3A family of P450
enzymes in the liver:
A. Loratadine C. Terfenadine
B. Cetirizine D. Azelastine
(answer: C /reference p. 655 / MPL 60)

42. Which of the following H1 antagonists has both antihistamine and antiserotonin activity?
A. Diphenhydramine C. Cyproheptadine
B. Pyrilamine D. Meclizine
(answer: C /reference p. 656 / MPL 60)

43. X-ray examination of the knee joints in a 73 year old female reveals arthritic changes. Which of the following should NOT be prescribed?
A. Aspirin C. Indomethacin
Pharmacology
38 / 68
B. Acetaminophen D. Diclofenac
(answer: B /reference p. 703 / MPL 90)

44. Irreversible acetylation of both COX-1 and COX-2, is the characteristic mechanism of action of this NSAID:
A. Mefenamic acid C. Indomethacin
B. Celecoxib D. Aspirin
(answer: D /reference p. 691 / MPL 90)

45. When the goal of therapy is for prophylaxis against thrombotic episodes, Aspirin should be given at a subtherapeutic dose range of:
A. 40-80 mg daily C. 60-100 mg daily
B. 325-650 mg daily D. 80-325 mg daily
(answer: A /reference p. 701 / MPL 60)

46. Closure of patent ductus arteriosus in 70% of neonates can be expected with this drug:
A. Indomethacin C. Glucocorticoid
B. PGE1 D. Aspirin
(answer: A /reference p. 701 / MPL 60)

47. In rheumatic disease therapy, one of the following manifestations may be a reliable index of therapeutic plasma concentration of salicylates:
A. hypothermia C. blurred vision
B. tinnitus D. excessive sweating
(answer: B /reference p. 701 / MPL 60)

48. ACE inhibitors are associated with which of the following adverse reactions?
A. dry cough C. anemia
B. hypokalemia D. hepatitis
(answer: A/reference p. 828 / MPL 60)

49. A 56 year old diabetic patient with a BP of 150/90 will most likely benefit from this antihypertensive drugbecause it can prevent or delay progression
of renal disease:
A. Betaxolol C. Enalapril
B. Nifedipine D. Methyldopa
(answer: C /reference p. 827 / MPL 90)

50. The efficacy of ARBs with regard to lowering BP is comparable to that of ACE inhibitors. However, unlike ACE inhibitors, ARBs do not:
A. enhance BP lowering effect of other antihypertensives
B. produce cough.
C. fetopathic potential.
D. affect patients whose arterial BP is highly dependent on RAS.
(answer: B /reference p. 833 / MPL 60)

51. Which of the following is the preferred drug for treatment of hypertension during pregnancy but may cause hemolytic anemia with prolonged use:
A. Minoxidil C. Propranolol
B. Nifedipine D. Methyldopa.
(answer: D /reference p. 879 / MPL 60)

52. The mechanism by which beta adrenergic receptor antagonist drugs reduce BP include all of the following, EXCEPT:
A. delayed normalization of vascular resistance in the presence of a persistently reduced cardiac output.
B. reduction of Angiotensin II production as a result of decreased renin secretion
C. altered sympathetic outflow at the CNS level
D. vasodilatation as direct effect on vascular wall
(answer: D /reference p. 883 / MPL 60)

53. A common untoward effect of the older less soluble sulfonamides:


A. agranulocytosis C. hypersensitivity reaction
B. aplastic anemia D. crystalluria
(answer: D /reference p. 1175 / MPL 90)

54. A synergistic ratio of 20:1 sulfonamide with another drug combination that provides greater antibacterial spectrum / less bacterial resistance against
systemic infections:
A. Silver : Sulfadiazine C. sulfadoxine: Pyrimethamine
B. Sulfamethoxazole: Trimethoprim D. Pyrimethamine : Sulfadiazine
(answer: B /reference p. 1177 / MPL 60)

55. An advantage of triazole antifungals over the imidazoles is:

Pharmacology
39 / 68
A. greater inhibition of fungal sterol 14-a-demethylase
B. do not produce nausea and vomiting
C. less effect on human sterol synthesis
D. lesser toxicity due to rapid metabolism
(answer: C /reference p. 1301 / MPL 60)

56. A dermophyte-selective antifungal drug that utilizes sweat and transepidermal fluid loss as its mode of transfer from keratin precursors to the stratum
corneum:
A. Ketoconazole C. Terbinafine
B. Griseofulvin D. Potassium iodide
(answer: B /reference p. 1306 / MPL 60)
.
57. The first effective antiretroviral agent that is recommended in HIV-infected pregnancy to prevent prenatal transmission of virus :
A. Zidovudine C. Stavudine
B. Didanosine D. Lamivudne
(answer: A /reference p. 1355 / MPL 90)

58. When faced with a patient with pernicious anemia, which of the following manifestations would make you decide to prioritize on Vit B12 therapy
instead of folic acid only?
A. presence of neurologic abnormalities
B. absence of alcoholism in the patient
C. patient is strictly vegetarian
D. a (+) macrocytic hyperchromic blood profile
(answer: A /reference p. 1508-1509 / MPL 90)

59. Epoietin alfa is effective in the treatment of anemias associated with:


A. chronic renal failure
B. pregnancy
C. mixed parasitism
D. a highly competitive athlete to improve performance
(answer: A /reference p. 1489-1491 / MPL 90)

60. This anticoagulant is the drug of choice for anticoagulation in pregnancy because it does not cross the placenta:
A. Dicumarol C. Heparin
B. Warfarin D. Phenprocoumon
(answer: C /reference p. 1523 / MPL 90)

61. A pure estrogen antagonist that is mainly used as a fertility pill to induce ovulation:
A. Tamoxifen C. Toremifene
B. Raloxifene D. Clomiphene
(answer: D /reference p. 1613-1614 / MPL 90)
.
62. The major indication for testosterone treatment is:
A. male senescence C. male hypogonadism
B. to improve athletic performance D. male contraception
(answer: C /reference p. 1635 -1643/ MPL 90)

63. Inflammatory skin lesions may be treated with this glucocorticoid through intralesional administration:
A. triamcinolone hexacetonide C. dexamethasone sodium phosphate
B. betamethasone dipropionate D. hydrocortisone valerate
(answer: A /reference p. 1800 / MPL 90)

64. A commonly used parenteral anesthetic that is advantageous for procedures where rapid return to a preoperative mental status is desired:
A. Ketamine C. Etomidate
B. Propofol D. Thiopental
(answer: B /reference p. 347 / MPL 60)
.
65. Sumatriptan is effective in the treatment of migraine headache by acting as:
A. selective agonist at 5-HT!D and 5HT1B receptors
B. agonist at alpha-1 & 2 adrenergic receptors
C. selective antagonist at dopaminergic receptors
D. antagonist at beta-1 & 2 adrenergic receptors
(answer: A /reference p. 280 / MPL 60)

Pharmacology
40 / 68
66. Vectors for gene transfer include the following, except:
A. naked plasmid DNA
B. genetically engineered herpes simplex virus
C. genetically engineered Moloney murine leukemia retrovirus
D. RNA-coated gold particles
(answer: D/reference p. 82/ MPL 60)

67. Combination of penicillin and tetracycline for the treatment of acute tonsillo- pharyngitis may result to:
A. greater incidence of superinfection
B. total bacteriostatic effect
C. an effective cure due to enhanced bactericidal activity
D. a state of indifference as regards antibacterial effect
(answer: A /reference p. 1163/ MPL 60)

68. The MOA of choloroquine in P. falciparum malaria is elimination of:


A. sporozoites C. exoerythrocytic schizonts
B. erythrocytic stage D. secondary tissue schizonts
(answer: B /reference p. 1077 / MPL 60)

69. A Filipino OCW went to a South African jungle region to work where cholorquine resistant P. falciparum is endemic. He took a drug for prophylaxis
but he still developed severe attack of P. vivax malaria. The drug he took for prophylaxis was most likely:
A. cholorquine C. proguanil
B. atovaquone D. mefloquine
(answer: A/reference p. 1079 / MPL 60)

70. A 51 year old female teacher who has just finished chemotherapy for lung cancer is admitted in the hospital for fever and chills. Gram stain of blood
reveals gram negative bacilli. History revealed she developed urticarial rashes with breathing difficulty following a parenteral penicillin injection
for an infection more than 3 months ago. Which of the following is the most appropriate antibiotic for empiric treatment of this patient?
A. Piperacillin C. Imipenem +cilastatin
B. Aztreonam D. Ceftriaxone
(answer: B /reference p. 1214 / MPL 60)

71. One of the mechanisms associated with bacterial resistance to penicillin is :


A. ability of bacteria to produce an acid medium
B. increase renal excretion of penicillin
C. bacterial production of lyzozymes
D. alteration of PBPS
(answer: D /reference p. 1192/ MPL 90)

72. Which of the following cephalosporins would have increased activity against Bacteroides fragilis?
A. Cefoxitin C. Cefaclor
B. Cefamandole D. Cephalothin
(answer: A /reference p. 1210/ MPL 60)

73. Toxic effects of corticosteroids do not include:


A. salt retention C. hypoglycemia
B. inhibition of growth D. hypertension
(answer: C /reference p. 1667/ MPL 90)

74. The use of epinephrine in anaphylactic shock is an example of:


A. physiological antagonism C. dispositional antagonism
B. chemical antagonism D. receptor antagonism
(answer: A /reference p. 73/ MPL 60)

75. Epinephrine when mixed with anesthetics like procaine will:


A. facilitate their distribution along nerve endings
B. stimulate wound repair
C. retard their systemic absorption
D. promote hemostasis
(answer: C /reference p. 372/ MPL 90)

76. Which of the following statements is not true regarding dobutamine?


A. it activates dopaminergic receptors in renal and mesenteric vascular beds
B. it is used to increase cardiac output in patients with severe cardiac failure

Pharmacology
41 / 68
C. it is administered by IV infusion
D. it is a selective agonist at B1 receptors at low doses
(answer: A /reference p. 228/ MPL 60)

77. A 56-year-old male executive with a 2 year history of hypertension is given prazosin which is a/an:
A. calcium channel antagonist C. beta adrenergic antagonist
B. alpha adrenergic antagonist D. sodium channel antagonist
(answer: B /reference p. 246/ MPL 60)

78. Which of the following drugs act within sympathetic neurons to depress neurotransmitter release thus is used to treat hypertension?
A. metoprolol C. methyldopa
B. phentolamine D. nifedipine
(answer: C /reference p. 877/ MPL 60)

79. A 55 year old male alcoholic with early liver cirrhosis requires suturing for a deep scalp laceration. Which of the following local anesthetic
could potentially be toxic to this patient?
A. benzocaine C. procaine
B. tetracaine D. bupivacaine
(answer: D /reference p. 374/ MPL 60)

80. Which of the following is the drug of choice for a 14-year-old male who abruptly experienced impairment of consciousness with blank stare lasting
over 30 seconds?
A. Phenobarbital C. Clonazepam
B. Diazepam D. Phenytoin
(answer: C /reference p. 538/ MPL 60)

81. An 18 year old female epileptic who is on maintenance medication is noted to have increasing drowsiness and inattentiveness. She has
gingival hyperplasia and ataxic gait. What medication is she taking?
A. Phenytoin C. Carbamazepine
B. Phenobarbital D. Valproate
(answer: A/reference p. 530/ MPL 90)

82. Which of the following diuretics should not be used in the presence of hyperkalemia?
A. acetazolamide C. chlorothiazide
B. ethacrynic acid D. spironolactone
(answer: D /reference p. 780/ MPL 90)

83. Which of the following diuretics will inhibit Na+/K+/2Cl- cotransporters in the ascending limb of Henle’s loop?
A. chlorthalidone C. acetazolamide
B. ethacrynic acid D. mannitol
(answer: B /reference p. 769/ MPL 90)

84. The cytoprotective action of misoprostol on the GI mucosa is because it:


A. neutralizes the acid secretion
B. coats the mucosa
C. enhance secretion of mucin and bicarbonate
D. relieves ulcer symptoms
(answer: C /reference p. 1011/ MPL 70)

85. The mechanism of action of omeprazole is mainly based on its action on:
A. H+ - K+ - ATPase
B. gastric mucosal carbonic anhydrase
C. H2 receptors of parietal cells
D. ulcer craters
(answer: A /reference p. 1007/ MPL 90)

86. Which of the following laxatives is not properly matched with its mechanism of action?
A. lactulose: non absorbable sugar which draws water into the lumen
B. mineral oil: penetrates and softens the stool
C. docusate sodium: 5HT4 receptor agonist
D. methylcellulose: retention of intraluminal fluid by hydrophilic mechanism
(answer: C /reference p. 1046/ MPL 60)

87. The prokinetic effect of dromperidone is due to:


A. increased availability of acetylcholine from myenteric motor neurons
Pharmacology
42 / 68
B. dopamine (D2) receptor antagonism
C. serotonin (5HT4) receptor antagonism
D. muscarinic receptor activation
(answer: B A/reference p. 1024/ MPL 60)

88. What is the “on/off phenomenon” associated with prolonged used of levodopa?
A. the duration of effect exceeds the plasma life time of drug
B. with time, the “buffering” capacity of drug is lost
C. fluctuation in clinical response irrespective of drug levels
D. improvement of clinical response after a drug holiday
(answer: C /reference p. 557/ MPL 60)

89. Spasticity can be alleviated with which of the following drugs?


A. baclofen C. selegiline
B. bromocriptine D. amantadine
(answer: A /reference p. 565/ MPL 60)

90. The drug of choice for a serious digitalis induced arrhythmia is:
A. quinidine C. antidigoxine Fab fragments
B. atropine D. cardiac pacing
(answer: C /reference p. 958/ MPL 60)

91. Which of the following is leukotriene – receptor antagonist?


A. montelukast C. cromolyn
B. zileuton D. nedocromil
(answer: A /reference p. 740/ MPL 60)

92. The drug that is proven to be effective in the immediate treatment of severe asthma exacerbation is:
A. inhaled glucocorticoids
B. inhaled B-adrenergic agonist
C. aminophylline
D. aminophylline + B-adrenergic agonist
(answer: B /reference p. 747/ MPL 90)

93. Treatment of hypotonicity in SIADH include which of the following?


A. unrestricted dietary salt C. demeclocycline
B. spironolactone D. intranasal desmopressin
(answer: C /reference p. 803/ MPL 60)

94. Prevention of transplant rejection by cyclosporine is due to which of the following mechanism?
A. suppression of humoral immunity
B. suppression of T cell dependent immune mechanism
C. inhibition of innate immune response
D. inhibition of B lymphocyte production of antibody
(answer: B /reference p.1466/ MPL 60)

95. Which of the following immunosuppressive drugs is not correctly matched to its toxicity?
A. corticosteroid: avascular necrosis of bone
B. tacrolimus: diabetes mellitus
C. azathioprine: increased susceptibility to infections
D. cyclosporin: bone marrow depression
(answer: D /reference p.1469/ MPL 60)

96. Aminoglycosides are excreted from the body by which route?


A. tubular secretion C. glomerular filtration
B. biliary secretion D. hepatic metabolism
(answer: C /reference p. 1226/ MPL 90)

97. Which of the following aminoglycosides is most resistant to inactivation by aminoglycoside-inactivating enzymes?
A. gentamicin C. streptomycin
B. tobramycin D. amikacin
(answer: D /reference p. 1233/ MPL 60)

98. Which of the following is used as prophylaxis of meningococcal disease?


Pharmacology
43 / 68
A. rifampin C. erythromycin
B. gentamicin D. clindamycin
(answer: A /reference p. 1278/ MPL 90)

99. Which of the following is not a toxic effect of amikacin?


A. hearing impairment due to loss of hair cells in the cochlea
B. neuromuscular blockade
C. postural imbalance due to irreversible vestibular damage
D. nephrotoxicity
(answer: C /reference p. 1228/ MPL 70)

100. Which of the following oral antidiabetic drugs does not cause hypoglycemia?
A. metformin C. chlorpropamide
B. repaglinide D. gliclazide
(answer: A /reference p. 1705/ MPL 70)

__A__ The measure of the body’s ability to eliminate drugs is termed as:
A. Clearance C. Potency
B. Volume of distribution D. Half-life

__C__ Rationale of giving a loading dose:


A. To improve patient’s compliance
B. To minimize the incidence of adverse effects
C. To achieve the target concentration of the drug rapidly
D. To facilitate renal excretion

__D__ The maximal response produced by a drug is termed as:


A. Agonist C. Slope
B. Potency D. Maximal efficacy

__C__ The difference between the effects of IV infusion of epinephrine and norepinephrine
on the cardiovascular system is:
A. Epinephrine increase pulse pressure
B. Norepinephrine decreases renal blood flow
C. Small doses of norepinephrine do not cause vasolidation
D. Cutaneous blood flow is decreased with epinephrine

__C__ All of the following are common adverse effects of systemically administered β -adrenergic
agonists, EXCEPT:
A. Restlessness C. Bradycardia
B. Hypokalemia D. Tremors

__D__ Which of the following adrenergic receptor antagonists produces bronchoconstriction?


A. Metropolol C. Yohimbine
B. Terazosin D. Nadolol

__C__ Which of the following glands is stimulated by Acetylcholine?


A. Thyroid gland C. Salivary gland
B. Adrenal gland D. Pituitary gland

__A__ Which of the following cholinergic agonists does not produce negative inotropism and
negative chronotropism?
A. Bethanecol C. Methacholine
B. Carbachol D. Acetylcholine

__D__ Which of the following muscarinic agonists is least antagonized by Atropine?


A. Acetylcholine C. Bethanecol
B. Pilocarpine D. Carbachol

MULTIPLE CHOICE: Write the letter, which corresponds to the BEST answer.

__D__ Which of the following is NOT a pharmacological effect mediated by muscarinic receptors?
A. Carbachol – increased GIT tone and motility
B. Bethanecol – contraction of the detrusor muscle of the bladder
C. Pilocarpine – diaphoresis
D. Arecoline – diuresis

__C__ Bacterial resistance to Tetracycline could be due to:


A. Incomplete absorption when given orally
B. Failure to pass through the blood-brain barrier
C. Decreased access of the agents to the ribosome
Pharmacology
44 / 68
D. Extensive first-pass effect

__D__ Which of the following agents compete with Chloramphenicol for binding to 50S ribosomal unit?
A. Vancomycin C. Ticarcillin
B. Imipenem D. Clindamycin

__A__ All of the following antiarrthythmics have autonomic effects. Which of the following
produces adrenergic stimulation followed by ganglionic blockade?
A. Bretylium C. Disopyramide
B. Procainamide D. Amiodarone

__D__ The following are antiarrhythmics with their corresponding major actions. Which of the following combinations is correct?
A. Lidocaine – Ca+2 channel block
B. Verapamil – Beta-blockade
C. Metoprolol – Na+ channel block
D. Amiodarone – K+ channel block

__C__ All of the following increase action potential, EXCEPT:


A. Procainamide C. Lidocaine
B. Quinidine D. Dispoyramide

__B__ Which of the following statements is true of low-molecular weight heparin?


A. It is absorbed through the GIT
B. Monitoring of its effect is not done routinely
C. Have similar histological half-life with the standard preparation of the drug
D. More effective thus, standard heparin for prophylaxis of thromboembolism

__A__ Which of the following statements is true of Warfarin?


A. Crossed the placenta
B. Protamine reverses its anticoagulant effect
C. Can be given IV
D. Is a vasodilator

MULTIPLE CHOICE: Write the letter, which corresponds to the BEST answer.

__D__ A greater risk of respiratory depression is produced when Morphine is combined with:
A. Local anesthetics C. Bronchodilators
B. Anorexiants D. Tranquilizers

__D__ The primary site of action of local anesthetics is the:


A. Cytoplasm C. Golgi apparatus
B. Nucleus D. Cell membrane

__A__ Which of the following statements is true of the sensitivity of nerve fibers to local anesthetics?
A. Small nerve fibers are more susceptible to the action of local anesthetics
B. Unmyelinated fibers are blocked before myelinated ones of the same diameter
C. A fiber of the AB type is more sensitive to block than C fibers (dorsal root)
D. Nerve fibers located at the core of a nerve trunk are first to be affected than those located at the periphery

__A__ The following are anticonvulsants with their corresponding sites of action. Which of the
following combinations is correct?
A. Phenobarbital – GABAA receptor
B. Phenytoin – CA2+ currents (T currents)
C. Ethosuximide – CA2+ currents (L current)
D. Carbamazepine – K+ currents

__B__ All are indications for the use of Carbamazepine, EXCEPT :


A. Trigeminal neuralgia C. Bipolar disorders
B. Absene seizures D. Diabetic polyneuropathy

__C__ Which of the following is a selective inhibitor of MAO-B?


A. Amantadine C. Selegiline
B. Bromocriptine D. Pergolide

__C__ Which of the following combination of diuretics and MOA is correct?


A. Furosemide – inhibition of carbonic anhydrase
B. Acetazolamide – osmotic effect
C. Hydrocholorothiazide – inhibition of Na+ -Cl- symport
D. Indapamide – inhibition of Na+ -K+ -2Cl- symport

__A__ Which of following combination of diuretic and site of action on the nephron is correct?
A. Ethacrynic acid – thick ascending limb of loop of Henle
B. Furosemide – proximal tubule
C. Hydrochlorothiazide – late distal tubule
D. Spironolactone – distal convoluted tubule (DCT)

Pharmacology
45 / 68
MULTIPLE CHOICE: Write the letter, which corresponds to the BEST answer.

__D__ Reflex tachycardia is the most common consequence of which of the following anti- hypertensive drugs?
A. Metoprolol C. Valsartan
B. Diltiazem D. Nifedipine

__C__ The mechanism of cough production after administration of an ACE inhibitor is secondary to an increase in the level of:
A. Angiotensin II C. Bradykinin
B. Renin D. Aldosterone

__A__ The mechanism of the anti-hypertensive effect of Clonidine is brought about by:
A. α 2-adrenergic agonism
B. β 1-adrenergic blockade
C. Angiotensin II receptor blockade
D. Direct relaxation of the arteriolar smooth muscle

__A__ The basis of the anti-arrhythmic action of digitalis is:


A. Decrease in automaticity and prolongation of the ERP in the AVN
B. (+) inotropic effect
C. Increased sympathetic nervous system activity
D. Increased in the spontaneous (phase 4) rate of diastolic depolarization

__D__ All of the following are factors that alter patient sensitivity to Digitalis, EXCEPT :
A. Hypokalemia C. Renal failure
B. Hyperkalemia D. Euthyroid state

__B__ Which of the following drugs is the least useful in Acute Congestive Heart Failure?
A. Captopril C. Digoxin
B. Metoprolol D. Furosemide

__C__ The most effective route of administration of nitrates to abort an acute attack of angina is:
A. Intravenous route C. Sublingual route
B. Oral route D. Transdermal route

__C__ The most important mechanism of relief of symptoms of angina pectoris after administration of nitrates is:
A. Dilation of epicardial coronary arteries
B. Decreased preload
C. Reduction of myocardial work
D. Decreased afterload

__B__ The active form of statins:


A. β -hydroxylacetic acid C. α -hydroxy acid
B. β -hydroxy acid D. α -hydroxylacetic acid

MULTIPLE CHOICE: Write the letter, which corresponds to the BEST answer.

__A__ The prototype fibric acid derivative :


A. Clofibrate C. Bensafibrate
B. Ciprofibrate D. Fenofibrate

__B__ Which of the following statements regarding aminoglycosides is true?


A. Anaerobic microorganism are sensitive to these agents
B. The duration of postantibiotic effect is concentration-dependent
C. Rapidly absorbed in the gastric mucosa
D. Contraindicated in patients with auditory impairment

__B__ Which of the following statements concerning Sulindac is true?


A. Selectively inhibits COX-2
B. It is a prodrug
C. Has a narrow margin of safety
D. Binding to plasma protein is negligible

__C__ Inhibition of these prostaglandins is responsible for the GI adverse effects of NSAIDs:
A. Thromboxane A2 C. PGI2 and PGE3
B. PGF2 and PGI2 D. PGE1 and PGD2

__D__ Toxic doses of Aspirin will produce:


A. Compensated respiratory alkalosis
B. Metabolic alkalosis
C. Compensated respiratory acidosis
D. Metabolic acidosis

__D__ Which of the following is a polypeptide autocoid?


A. Prostaglandins C. Leukotriene E4
B. Serotonin D. Substance P
Pharmacology
46 / 68

__D__ Serotonin is derived from dietary:


A. Histidine C. Glycine
B. Methionine D. Tryptophan

__B__ Which of the following statements is true of H2 histamine receptor antagonists?


A. The inhibition is non-selective
B. The degree of inhibition of gastric secretion parallels the concentration of drug in the plasma
C. Inhibit only the basal (fasting) but not nocturnal acid secretion
D. The inhibition is non-competitive

MULTIPLE CHOICE: Write the letter, which corresponds to the BEST answer.

__D__When given in a sufficient dosage, Omeprazole can reduce the daily production of gastric acid by more than 95%. This is because Omeprazole:
A. Inhibits the interaction of histamine with H2 receptors
B. Neutralizes the directly the HCl secreted by gastric parietal cells
C. Increases the secretion of mucus and bicarbonate
D. Inhibits the H+, K+ -ATPase, the ultimate mediator of acid secretion

__C__ The following statements are true of Sucralfate, EXCEPT :


A. Adheres so avidly to the base of the ulcer crater than in normal epithelial cells
B. Adheres to ulcerated epithelium for longer than 6 hours
C. Cytoprotective effect is enhanced when combined with a “proton pump” inhibitor
D. None of the above

__B__Which of the following drug-adverse effect combination is true?


A. Streptomycin – optic neuritis
B. Pyrazinamide – hyperuricemia
C. Ethambutol – ototoxicity
D. Rifampin – nephrotoxicity

__A__ The concomitant use of Pyridoxine is recommended when this drug is used for anti-TB therapy:
A. Ethionamide C. Kanamycin
B. Aminosalicylic acid D. Capreomycin

__C__ Which of the following statements is true for Isoniazid?


A. It is poorly absorbed from the GIT
B. It cannot penetrate caseous material
C. It diffuses readily into all body fluids and cells
D. It is excreted in the urine largely in an unchanged form

__C__ All of the following are clinical uses of Rifampin, EXCEPT :


A. Prevention of meningococcal disease
B. Chemoprophylaxis of household contacts of a patient with H, influenzae meningitis
C. Treatment of MDR-TB
D. Treatment of nasal carriage of S, aureus
__B__ An antiviral agent that alters Dopamine release or re-uptake:
A. Selegiline C. Bromocriptine
B. Amantadine D. Pergolide

__D__ Which of the following has anticholinergic properties, currently used in the treatment of
Parkinson’s disease?
A. Carbidopa C. Bromocriptine
B. Selegiline D. Trihexyphenidyl HCl

MULTIPLE CHOICE: Write the letter, which corresponds to the BEST answer.

__C__ Which of the following statements concerning Chloramphenicol is true?


A. A derivative of hydrochloric acid
B. Produces inhibition of protein synthesis by irreversible binding to the 50S ribosomal subunit.
C. The major route of elimination is hepatic metabolism to the inactive glucoronide
D. It reaches therapeutic concentrations only in the presence of meningitis

__C__ Which of the following statements concerning Erythromycin is true?


A. E. Coli accumulates 100x more Erythromycin than Streptococcus species
B. Absorption from the GIT is hastened by the presence of food
C. Antibacterial activity is achieved at all sites except the brain and CSF
D. Excreted as the inactive form in the bile

__C__ Which of the following quinolones can cause fatal hepatic damage?
A. Levofloxacin C. Trovafloxacin
B. Gatifloxacin D. Cirpofloxacin

__C__ Which of the following antifungal agents act by inhibiting thymidylate synthetase resulting in
Pharmacology
47 / 68
impairment of fungal DNA synthesis?
A. Ketoconazole C. Flucytosine
B. Griseofulvin D. Amphotericin B

__D__ Which of the following statements concerning Fluconazole is true?


A. Less water-soluble than Ketoconazole and Itraconazole
B. Effective against Candida krusei infections
C. More toxic than Flucytosine
D. >90% of a dose is eliminated through the kidneys

__B__ Which of the following oral antifungal agents has the longest half-life?
A. Ketoconazole C. Fluconazole
B. Itraconazole D. Miconazole

__B__ Which of the following statements concerning antiviral agents is true?


A. Most agents are associated with minor adverse effects in humans
B. They target a specific viral protein, usually an enzyme, essential in nucleic acid synthesis
C. Current effective agents inhibit both replicating and nonreplicating viruses
D. In vitro sensitivity tests can predict response to treatment
__C__ Which of the following is a prodrug?
A. Acyclovir C. Famiclovir
B. Penciclovir D. Ganciclovir

MULTIPLE CHOICE: Write the letter, which corresponds to the BEST answer.

__C__ The most common mechanism of resistance of HSV to Acyclovir and Valacyclovir:
A. Altered thymidine kinase susbtrate specificity
B. Altered viral DNA polymerase
C. Absence or partial production of thymidine kinase
D. Active efflux mechanism

__B__ Which of the following statements concerning Amantadine and Rimantadine is true?
A. Treatment of Influenza A and B infections
B. Dose adjustments must be made for patients >65 years old
C. Absorption is markedly decreased by the presence of food in the stomach
D. Remains effective if given >48 hours after the onset of symptoms

__D__ Which of the following statements concerning 5-HT agonists is correct?


A. 90% of a dose is excreted through the kidneys so there is a need for dosage adjustment for patients with renal impairment
B. Concurrent administration of caffeine increases the bioavailability by about 50%
C. Exclusively interact with serotonergic receptors in the brain
D. Use is restricted to patients who have frequent, moderate or infrequent, severe migraine attacks

__C__ The triptans are contraindicated in which of the following:


A. A patient with a mild and transient renal impairment
B. A patient with stable hypertension
C. A patient with a history of ischemic heart disease
D. A patient taking an ACE inhibitor

__B__ The antidote for cyanide poisoning is:


A. 100% O2 C. Carbocysteine
B. Amyl nitrite D. Thiosulfate

__A__ The chelating agent of choice for iron poisoning is:


A. Deferoxamine C. Dimercaprol
B. Penicillamine D. EDTA

__A__ The smallest dose of a given chemical that will kill 50% of a test group of animals:
A. Median lethal dose (LD50)
B. Median lethal concentration (LC50)
C. Toxicity level
D. Threshold limit value (TLV)

__C__ Which of the following statements regarding the pharmacokinetic properties of


Metronidazole is correct?
A. Penetrates well into the placenta
B. Therapeutic concentrations are not achieved in the CSF
C. Oral, intravenous, intravaginal and topical formulations
D. Hepatic metabolism is increased by the concurrent administration of Cimetidine

MULTIPLE CHOICE: Write the letter, which corresponds to the BEST answer.

__C__ Which of the following is the most common adverse effect of Metronidazole?
A. Pruritus C. Metallic taste
Pharmacology
48 / 68
B. Dizziness D. Nausea

__C__ Which of the following statements concerning hormones is correct?


A. Vasopressin regulates potassium osmolality
B. Cortisol controls fat metabolism
C. PTH regulates calcium and phosphorus levels
D. Insulin maintains euglycemia only in the fasted state

__C__ Which of the following organs is NOT part of the feedback regulation of the endocrine axes?
A. Adrenal glands C. Kidneys
B. Hypothalamus D. Pituitary gland

__B__ Which of the following corticosteroids has the greatest anti-inflammatory potency relative to cortisol?
A. Prednisone C. Aldosterone
B. Dexamethasone D. Triamcinolone

__C__ Which of the following corticosteroids has the longest duration of action?
A. Cortisol C. Dexamethasone
B. Prednisone D. Aldosterone

__B__ Which of the following statements concerning the use of testosterone for the treatment of male hypogonadism is correct?
A. The testosterone esters are not useful
B. Treatment efficacy is monitored by regular and frequent measurements of serum testosterone concentration
C. Hepatocellular cancer is a common adverse effect usually seen in men >55 years old
D. Increased bone mineral density is the earliest treatment effect seen, usually within a few weeks of treatment

__C__ Which of the following cell structures is exclusively found in gram-negative microorganisms?
A. Peptidoglycan layer C. Lipopolysaccharide (LPS)
B. Penicilline binding protein D. Phospholipid

__A__ Factors that can appreciably influence the ability of β -lactam antibiotics to kill bacteria include the following, EXCEPT:
A. Presence of proteins and other constituents of pus at the site of infection
B. Density of the bacterial population
C. Age of the infection
D. None of the above

MULTIPLE CHOICE: Write the letter, which corresponds to the BEST answer.

__A__ Which of the following statements regarding Penicillins is true?


A. Gastric juice at pH 2 rapidly destroys the oral formulation of the antibiotic
B. Penicillin V is 5-10x more active against Neisseria species than Penicillin G
C. Approximately 60% of Penicillin G in plasma is irreversibly bound to albumin
D. Penicillin readily enters the CSF when the meninges are normal

__B__ Prophylactic uses of Penicillins include:


A. Gram-negative sepsis
B. Recurrences of Rheumatic Fever
C. Treatment of Syphilis
D. Treatment of Rat Bite Fever

__C__ Which second generation cephalosporin can be used for the treatment of bacterial meningitis?
A. Cefriaxone C. Cefuroxime
B. Cefatoxime D. Ceftizoxime

__C__ Which of the following agents is a monocyclic β -lactam compound is isolated from
Chromobacterium violaceum, that is usefule for gram-negative infections and does not cause
allergic reactions seen in some patients given Penicillins or other cephalosporins?
A. Imipenem C. Aztreonam
B. Cefepime D. Tazobactam

__C__ Which of the following sulfonamides has the longest half-life and is therefore long-acting?
A. Sulfadiazine C. Sulfadoxine
B. Sulfacetamide D. Sulfisoxazole

__B__ Which of the following disorders of the hematopoietic system secondary to Sulfonamides is
characterized by the complete suppression of bone marrow activity with profound anemia,
granulocytopenia and thrombocytopenia?
A. Acute hemolytic anemia C. Agranulocytosis
B. Aplastic anemia D. Idiopathic anemia

__D__ Which of the following statements regarding Trimethoprim is correct?


A. A potent and selective competitive inhibitor or microbial dihydrofolate synthase
B. A potent competitive inhibitor or microbial dihydropteroate synthase
C. Volume of distribution is almost the same with that of Sulfamthoxazole
D. In combination with Sulfamethaxazole, it readily enters the CSF and sputum
Pharmacology
49 / 68

__C__ The parent compound of barbiturates:


A. Trichloreoethanol C. Barbituric acid
B. Oxybarbiturate D. Phenobarbital

__C__ Manifestations of acute barbiturate overdosage include:


A. Seizures C. Respiratory depression
B. Increased salivation D. Renal failure

MULTIPLE CHOICE: Write the letter, which corresponds to the BEST answer.

__C__ Rationale for incliusion of progestin in HRT:


A. To prevent withdrawal bleeding
B. To lower the incidence of thromboembolism
C. To prevent endometrial hyperplasia
D. To prevent osteoporosis

__D__ The progestin with the least androgenic activity:


A. Norgestrel C. Medroxyprogesterone
B. Ethynodiol diacetate D. Desogestrel

__B__ Which of the following statements is true agranulocytosis following the use of antithyroid drugs?
A. Periodic WBC count is greatly of help in early detection
B. It is reversible upon discontinuation of the drug
C. Occurs during the first week, not after 1 month
D. Mostly seen with Methimazole than with PTU

__A__ In severe hyperthyroid state or thyroid storm, which of the following antithyroid is preferred?
A. PTU C. Propanolol
B. Carbimazole D. Methimazole

__D__ A.C. has hyperthyroidism. She is 12 weeks pregnant. Which of the following antithyroid drugs is preferred?
A. Carbimazole C. RAI
B. Methimazole D. PTU

__D__ Which of the following has the potential of producing dose-related malabsorption, flatulence and abdominal bloating?
A. Metformin C. Rosiglitazone
B. Chlorpropamide D. Acarbose

__B__ Which of the following statements is true of Metformin?


A. An antihyperglycemic and hypoglycemic
B. Absorbed mainly from the small intestine
C. Causes insulin release from the pancreas
D. 20% of the drug is bound to albumin

__B__ Insulin usually is injected into the subcutaneous tissue. Absorption is usually most rapid from the :
A. Thigh C. Buttocks
B. Abdominal wall D. Arm

MULTIPLE CHOICE: Write the letter, which corresponds to the BEST answer.

__B__ Which of the following statements concerning the physiologic effects of oxytocin is correct?
A. The effects on the uterus are not affected by estrogen levels
B. The increase in uterine sensitivity in the last half of the pregnancy is associated with an increase in oxytocin receptors
C. Uterine sensitivity during the last trimester of pregnancy is due to the increasing levels of progesterone.
D. The more immature the uterus, the more sensitive it is to the effects

__D__ Which of the following statements concerning the use of oxytocin for induction of labor is correct?
A. As labor progresses, the dose required to maintain good uterine contractions increases
B. Higher doses promote diuresis resulting in hypotension
C. Definitely useful in cases of arrested labor due to abnormal fetal positions
D. Satisfactory uterine contractions are unlikely to develop with doses higher than 30-40 mU/min.

__B__ Which of the following bensimidazoles can cause hypospermia?


A. Thiabendazole C. Albendazole
B. Mebendazole D. None of the above

__A__ Which of the following anthelmintics possesses a potent inhibitory effect on the feeding behavior of worms by acting on glutamate-gated
channels expressed in the pharyngeal muscles of these worms?
A. Ivermectin C. Diethylcarbamazine
B. Thiabendazole D. Pyrantel pamoate

__B__ Which of the following statements concerning Praziquantel is correct?


A. The primary site of action is the parasite’s skeletal structure
B. The clinical efficacy correlates with the drug’s action on the parasitic tegument
C. Possesses a desirable taste making it the drug of choice for pediatric patients
Pharmacology
50 / 68
D. Undergoes minimal first-pass metabolism

__B__ Which of the following antiandrogens inhibits type II 5α -reductase and is used in the treatment of benign prostatic hyperplasia?
A. Flutamide C. Bicalutamide
B. Finasteride D. Cyproterone acetate

__C__ Which of the following antimalarial drugs is available in a fixed combination with Sulfadoxine?
A. Trimethoprim C. Pyrimethamine
B. Primaquine D. Proguanil

MULTIPLE CHOICE: Write the letter, which corresponds to the BEST answer.

__D__ Which of the following statements concerning Quinine is correct?


A. Poor absorption after an oral dose
B. Renal excretion is delayed in acidic urine
C. Gametocytocidal for all plasmodial species
D. Can commonly produce hypoglycemia which can be life-threatening

__C__ Which of the following antimalarial drugs should NOT be used in acute attacks?
A. Quinine C. Primaquine
B. Mefloquine D. Chloroquine

QUESTIONS IN PHARMACOLOGY

Question Answer Reference MPL


1. Drug A loses 80% of its effect before reaching the systemic circulation. This would most B Goodman and 0.5
probably happen in which mode of administration? Gilman 8th ed p.5
A. Rectal
B. Oral
C. Sublingual
D. Intravenous

2. Drugs A and B act on the same tissue or organ through independent receptors, resulting in C Goodman and 0.5
effects in opposite directions. This effect is known as Gilman 8th ed
A. Competitive antagonism pp.53-54
B. Irreversible antagonism
C. Physiologic antagonism
D. Chemical antagonism

3. If drug X is repeatedly administered at dosing intervals equal to its elimination half life, the B Goodman and 0.25
number of doses required for the plasma concentration of the drug to reach steady state is Gilman 8th ed
A. 1 – 3 pp.26-27
B. 4 – 5
C. 6 – 7
D. 8 – 9

4. This pharmacokinetic value most reliably reflects the amount of drug reaching the target tissue B Goodman and 0.5
after oral administration Gilman 8th ed p.25
A. Volume of distribution
B. Area under the blood concentration-time curve
C. Peak blood concentration
D. Time to peak blood concentration

5. It was determined that 90% of an oral 80-mg dose of a drug was absorbed in a 70 kg patient. D Goodman and 0.25
However, because of extensive biotransformation during its first pass through the portal Gilman 8th ed pp.5,
circulation, the bioavailability of the drug was only 25%. AAUMINF liver blood flow of 1500 21
mL/min, the hepatic clearance of this drug in this situation was
A. 50 mL/min
B. 310 mL/min
C. 760 mL/min
D. 1080 mL/min

6. Of the many names applied to a drug, the one that is the official name and refers only to that A Generics Act of 1.0
drug and not to a particular product is 1988
A. trade name
B. brand name
C. proprietary name
D. generic name

7. The route of excretion of drugs or their metabolic derivatives that is quantitatively the LEAST D Goodman and 0.5
significant is which of the following? Gilman 8th ed
A. Biliary tract pp.18-20
B. Kidneys
C. Lungs
D. Milk
Pharmacology
51 / 68
8. The table below shows the effective dose –50s (ED50) and therapeutic indices (TI) of drugs A C Goodman and 0.50
to D. They have the same therapeutic effects through a common mechanism of action. Gilman 9th ed p.44

Drug ED50 TI
A 20 20
B 33 10
C 9 8
D 15 10

Which drug is the most potent?


A. Drug A
B. Drug B
C. Drug C
D. Drug D

9. The table below shows the effective dose –50s (ED50) and therapeutic indices (TI) of drugs A A Goodman and 0.25
to D. They have the same therapeutic effects through a common mechanism of action. Gilman 8th ed p.68

Drug ED50 TI
A 20 20
B 33 10
C 9 8
D 15 10

Which drug is the safest?


A. Drug A
B. Drug B
C. Drug C
D. Drug D

10. Which of the following drug interactions results to a decreased drug effectiveness due to A Goodman and 0.5
chemical interaction? Gilman 8th ed
A. Tetracycline and mild p.1119
B. Amobarbital and secobarbital
C. Soap and benzalkonium chloride
D. Sulfamethoxazole and trimethoprim

11. A newborn suffers from jaundice after being treated with sulfonamides. This is due to C Goodman and 0.25
A. Enhanced synthesis of bilirubin Gilman 8th ed.
B. Inhibition of bilirubin degradation p.1037
C. Displacement of bilirubin bound from albumin
D. Inhibition of urinary excretion of bilirubin

12. A patient taking an antacid experiences decreased response to a protein synthesis inhibitor A Goodman and 0.5
antibiotic. The antibiotic is most probably Gilman 8th ed
A. Tetracycline p.1119
B. Chloramphenicol
C. Sulfamethoxazole
D. Penicillin

13. A health worker is going to towns which are endemic for Plasmodium vivax and P. ovale. What D Goodman and 0.5
should he use for causal prophylaxis? Gilman 8th ed
A. Pyrimethamine pp.988 – 989
B. Quinacrine
C. Chloroquine
D. Primaquine
14. A stroke patient suffers from pneumonia after 3 weeks of hospitalization. Which of the following C Goodman and 0.50
may be a useful anti-infective agent in his case? Gilman 8th ed
A. Ofloxacin pp.1059 – 1060
B. Norfloxacin
C. Ciprofloxacin
D. Lomefloxacin

15. A 5-year old child suffers from neck and back stiffness with positive Kernig and Brudzinski D Goodman and 0.5
signs after a bout with pneumonia. Which drug would most probably be effective? Gilman 8th ed
A. Penicillin G pp.1090-1091
B. Penicillin VK
C. Norfloxacin
D. Ceftriaxone

16. Clavulanic acid is combined with amoxicillin because it B Goodman and 1.0
A. easily penetrates gram-negative bacteria Gilman 8th ed
B. inactivates bacterial β -lactamases p.1093
C. is a potent inhibitior of cell-wall transpeptidase
D. has a spectrum of activity similar to penicillin G
Pharmacology
52 / 68

17. Ethambutol is given together with other antitubercular drugs in the treatment of tuberculosis in D Goodman and 0.33
order to Gilman 8th ed
A. facilitate penetration of the blood-brain barrier p.1158
B. delay excretion of other antitubercular drugs by the kidney
C. retard absorption after intramuscular injection
D. retard the development of resistance

18. Chronic candidiasis infections of the gastrointestinal tract and oral cavity are treated orally with A Goodman and 0.33
A. Fluconazole Gilman 8th ed
B. Clotrimazole pp.1177 – 1179
C. Amphotericin B
D. Miconazole

19. The drug which would most likely be effective for the treatment of patients with Legionella B Goodman and 0.5
pneumonia is Gilman 8th ed
A. Streptomycin pp.1133 – 1134
B. Erythromycin
C. Penicillin G
D. Chloramphenicol

20. A 3-year old patient passes out noodle-like worms and is given an appropriate anti-helminthic. B Goodman and 1.0
The mechanism of this drug is most probably via Gilman 8th ed
A. interference with cell-wall synthesis pp.969 – 970
B. inhibition of neuromuscular transmission
C. interference with cell division
D. interference with protein synthesis

21. Streptomycin and other aminoglycosides inhibit bacterial protein synthesis by C Goodman and 0.5
A. Peptidoglycan units in the cell wall Gilman 8th ed
B. DNA p.1100
C. 30S ribosomal particles
D. RNA polymerase

22. A patient with upper respiratory tract infection is resistant to Ampicillin. Which of the following A Goodman and 0.33
drugs may be used? Gilman 8th ed
A. Oxacillin pp.1068 – 1069
B. Carbenicillin
C. Mezlocillin
D. Penicillin G

23. A patient with nosocomial pneumonia is given ceftazidime and another antibiotic. The most D Goodman and 0.5
probable purpose for adding the second antibiotic is to Gilman 8th ed
A. Increase renal excretion pp.1098 – 1113
B. Decrease systemic toxicity
C. Increase oral absorption
D. Prevent the emergence of resistant bacteria

24. A pediatric patient is being treated for typhoid fever. Which of the following adverse effects D Goodman and 1.0
may occur? Gilman 8th ed
A. Bone marrow stimulation p.1127
B. Stainng of teeth
C. Alopecia
D. Aplastic anemia

25. About 75% of patients in this condition may suffer from vertigo, inability to perceive termination B Goodman and 0.5
of movement and difficulty in sitting or standing without visual clues Gilman 8th ed pp.
A. Patients allergic to penicillin 1104-1008
B. Patients receiving streptomycin therapy
C. Patients on isoniazid for tuberculoss
D. Patients on amphotericin B therapy

26. A kidney transplant patient is given a drug for immunosuppression to prevent rejection of the B Goodman and 0.5
allografted organ. The drug is most likely Gilman 8th ed
A. Cyclosporine p.1270
B. Azathioprine
C. Cyclophosphamide
D. Vincristine

27. A patient has a neoplastic growth which is resistant to most chemotherapeutic agents and B Di Palma 4th ed 1.0
requires increased dosages to obtain a response. The tumor probably has a large proportion of pp.653 – 655
cells in the
A. S phase
B. G0 phase
Pharmacology
53 / 68
C. G1 phase
D. M phase

28. This drug causes a nucleophilic attack on DNA that disrupts DNA base pairing D Goodman and 0.33
A. Fluorouracil Gilman 8th ed
B. Methotrexate p.1210
C. Prednisone
D. Cyclophosphamide

29. Cardiotoxicity limits the clinical usefulness of which of the following antitumor antibiotics? A Goodman and 0.25
A. Doxorubicin Gilman 8th ed
B. Dactinomycin pp.1238 - 1239
C. Cisplatin
D. Vincristine
30. This tumor is LEAST susceptible to “cell-cycle specific” anti-cancer agents B Goodman and 0.25
A. Acute lymphoblastic leukemia Gilman 8th ed
B. Adenocarcinoma of the colon pp.1202 – 1207
C. Burkitt’s lymphoma
D. Acute granulocytic leukemia

31. A 40-year old cancer patient who has smoked three pack years of cigarettes for 12 years C DiPalma 4th ed 0.25
presents a decreased pulmonary function test. Physical examination and chest x-rays suggest pp.668 – 669
pre-existing pulmonary disease. All the following drugs may be prescribed EXCEPT
A. Vinblastine
B. Doxorubicin
C. Bleomycin
D. Cisplatin

32. A drug that alkylates target molecules and is useful for ovarin cancer is A Goodman and 0.5
A. Hexamethylmelamine Gilman 8th ed p.
B. Methotrexate 1025
C. Vincristine
D. Busulfan

33. A drug that inhibits DNA topoisomerase II and is effective for testicular cancer is D Goodman and 0.5
A. Vinblastine Gilman 8th ed
B. Bleomycin p.1026
C. Interferon-alfa
D. Etoposide

34. An antagonist of hormone dependent tumor used mainly in breast cancer A Goodman and 1.0
A. Tamoxifen Gilman 8th ed
B. Bleomycin pp.1025 – 1027
C. Cisplatin
D. Fluorouracil

35. Which of the following agents is an antimetabolite? A Goodman and 0.5


A. Thioguanine Gilman 8th ed
B. Ifosfamide pp.1025 – 1027
C. Mitomycin
D. Leuprolide

36. The drug of choice for the prevention of deep vein thrombosis is C DiPalma 4th ed 0.5
A. Aspirin pp.528, 533 – 535
B. Streptokinase
C. Enoxaparin
D. Desmopressin

37. A patient experiences chest pain and is given beta-blockers. The therapeutic action of this B Goodman and 0.5
class of drugs in this case is due to Gilman 8th ed. p.780
A. increased peripheral resistance
B. decreased requirement for myocardial oxygen
C. dilatation of coronary vasculature
D. reduced production of catecholamines

38. A hypertensive patient is treated by a doctor and experiences hemolytic anemia with a positive C DiPalma 4th ed 0.5
Coomb’s test. This is most likely caused by p.472
A. Clonidine
B. Captopril
C. Methyldopa
D. Prazosin

39. An 80-year old patient is taking captopril. His doctor must watch out for D Goodman and 0.5
A. bone marrow depression Gilman 8th ed p.759
Pharmacology
54 / 68
B. drug fever
C. hepatic injury
D. renal failure

40. An elderly chronic smoker with greatly compromised pulmonary reserve and almost persistent D Goodman and 0.5
bronchoconstriction may benefit from cromolyn because it is Gilman 8th ed p.630
A. a bronchodilator
B. an H1-receptor antagonist
C. a beta2-agonist
D. an inhibitor of mediator release

41. A patient was noted to have elevated total cholesterol, LDL and VLDL. Which among the C Goodman and 1.0
following useful drugs for this purpose operates by blocking HMG-CoA reductase to inhibit Gilman 8th ed
cholesterol synthesis? pp.881 – 884
A. Probucol
B. Clofibrate
C. Lovastatin
D. Nicotinic acid

42. A patient continues to have bilateral edema and 3-pillow orthopnea after adequate therapy with A Goodman and 0.5
diuretics and digoxin may need Gilman 8th ed p.758
A. Enalapril
B. Prazosin
C. Minoxidil
D. Hydralazine

43. In a hypertensive patient who is taking insulin to treat diabetes, which of the following drugs is A Goodman and 0.5
able to be used with extra caution and advise to the patient? Gilman 8th ed p.239
A. Propranolol
B. Methyldopa
C. Hydralazine
D. Prazosin

44. The enhancement of contractility of the cardiac muscle fiber brought about by digitalis is B Goodman and 0.5
related to Gilman 8th ed p.817
A. stimulation of calmodulin
B. inhibition of the sodium pump
C. beta-adrenergic stimulation
D. increased production of adenosine

45. Significant relaxation of smooth muscle of both venules and arterioles is produced by which of C Goodman and 0.25
the following drugs? Gilman 8th ed p.803
A. Minoxidil C. Sodium nitroprusside
B. Diazoxide D. Hydralazine

46. Which of the following agents is useful for the treatment of malignant hyperthermia? B Goodman and 1.0
A. Halothane Gilman 8th ed
B. Dantrolene pp.480 – 481
C. Diazepam
D. Baclofen

47. A patient was repeatedly given an inhalational anesthetic and develops post operative jaundice D Goodman and 0.5
and hepatic necrosis. The agent is most likely Gilman 8th ed
A. Methoxyflurane pp.286 – 292
B. Enflurane
C. Nitrous oxide
D. Halothane

48. The preferred treatment for a patient who suffers a series of seizures without complete A Goodman and 0.5
recovery is Gilman 8th ed p.459
A. diazepam
B. chlorpromazine
C. succinylcholine
D. ethosuximide

49. A patient has been taking imipramine for depression. He would most likely experience which of B Goodman and 1.0
the following adverse effects? Gilman 8th ed
A. Seizures pp.405 – 414
B. Anticholinergic effects
C. Hepatotoxicity
D. Nephrotoxicity

50. The use of which of the following drugs is associated with a syndrome characterized by D Goodman and 0.25
hyperthermia, skeletal muscle hypertonicity and catatonia, among other signs? Gilman 8th ed p.399
A. Amobarbital
Pharmacology
55 / 68
B. Oxazepam
C. Phenytoin
D. Trifluoperazine hydrochloride

51. Chlorpromazine may be used not only in treating schizophrenia but is also effective A DiPalma 4th ed 0.5
A. in reducing nausea and vomiting pp.275 – 277
B. as an antihistaminic
C. as an antihypertensive agent
D. for treating bipolar affective disorder

52. A child suffers from lapses of consciousness or vacant stares lasting about 10 seconds, often C Goodman and 0.5
in clusters. The drug which may be most helpful for his disorder is Gilman 8th ed
A. Phenytoin pp.449 - 453
B. Carbamazepine
C. Ethosuximide
D. Phenobarbital

53. Which of the following agents is a selective D2 (dopamine receptor) agonist? B Goodman and 0.5
A. Promethazine Gilman 8th ed p.942
B. Bromocriptine
C. Haloperidol
D. Chlorpromazine

54. Which of the following local anesthetics is useful for topical administration only? D Goodman and 0.5
A. Procaine Gilman 8th ed p.321
B. Bupivacaine
C. Lidocaine
D. Benzocaine

55. A dopamine receptor agonist useful in the therapy of Parkinson’s disease is A DiPalma 4th ed 0.5
A. Bromocriptine pp.317 – 318
B. Selegine
C. Amantadine
D. Belladona

56. A drug used to prevent or reduce nausea and vomiting associated with motion sickness in the A Goodman and 0.5
form of a transdermal patch is Gilman 8th ed
A. scopolamine pp.163 – 164
B. ondansetron
C. diphenhydramine
D. chlorpromazine

57. The non-selective beta-adrenergic blocking agent that is also also a competitive antagonist at A Goodman and 0.5
alpha1-adrenoceptors is Gilman 8th ed
A. Labetalol pp.235 – 236
B. Timolol
C. Pindolol
D. Acebutolol

58. The reversible cholinesterase inhibitor indicted in the treatment of Alzheimer’s disease is D DiPalma 4th ed 0.33
A. Edrophonium pp.159 – 160
B. Neostigmine
C. Pyridostigmine
D. Tacrine

59. A 30-year old G2P1 patient suffers experiences regular and increasingly frequent contractions A Goodman and 0.5
at 6 months of gestation. Which of the following agents may help in this case? Gilman 4th ed p.206
A. Ritodrine
B. Nicotine
C. Propranolol
D. Tacrine

60. A predictably dangerous side effect of nadolol that constitutes a contraindication to its clinical C Goodman and 1.0
use in susceptible patients is the induction of Gilman 8th ed
A. respiratory depression pp.232 – 233
B. hypersensitivity
C. asthmatic attacks
D. hypertension

61. Which of the following bronchodilators is the most selective beta2-agonist, is long acting, and D DiPalma 4th ed 0.5
can be administered by oral inhalation pp.123 – 124
A. Propranolol
B. Pindolol
C. Metaproterenol
D. Salmeterol
Pharmacology
56 / 68

62. Upon taking Penicillin, a patient suddenly experiences respiratory distress and circulatory B Goodman and 1.0
collapse. Which is drug would be most helpful in this situation? Gilman 8th ed p.198
A. Norepinephrine
B. Epinephrine
C. Isoproterenol
D. Atropine

63. Propranolol is either contraindicated in, or should be used with caution in all of the following B Goodman and 0.5
diseases, EXCEPT Gilman 8th ed
A. congestive heart failure pp.238 - 239
B. angina perctoris
C. hypoglycemia
D. bronchial asthma

64. Sumatriptan succinate is effective for the treatment of acute migraine headaches by acting as B DiPalma 4th ed 0.5
A. a agonist at nicotinic receptors pp.201 – 202
B. a selective agonist at 5-HT1D receptors
C. an inhibitor of prostacyclin synthase
D. an ataggonist at beta-adrenergic receptors

65. The pharmacologic effects of acetylsalicylic acid include D DiPalma 4th ed pp. 1.0
A. promotion of platelet aggregation 345 – 353
B. alleviation of pain by stimulation of prostaglandin synthesis
C. minimal gastric irritation
D. a reduction of elevated body temperature

66. Cyproheptadine is an antagonist at A DiPalma 4th ed 0.33


A. serotonin receptors p.203
B. histamine receptors
C. dopamine receptors
D. nicotine receptors

67. A company manager is suffering an attack of allergic rhinitis. He needs to control his symptoms C DiPalma 4th ed 0.5
but also needs to stay awake in a series of staff meetings. Which of the following agents would pp.193 – 194
be most helpful?
A. Chlorpheniramine
B. Hydroxyzine
C. Terfenadine
D. Diphenhydramine

68. All of the following are therapeutic uses of natural prostaglandins or synthetic prostaglandins, C DiPalma 4th ed 0.5
EXCEPT pp.214 – 215
A. temporary maintenance of the patency of the ductutus arteriosus in preterm
neonates
B. cervical ripening in pregnant women
C. treatment of chronic obstructive pulmonary diseases like bronchial asthma
D. prevention of gastric ulceration caused by nonsteroidal anti-inflammatory drugs

69. Hyperkalemia is a contraindication to the use of which of the following drugs? D Goodman and 0.5
A. Chlorothiazide Gilman 8th ed p.726
B. Acetazolamide
C. Ethacrynic acid
D. Spironolactone
70. A reduction in insulin release from the pancreas may be cause by which of the following A Goodman and 0.25
diuretics? Gilman 8th ed p.721
A. Chlorothiazide
B. Triamterene
C. Amiloride
D. Acetazolamide
71. A gout patient developed a nephropathy characterized by overproduction of uric acid and B Goodman and 1.0
extreme hyperuricemia. What agent could have helped prevent this? Gilman 8th ed p.678
A. Cyclophosphamide
B. Allopurinol
C. Sodium chloride
D. Antidiuretic hormone

72. A patient developed stones which are radioopaque without apparent predisposing factors. The A DiPalma 4th ed 0.5
drug which would most likely be helpful is pp.457 – 460
A. hydrochlorothiazide
B. bumetanide
C. furosemide
D. triamterene
73. A hospitalized patient who has been maintained on parenteral alimentation for 3 weeks, C Goodman and 0.5
develops weakness, tremors, agitation and finally, coma. The most likely fluid and electrolyte Gilman 8th ed p.705
Pharmacology
57 / 68
disturbance is
A. hypercalcemia
B. acidosis
C. hypomagnesemia
D. hyperkalemia

74. Antidiuretic hormone is used therapeutically for D Katzung 6th ed 0.33


A. treating nephrogenic diabetes insipidus pp.574 – 575
B. increasing uterine contractility
C. treating polyuria caused by hypercalcemia
D. treating pituitary diabetes insipidus

75. An enhancement of parathyroid hormone-mediated reabsorption of calcium in the distal tubule B Goodman and 0.25
is caused by which of the following diuretics? Gilman 8th ed
A. Bumetanide pp.719, 722
B. Hydrochlorthiazide
C. Actazolamide
D. Furosemide

76. Mannitol may be helpful in all the following procedures, EXCEPT D Goodman and 0.5
A. treatment of elevated intracranial pressure Gilman 8th ed
B. treatment of elevated intraocular pressure pp.714 – 715
C. prophylaxis of acute renal failure
D. treatment of pulmonary edema with congestive heart failure
77. A hypertensive patient with signs of heart failure is on furosemide. He may develop the A Goodman and 0.5
following adverse effects due to the drug EXCEPT Gilman 8th ed
A. metabolic acidosis pp.723 – 724
B. hyperglycemia
C. tinnitus
D. hypotension

78. Adverse reactions associated with both acetazolamide and antibacterial sulfonamides include B DiPalma 4th ed 0.5
all the following EXCEPT p.465
A. formation of urinary calculi
B. metabolic acidosis
C. crystalluria
D. fever
79. A patient with peptic ulcer and on therapy with cimetidine may experience decreased effects of D Goodman and 1.0
many other drugs because cimetidine is an inhibitor of Gilman 8th ed p.901
A. the proton pump
B. tyrosine kinase
C. monoamine oxidase
D. cytochrome P-450

80. Omprazole, a new agent for the promotion of healing of peptic ulcers, has a mechanism of A Goodman and 1.0
action based on Gilman 8th ed
A. H+K+ - ATPase pp.902 – 904
B. prostaglandins
C. gastric secretion
D. pepsin secretion

81. Metoclopramide has antiemetic properties because it C Goodman and 0.5


A. accelerates gastric emptying Gilman 8th ed
B. lowers esophageal sphincter pressure pp.926 – 928
C. is a CNS dopamine receptor antagonist
D. has sedative properties

82. A patient with severe liver deficiency produces ammonia and other toxins in the intestine and A Goodman and 0.5
eventual encephalopathy. Which agent is important in the therapy of this condition? Gilman 8th ed 919 –
A. Lactulose 920
B. Lactate
C. Loperamide
D. Lorazepam

83. A patient suffering from persistent peptic ulcer disease was given bismuth salts aside from B DiPalma 4th ed 1.0
other anti-ulcer agents. This is because of the bactericidal activity of bismuth salts against p.566
A. Escherichia coli
B. Helicobacter pylori
C. Clostridium difficile
D. Staphylococcus aureus

84. Fat soluble vitamins have generally a greater potential toxicity compared with water soluble D Goodman and 0.5
vitamins because they are Gilman 8th ed
A. metabolized faster p.1524
B. more essential to vital metabolic processes
Pharmacology
58 / 68
C. administered in large doses
D. avidly stored in the body

85. A patient is undergoing quadruple chemotherapy for pulmonary tuberculosis. Which vitamin B Goodman and 0.5
would he most probably need a supplement of? Gilman 8th ed
A. Vitamin A p.1524
B. Pyridoxine
C. Thiamine
D. Vitamin C

86. A substance that enhances the probability of ovulation by blocking the inhibitory effect of A Goodman and 0.33
estrogens and thus stimulating the release of gonadotropin from the pituitary is Gilman 8th ed
A. clomiphene pp.1395 – 1396
B. oxymethaolone
C. ethinyl estradiol
D. progesterone

87. A naturally occurring substance useful in treating Paget’s disease of bone is D Goodman and 0.5
A. cortisol Gilman 8th ed
B. etidronate pp.1507 – 1510
C. parathyroid hormone
D. calcitonin

88. The preferred thyroid preparation for maintenance replacement therapy is which of the C DiPalma 4th ed 0.33
following drugs? pp.587 – 589
A. Liotrix
B. Dessicated thyroid
C. Levothyroxine
D. Liothyronine

89. A patient becomes markedly titanic following a recent thyroidectomy. This symptom can be D Goodman and 0.33
rapidly reversed by Gilman 8th ed
A. vitamin D p.1247
B. plicmycin
C. parathyroid hormone
D. calcium gluconate

90. Metyrapone is useful in testing the endocrine functioning of A Goodman and 0.33
A. pituitary adrenal axis Gilman 8th ed
B. Leydig cells of testes p.1458
C. Neurohypophysis
D. α cells of the pancreatic islets
91. Of the following mechanisms of anti-inflammatory and immunosuppressive effects of C Goodman and 0.50
glucocorticoids, which one is uniformly observed? Gilman 8th ed
A. Increased prostaglandin formation pp.1442 – 1445
B. Reduced formation of lipocortins
C. Reduced capillary permeability and edema a the inflammatory site
D. Increased influx of leukocytes to the site of inflammation

92. Bromocriptine is used to treat some cases of amenorrhea because it A Goodman and 0.5
A. inhibits prolactin release Gilman 8th ed
B. increases the synthesis of follicle-stimulating hormone p.1346
C. stimulates the ovary directly
D. stimulates release of gonadotropin-releasing hormone

93. Convulsions caused by drug poisoning are most commonly associated with C Goodman and 1.0
A. phenytoin Gilman 8th ed
B. chlorpromazine pp.1632 – 1633
C. strychnine
D. Phenobarbital

94. Alkalinization of the urine with sodium bicarbonate is useful in the treatment of poisoning with B Goodman and 0.5
A. amphetamine Gilman 8th ed pp.18
B. aspirin – 20
C. morphine
D. cocaine

95. A farmer experiences severe poisoning with parathion. Which of the following agents would be A Goodman and 0.5
most useful in its treatment? Gilman 8th ed p.122
A. Pralidoxime
B. Carbachol
Pharmacology
59 / 68
C. Ethylenediaminetetraacetic acid
D. N-Acetylcysteine

96. Activated charcoal may be used to treat poisoning by all of the following drugs EXCEPT C Goodman and 0.5
A. carbamazepine Gilman 8th ed p.58
B. aspirin
C. lithium
D. Phenobarbital

97. All of the following drugs may produce a syndrome of flushing, headache, nausea, vomiting, D Goodman and 0.25
sweating, hypotension and confusion after ethanol consumption EXCEPT Gilman 8th ed
A. moxalactam pp.378 – 379
B. cefoperazine
C. disulfiran
D. amitriptyline

98. N-acetylbenzoquinoneimine is the hepatotoxic metabolite of which drug? B Goodman and 0.5
A. Isoniazid Gilman 8th ed
B. Acetaminophen pp.657 – 658
C. Indomethacin
D. Sulindac

99. Zinc is an essential element for normal growth and development. However, toxicity can occur C DiPalma 4th ed 0.5
from excessive exposure. Manifestations of chronic poisoning include all the following pp.828 – 829
EXCEPT
A. thrombocytopenia
B. anemia
C. encephalopathy
D. fever

100. A 3-year old boy consumed a liquid from a container in the family garage. He shows central D Goodman and 0.33
nervous system depression, acidosis, suppressed respiration, and oxalate crystals in the urine. Gilman 8th ed
Beside supportive and corrective measures, ethanol was administered to the child. What drug p.1625
most likely caused the toxic effect?
A. Metronidazole
B. Nalidixic acid
C. Primaquine
D. Ethylene glycol

OVERALL MPL 53.46

RGM Questions

120. Which of the following statements is related to the pharmacokinetics of isoniazid?


A. It is readily absorbed from the GIT.
B. It is given with niacin to prevent neuropathy.
C. There is poor diffusion of isoniazid into body tissues.
D. Excretion of isoniazid is primarily through the bile.

Ans. A.

Isoniazid is readily absorbed from the GIT. It readily diffuses into all body fluids and tissues (hence its usefulness even in TB meningitis) where its
CNS and CSF concentration ranges between 20% a1nd 100% of simultaneous serum concentrations. Excretion of its metabolites is mainly in the urine
(as contrasted to rifampicin which is excreted mainly through the liver into bile). Isoniazid promotes the excretion of pyridoxine, pyridoxine, not
niacin, is recommended for patients taking isoaniazid who a1re predisposed to neuropa1thy.

121. Which of the following refer to amikacin?


A. It is effective for Bacteroides, Peptostreptococcus, and Clostridium spp..
B. It exhibits cross-resistance with gentamicin and tobramycin.
C. Once daily dosing of amikacin is ineffective.
D. It is more toxic to the cochlear division of cranial nerve VIII.

Ans. D.

Aminoglycosides, like amikacin, are irreversible bactericidal inhibitors of protein synthesis at the 30s ribosomal subunit. They are actively transported
across the cell membrane through an oxygen-dependent process. Hence, they are ineffective against agents that thrive in an anaerobic environment,
such as Bacteroides, Clostridium, and Peptostreptococcus. (Katzung p 770). The spectrum of antimicrobial activity of amikacin is the broadest of the
aminoglycosides and because of its unique resistance to the aminoglycoside-activating enzymes, it is useful in hospitals where tobra1mycin and
gentamicin resistant organisims are prevalent. (Goodman & Gilman p. 1233) Aminoglycosides may be given on a1 once-a-day basis beca1use of two
properties: (1) concentration-dependent killing, i.e. increasing concentrations kill a1n increasing proportion of bacteria at a1 more rapid rate; and (2)
post-antibiotic effect, such tha1t the antibacterial activity persists beyond the time during which measurable drug is present. (Katzung p 766).
Neomycin, kanamycin, and amikacin are the most ototoxic aminoglycosides. Streptomycin, gentamicin, and tobramycin are the most vestibulotoxic.
Neomycin, tobramycin, and gentamicin are the most nephrotoxic.

Pharmacology
60 / 68

122. Which of the following enzymes is inhibited by the binding of a beta lactam
antimicrobial with penicillin-binding proteins (PBPs)?
A. beta-lactamase C. peptidyl tranferase
B. transpeptidase D. acetylase

Ans. B

This is a question on the mechanism of action of β lactams. Penicillin-binding proteins (PBPs) catalyze the transpeptidase reaction that removes the
terminal alanine to form a crosslink with a nearby peptide. This gives the cell wall its structural rigidity. β lactam antimicrobials (penicillins,
cephalosporins, monobactams, carbapenems) are natural analogs of this terminal alanine substrate and they are covalently bound by PBPs at the active
site. After β lactams have attached to the PBP, the transpeptidation reaction is inhibited, peptidoglycan synthesis is blocked, and the cell dies. Choice
B is incorrect because β lactamase is the group of about 300 identified enzymes that cleave the β lactam ring. This is the most common
mechanism of resistance to β lactams. Some transferase enzymes are also responsible for the resistance of antimicrobials such as the
aminoglycosides. (Katzung, pp. 734-735, 764)

123. A poor farm helper from Palawan was diagnosed to have falciparum malaria by
blood smear. The safe, cost-effective, and readily available agent to prescribe if this
farmer has an allergy to sulfonamides is:
A. Fansidar C. chloroquine
B. quinine D. mefloquine

Ans. B

All of these antimalarials are theoretically useful for falciparum malaria but chloroquine reisitance is observed in highly endemic areas like Palawan.
Quinine, the chief alternative drug for chloroquine resistant strains, is the best choice for the farmer. It is given for free in some health centers. Fansidar
is not an option because it contains sulfadoxine, a sulfonamide. Mefloquine is expensive for a farmer (about 6 times more expensive than chloroquine).

124. Radical cure for malaria is achieved by giving:


A. chloroquine C. mefloquine
B. quinine D. primaquine

Ans. D

Radical cure aims to eliminate blood schizoots and the persisting liver hypnozoites (of the vivax and ovale species). This is achieved by giving a
blood schizonticide and a drug that destroys the persisting liver hypnozoites. Choices A, B, and C are also schizonticides but not to the hepatic stages.
Primaquine, however, is active against all hepatic stages of all human malarial parasites. It is the only available agent against the dormant
hypnozoite stage of P. vivax and P. ovale

125.

126. Which of the following anti-Koch’s preparations has the greatest activity as an
inducer of hepatic enzymes?
A. isoniazid C. pyrazinamide
B. rifampicin D. streptomycin

Ans. B

Rifampicin strongly induces most cytochrome P450 isoforms (CYP1A2, CYP2C9, CYP2C19, CYP2D6, and CYP3A4) which increase the elimination of
numerous other drugs, including methadone, anticoagulants, some anticonvulsants, protease inhibitors, and contraceptives. Administration of
rifampicin with ketoconazole, cyclosporin, or chlorampenicol results in significantly lower serum levels of these drugs. (Katzung p 785) On the other
hand, isoniazid is also known to inhibit the metabolism of other drugs like phenytoin (due to slow acetylation) and carbamazepine. (Katzung, pp. 62,
1116-1122)

127. There is a sudden increase of atypical pneumonia secondary to Mycoplasma,


Chlamydia, and Rickettsia in the Pneumonia Ward of a tertiary hospital. Majority of
patients were treated with doxycycline. All of the following ADRs are to be
watched out for in these patients EXCEPT:
A. photosensitivity C. superinfection
B. fetal bone defects D. renal failure

Ans. D.

In addition to choices A, B, and C, mottling of the teeth is another adverse effect of tetracycline. Doxycycline is a unique tetracycline because it does
not accumulate in patients with impaired renal function. It is excreted in the feces and is, therefore, one of the safest tetracyclines for treating
extrarenal infection.

128. The pharmacology of polyene antimicrobials is best described by the following


statement:
A. Amphothericin and nystatin are both polyene macrolides.
B. There is good absorption from the GIT of the clinically available preparations.
C. They are generally fungistatic agents.
D. They act by reducing ergosterol synthesis by inhibition of fungal cytochrome
P450 enzymes.

Pharmacology
61 / 68
Ans. A.

Amphothericin B is poorly absorbed from the gastrointestinal tract. Nystatin is also not significantly absorbed from the skin, mucus membranes, and
gastrointestinal tract. These drugs are fungicidal. They bind to fungal ergosterol in the cell wall and alter the permeability of the cell by forming pores
in the cell membranes. This mechanism of action is contrasted to that of the azoles which inhibit fungal cytochrome P450 enzymes causing a reduction
of ergosterol synthesis.

129. Treatment with the combination of vancomycin and an aminoglycoside


necessitates a close observation for which of the following:
A. low pitched tinnitus
B. moderately intense headache followed by an acute stage of nausea, vomiting,
and difficulty with equilibrium
C. decreased urine output
D. neuromuscular blockade and apnea

Ans. B.

Vancomycin and aminoglycosides are both potentially nephrotoxic and ototoxic antimicrobials. A moderately intense headache may precede the onset
of labyrinthine dysfunction. Choice A is incorrect because a high-pitched tinnitus is the first symptom of aminoglycoside cochlear toxicity. Acute
tubular nephrotoxicity is associated with a mild rise in serum creatinine, not by oliguria (Choice C). Choice D is an unusual toxic reaction referable to
aminoglycosides but not to vancomycin. Other potentially nepthrotoxic antimicrobials, aside from these two agents are polymixin, cephaloridine, and
tetracycline.

130. Which of the following agents may predispose to the development of


pseudomembranous colitis?
A. erythromycin C. vancomycin
B. metronidazole D. clidamycin

Ans. D

Pseudomembranous colitis clinically presents with the following: watery voluminous diarrhea with or without blood, abdominal cramps and tenderness,
fever, and leucocytosis. Majority of symptoms occur 4-10 days after antibiotic intake; 25% show symptoms after stopping the medications. The
etiological agent is toxigenic Clostridium difficile. The antimicrobials that may cause pseudomembranous colitis are: [1] clindamycin – most common
on a per use basis, [2] cephalosporins – most commonly reported, [3] amoxicillin and other penicillins – second most commonly reported, and [4]
tacrolimus, an anticancer agent. Theoretically, any drug can cause the condition, even vancomycin and metronidazole. The treatment consists of
stopping the offending drug and giving metronidazole 500 mg PO IV q8h or vancomycin 125 mg QID

131. A patient with persistent chronic cough and fever had no response to adequate doses
of oral penicillin. The PE showed normal chest findings. Further PE showed bullous
myringitis. A chest Xray showed infiltrates in the right lower lobe. Which of the
following antimicrobials would be effective?
A. parenteral penicillin G C. vancomycin
B. chlarithromycin D. gentamicin

Ans. B

Mycoplasma pneumonia may be associated with bullous myringitis and normal chest PE findings. This is managed with tetracyclines or macrolides.
Clarithromycin is a macrolide.

132. A 24 year old woman admitted to the emergency room complains of severe, acute,
generalized headache. Her temperature is 40 C and exhibits nuchal rigidity. Her
CSF revealed gram-Negative, diplococci and 200 leucocytes/mm3 (98% of which are
polymorphonuclear neutrophils). The most important management would be the
administration of:
A. IV sulfonamides C. IV penicillin
B. intrathecal penicillin D. intrathecal chloramphenicol

Ans. C

Streptococcus pneumoniae, a gram positive diplococcus, is the most common cause of meningitis in adults greater than 20 years. However, the
laboratory picture points to Neisseria meningitidis, a gram negative aerobic diplococcus, which accounts for nearly 60% of bacterial cases in children
and in young adults between the ages 2-20. (Harrison, 15th edition, p. 2462) Choice C is correct. Penicillin G 4 million units IV q4hours remains an
acceptable treatment in most countries although resistance has been reported. Cefotaxime (2 grams IV q8hours) or ceftriaxone (1 gram IV q12hours)
may be preferred for initial therapy, as it may cover for other bacteria. Chloramphenicol may be used but resistance has also been reported.
Intrathecal administration for acute bacterial meningitis is no longer recommended. Majority of strains of N. meningitidis are resistant to sulfonamides.
(Harrison p 930)

133. A 4 year old boy that has diarrhea and malnutrition was noted to have a mixed
parasitic infection of Ascaris, Capillaria philippinensis, and Trichiuris trichiura.
The single agent that could be effective for this patient is:
A. pyrantel pamoate C. diethylcarbamazine
B. mebendazole D. niclosamide

Ans. B

Pharmacology
62 / 68
Mebendazole, a benzimidazole, has a wide spectrum of antihelminthic activity and can cover for all these three parasites. It acts by inhibiting
microtubule synthesis. Pyrantel pamoate is also a broad spectrum antihelminthic but is ineffective for Trichiuris trichiura. Diethylcarbamazine
citrate, an agent for filariasis, immobilizes their surface structure displacing them from tissues and making them more susceptible to destruction by host
defense mechanisms. Praziquantel is the drug of choice for schistosomiasis. (Katzung, p 889-895) Taenia solium infection is treated with a single
doze of praziquantel. (Harrison p 1249) Niclosamide, a salicylate derivative, is a second line drug for the treatment of most tapeworm infection. It
inhibits oxidative phosphorylation or stimulates ATPase activity. (Katzung, p 889-895)

134. Live vaccines which should, as much as possible, be avoided in pregnancy includes:
A. tetanus C. influenza
B. hepatitis B D. mumps

Ans. D

Immunization is practiced to induce protection against many infectious agents and may utilize either inactivated (killed) materials or live attenuated
agents. (Katzung) The replication of the organism is essential to the development of immunity. Adverse effects from live attenuated vaccines occur
7-10 days after immunization after the organism has replicated and the immune system has responded. Adverse effects of live attenuated vaccines
mimic the disease. Live vaccines are contraindicated in pregnancy. These include: OPV, MMR, BCG, varicella, and cholera vaccines. In contrast,
the inactivated vaccines (which include influenza polyvalent vaccine, pneumococcal polyvalent, H. influenzae, cholera and meningococcal vaccines)
may be given during pregnancy. (Williams Obstetrics) Adverse effects may occurs 24-28 hours after.

135 Lovastatin exhibits all of the following effects on lipid profile EXCEPT:
A. decreased serum cholesterol
B. decreased LDL
C. increased triglycerides
D. increased HDL

Ans. C.

Statins inhibit the rate limitng HMG CoA reductase enzyme for cholesterol synthesis in the liver. Their lipid profile effects are all beneficial to the
patient, including decreased levels of serum cholesterol, triglycerides, VLDL, IDL, and LDL and increased levels of HDL. The bile acid sequestrants
(cholestyramine and colestipol) have the limitation of increasing the serum triglycerides, hence they are indicated only for hypercholesterolermia.
The fibrates such as gemfibrozil may either increase or decrease LDL, which may not be beneficial to the patient.

136. Fibrinolytics affect what specific biological reaction?


A. inactivation of cyclooxygenase
B. inactivation of thrombin
C. formation of gamma-carboxyglutamyl side chain
D. conversion of plasminogen to plasmin

Ans. D

Fibrinolytic drugs rapidly lyse thrombi by cataÿÿzingÿÿhe formation of the serine plasma protease plasmin from its precursor, plasminogen. This
creates a generalized lytic state breaking down the protective hemostatic thrombi and the target thromboemboli. (Katzung p 552) Choice A is the
mechanism for aspirin, the effect of which is the inhibition of the prostaglandin thromboxane A2 synthesis, thus interfering with platelet aggregation
in vitro and prolonging bleeding time in vivo. (Katzung p. 554) Choice B is an indirect effect of heparin. Heparin binding with antithrombin rapidly
inhibits the clotting factor proteases, especially thrombin (IIa), IXa, and Xa. (Katzung p. 545). Choice C requires a reduced Vitamin K to form
thrombin. Warfarin inhibits the enzyme complex (Vitamin K epoxide reductase) necessary for the reduced form of Vitamin K. (Katzung p. 550)

137. The following agent exhibits an increased safety for the treatment of hemophilia A:
A. fresh frozen plasma
B. partially purified clotting factor VIII
C. factor VIII purified by monoclonal antibodies
D. cryoprecipitate

Ans. C.

Hemophilia A is an X-linked deficiency or dysfunction of clotting factor VIII (the antihemophilic factor, AHF). (Harrison 15th edition, p. 751)
Hemophiliacs who have signs of active bleeding will need replacements with this factor. Plasma products enriched in factor VIII have revolutionized the
care of hemophilia patient. The widespread use of factor VII concentrates has produced serious complications, including viral hepatitis, chronic liver
disease, and AIDS. Cryoprecipitate and partially purified lyophilized factor VIII concentrates are obtained from pooled donors, which may predispose
to these complications. Three developments have increased the safety factor VIII therapy and have changed medical practice. First, heating of
lyophilized factor VIII concentrates under carefully controlled conditions can inactivate HIV without destroying factor VIII anticoagulant activity.
Second, highly purified factor VIII can be produced by adsorbing and eluting factor VIII from monoclonal antibody columns. Third, recombinant
factor VIII is now available. Patients with hemophilia should receive either monoclonal purified or recombinant factor VIII to minimize viral infections and
exposure to irrelevant proteins.

138. The laboratory test/s for heparin effect is/are:


A. plasma thromboplastin time C. WBC count
B. protime D. phytonadione

Ans. A

Patients with thrombophilia (tendency to form thrombi) may require anticlotting agents. Heparin is indicated for the initial management of venous
thrombosis to reduce the incidence and mortality from pulmonary emboli. Effective heparinization is required to maintain the plasma thromboplastin time
(PTT) at 2 – 2 ½ times the control, a level which does not cause bleeding but prevents thrombin formation. (Katzung p 548). Choice B is a laboratory
parameter for adequacy of warfarin treatment. The protime should be increased to a level representing a reduction of prothrombin activity to 25% of
Pharmacology
63 / 68
normal and maintained there for long term therapy. When the activity is less than 20%, the warfarin dosage should be reduced or omitted until the
activity rises above 20%. WBC count is not affected by heparinization. However, heparin causes a transient thrombocytopenia in 25% of patients
and severe thrombocytopenia in 5%. This may result from heparin-induced aggregation that is postulated to be benign and transient in character.
(Katzung, p 547) Phytonadione is Vitamin K1, the antidote for warfarin toxicity.

139. The rational anticoagulant for a pregnant patient is:


A. heparin C. hirudin
B. warfarin D. dicuomarol

Ans. A

The coumarin anticoagulants (warfarin and dicoumarol) crosses the placenta readily and can cause hemorrhagic disorder in the fetus, abnormal
bone formation, and other birth defects. (Katzung p 554) Lepirudin is a recombinant derivative of hirudin, a direct thrombin inhibitor present in the
salivary gland of medicinal leech. It is used cautiously in patients with renal failure, since it can accumulate and cause bleeding in these patients.
(Goodman & Gilman p 1526).

140. The following are mucosal protective agents EXCEPT:


A. sucralfate C. misoprostol
B. bismuth subsalicylate D. pirenzepine

Ans. D

Sucralfate, misoprostol, and bismuth are mucosal protective agents. (Katzung, p 1042-1044) Pirenzepine, a potent muscarinic, reduces gastric acid
secretion with fewer adverse effects than atropine. It has been marketed in the past in the Philippines but is still investigational in the US. (Katzung p
1114)

141. Which of the following drug group for PUD has a better inhibition of nocturnal
gastric secretion compared to their 24-hour gastric acid inhibition?
A. antacids C. proton pump inhibitors
B. H2 receptor blockers D. mucosal protective agents

Ans. B.

When given in their usual prescription doses, all of the H2 antagonists inhibit 60-70% of total 24-hour acid secretion. H2 antagonists are especially
effective at inhibiting nocturnal acid (which depends largely on histamine) but have a modest impact on meal-stimulated acid secretion (which is
stimulated by gastrin and acetylcholine as well as histamine). H2 antagonists block more than 90% of nocturnal acid. (Katzung p 1036)

142. The vitamin that is deficient in a patient with diarrhea, dermatitis, and dementia is:
A. thiamin (B1) C. niacin (B3)
B. riboflavin (B2) D. ascorbic acid (C)

Ans. C

Pellagra is characterized by the triad of diarrhea, dermatitis, dementia (three D’s) and is due to a deficiency in niacin. This deficiency seen in chronic
alcoholism, deficiencies of multiple vitamins, protein calorie malnutrition. Other vitamin deficiencies include the following: [1] thiamin – beri-beri; [2] B2
(riboflavin) - characterized by angular stomatitis, cheilosis, red denuded lips, keratitis (seborrheic dermatitis), glossitis (Magenta tongue). These are
followed by anemia and neuropathy; [3] B3 (pantothenic acid) – graying of hair; [4] biotin – Leinnier’s disease and in infants, seborrheic dermatitis;
[5] B6 (pyridoxine) - seborrheic dermatitis, glossitis, stomatitis, and cheilosis, as frequently seen with other B vitamin deficiencies. Microcytic,
hypochromic anemia is due to diminished hemoglobin synthesis; [6] folic acid – megaloblastic anemia; [7] B12 (cyanocobalamin) – pernicious
anemia; [8] C (ascorbic acid) – scurvy; [9] A – night blindness/nyctalopia (most recognizable early symptom of deficiency; skin lesions such as
follicular hyperkeratosis, infections; Bitot’s spot, xerophthalmia, keratomalacia, decreased elasticity of the lungs and other tissues, faulty molding of bone
[10] D (ergocalciferol D2, cholecalciferol D3) – rickets and osteomalacia

143. Rapidly growing tissues may revert to undifferentiated states if there is a deficiency of which vitamin?
A. Vitamin A C. Vitamin E
B. Vitamin D D. Vitamin K

Ans. A

Vitamin A (retinoic acid – the acid form, retinol – the alcohol from, and retinal – the aldehyde form) has several effects on epithelial tissues. Its
functions include: the maintenance of visual purple and vision in dim light, its importance in the growth and differentiation of epithelial cells, bone growth,
reproduction, and embryonic development; and the prophylaxis and perhaps treatment of malignancies. Its action in acne has been attributed to the
decreased cohesion between epidermal cells and increased epidermal turnover. Vitamin D is necessary for normal bone formation affecting
calcium and phosphorus metabolism. Vitamin E is an antioxidant and a coenzyme in some reactions. Vitamin K (K1 – phytonadione and K2 –
menaquinone) confers biologic activity to the clotting factors II (prothrombin), VII, IX, and X.

144. Multivitamin-mineral doses for nutritional support in pregnancy must contain higher doses of:
A. folate C. thiamin
B. iron D. A and B only

Ans. D

Studies have shown a strong correlation between maternal folic acid deficiency and the incidence of neural tube defects such as spina bifida and
anencephaly. Pregnant women have an increased requirement for folic acid. Ferrous iron is also supplemented to pregnant women and other patients
with increased requirements such as lactating women, premature infants, children during the rapid growth periods, and gastrectomized patients.
Pharmacology
64 / 68
(katzung p 531-536). Thiamin is not associated with dietary lack in pregnancy. In Western countries, the primary causes of thiamine deficiency are
alcoholism and chronic illness such as cancer. Poor dietary intake is the most common cause of thiamin deficiency worldwide. (Harrison p 461)

145. From among the following, ingestion of gasoline or kerosene may cause the most
severe reaction to which of the following organs:
A. esophagus C. lungs
B. stomach D. skin

Ans. C

146. In a patient who is not comatose, emesis is indicated after acute poisoning following oral ingestion of all of the following EXCEPT:
A. strong alkali or acid C. digitalis
B. antihistamine D. morphine

Ans. A

Emesis can be induced in poisoning with ipecac syrup (never extract of ipecac). It should not be done if the suspected intoxicant is a corrosive agent
(like strong acids and bases), a petroleum distillate, or a rapidly acting anticonvulsant. (Katzung p 987)

147. Activated charcoal may be used to treat poisoning due to the following EXCEPT:
A. primaquine C. propoxyphene
B. aspirin D. iron overdose

Ans. D

Activated charcoal can adsorb many drugs and poisons owing to its large surface area. It is most effective if given in a ratio of at least 10:1 of charcoal
to the estimated dose of toxin. Charcoal does not bind iron, lithium, or potassium, and it binds alcohols and cyanide only poorly.

148. A patient who is taking metronidazole or oral hypoglycemic sulfonylureas is


advised to refrain from alcohol intake to avoid:
A. metallic taste C. disulfiram-like effects
B. severe hypertension D. hyperglycemia

Ans. A

Metronidazole, certain cephalosporins (like cefotetan), trimethoprim, sulfonylureas, and chloral hydrate produce disulfiram-like ADRs when taken
with alcohol. (Katzung p 375) The primary pathway for alcohol metabolism involves alcohol dehydrogenase, the enzyme that catalyzes the
conversion of alcohol to acetaldehyde. Oxidation of acetaldehyde is inhibited by disulfiram. When acetaldehyde accumulates, it causes an unpleasant
reaction of facial flushing, nausea, vomiting, dizziness, and headache. (Katzung p 368).

149. This herbal plant endorsed by the DOH is useful for the relief of muscle pain:
A. Yerba Buena C. Tsaang gubat
B. Akapulko D. Sambong

Ans. A

Akapulko is used as an anti-fungal. Tsaang gubal is used for stomach pain. Sambong is a diuretic and an anti-urolithiasis agent.

150.. The drug recommended as single agent treatment of acute poisoning of arsenic
and mercury is:
A. penicillamine C. dimercaprol
B. edentate calcium disodium D. deferoxamine

Ans. C

In addition to its primary indication for mercury and arsenic poisoning, dimercaprol is a secondary drug for lead poisoning. Penicillamine is used
chiefly for treatment of poisoning with copper or to prevent copper accumulation, as in Wilson’s disease. EDTA is an efficient chelator of many divalent
and trivalent metals in vitro. It is used chiefly for the chelation of lead but may be used for zinc, manganese, and certain heavy radionuclides.
Deferoxamine binds iron avidly; however, it binds essential trace metals poorly, making it the chelator of choice for iron poisoning. (Katzung p 978-
980)

151. The necessary enzyme for folic acid synthesis that is inhibited by
diaminopyrimidines is:
A. dihydrofolate reductase C. mycolase synthetase
B. dihydropteroate synthetase D. reverse transcriptase

Ans. A

Dihydrofolate reductase in inhibited by diaminopyrimidines, while dihydripteroate synthetase is inhibited by sulfonamides. These are the necessary
enzymes for folic acid synthesis. Reverse transcriptase is a vital HIV enzyme necessary for viral nucleic acid synthesis which is inhibited by certain anti-
retroviral agents (zidovudine, didanosine, lamividine – are among the nucleoside reverse transcriptase inhibitors; nevirapine, delavirdine, and
efavirenz – are among the non-nucleoside reverse transcriptase inhibitors). (Katzung) transc

152. Which of the following refers to erythromycin base?


A. it is an azalide antimicrobial
B. it inhibits protein synthesis at the 23s ribosomal subunit
Pharmacology
65 / 68
C. it remains stable in an acidic environment
D. it is excreted in the bile and feces

Ans. D

The macrolides include erythromycin, clarithromycin, and azithromycin. Azithromycin, however, is more specifically an azalide. Their mechanism of
action is inhibition of protein synthesis by binding to the 50s ribosomal RNA. Clindamycin and chloramphenicol also act by binding to the 50s
ribosomal RNA. Tetracyclines, however, bind to the 30s ribosomal unit. It is the oxazolidinone, lineizolid, that binds to the 23s ribosomal RNA of the
50s subunit. (Katzung p 754-762)

153. The poor-healing wound in the hand of a non-diabetic mechanical engineer did not
respond to oral penicillin V. The single rational beta-lactam antimicrobial that
could be given for this patient on the basis of efficacy, affordability, and availability
from among the following is:
A. benzathine penicillin C. ticarcillin
B. methicillin D. cloxacillin

Ans. D

Cloxacillin is the rational anti-staphylococcal penicillin for this patient. Methicillin, a nephrotoxic anti-staphylococcal agent, is not available in the
market. Ticarcillin is effective for gram-negative organisms.

154. From among the following, the rational antimicrobial for a 7 month old Filipino
with community-acquired pneumonia (if a laboratory is not available) is:
A. cephalexin C. chloramphenicol
B. ciprofloxacin D. vancomycin

Ans. C

Without a laboratory available, the choice of an antimicrobial is based on the most common etiological agents for this age group which are H. influenzae
and Streptococcus pneumoniae. Chloramphenicol can adequately cover for both organisms (as well as the aminopenicillins, ampicillin and
amoxicillin, and second generation cephalosporins). Vancomycin is too expensive and too toxic to consider for a community acquired pneumonia. It is
reserved for the more resistant strains of bacilli. Cephalexin is relatively ineffective for H. influenzae. Without the benefit of a good laboratory, it will
not be a good choice.

155. If the agents below are available in the nearby drug store, the efficacious and
affordable agent for a school teacher with Tinea (pytiriasis) versicolor on large
areas on her chest and her back:
A. selenium sulfide C. amphothericin b
B. fluoconazole D. terbinafine

Ans. A

156. Which of the following statements is TRUE about diloxanide furoate?


A. it is a effective for extraintestinal amoebiasis
B. it is the agent of choice for hepatic abcess
C. it is effective for asymptomatic cyst passers
D. it is safe for infants and pregnant patients

Ans. C

157. On regular follow-up for tuberculosis, you noticed that the visual acuity of Vilma
worsened from 20/20 to 20/50 in both eyes after only three weeks of HRPE. Your
approach in management would be to discontinue:
A. all of the 4 drugs and reintroduce one drug at a time
B. ethambutol and refer to an ophthalmologist
C. pyrazinamide and refer to an ophthalmologist
D. ethambutol, reintroduce it after 1 week, and refer to an ophthalmologist
Ans. B

158. While on multiple drug treatment for pulmonary Koch’s infection, hearing
impairment, ringing of the ears, and dizziness may be attributable to which of the
following?
A. rifampicin C. streptomycin
B. isoniazid D. ethambutol
Ans. C

159. Drug toxicity in patients with renal failure may be avoided by:
A. maintaining the normal therapeutic dose
B. peritoneal dialysis
C. interval prolongation
D. hemodialysis
Ans. C

160. Fetal alcohol syndrome is characterized by:


A. intrauterine growth retardation C. microcephaly
Pharmacology
66 / 68
B. indistinct philtrum D. all of the above.

Ans. D.

These features are easily seen on PE. Mental retardation is another prominent feature of fetal alcohol syndrome.

161. A 5 year old male child with nephritic syndrome on steroid therapy was exposed to
a sister with measles. You would administer:
A. measles vaccine C. antisera
B. human immunoglobulin D. vitamin A

Ans. B.

162. The following statement is NOT TRUE regarding mannitol:


A. It produces diuresis in the presence of reduced GFR.
B. It is the diuretic of choice in the treatment of chronic pulmonary edema.
C. It is filterable and poorly reabsorbed in the renal tubules.
D. It produces very little, if any, disturbance of acid-base balance.
Ans. B.

163. To avoid the risk of drug toxicity in patients with renal failure, which of the
following must be done:
A. maintaining the normal therapeutic dose
B. peritoneal dialysis
C. interval prolongation
D. hemodialysis
Ans. C.

164. A 5 year old girl with nephritic syndrome in relapse has been taking furosemide
prescribed by a health center officer for persistent edema for the past 3 weeks. She
developed marked weakness with inability to ambulate. This reaction is probably
due to:
A. hypersensitivity to furosemide C. hypokalemia
B. hyponatremia D. metabolic acidosis
Ans. C.

SPARE RGM QUESTIONS

82. The following may be considered for Salmonella typhi enteritis:


A. ciprofloxacin (Ciprobay) C. ceftriaxone (Rocephin)
B. chloramphenicol (Chloromycetin) D. all of the above

Ans. D

110. Which of the following agents is primarily eliminated by the kidneys?


A. ethanol C. amikacin
B. paracetamol D. vancomycin

Ans. C

133. Theophylline plasma levels may be increased by inhibiting microsomal enzyme


activity is given concomitantly with:
A. rifampicin C. Phenobarbital
B. erythromycin D. griseofulvin

Ans. B

135. While eating shrimps, a 20 year old patient suddenly develops stridor, dyspnea, and
cyanosis. His blood pressure and heart rate are going down. As the attending ER
physician, you should give immediately:
A. IV hydrocortisone C. IV phenylephrine
B. IV aminophylline D. subcutaneous aqueous epinephrine

Ans. D

136. Succinylcholine as a muscle relaxant acts by:


A. persistent depolarization of the muscle membrane
B. blocking the nicotinic acetylcholine receptor at the neuromuscular junction
C. blocking the muscarinic receptor sites
D. facilitating GABA transmission

Ans. A

139. Neostigmine can antagonize the neuromuscular blockade caused by non-


depolarizing drugs by:
A. increasing the availability of acetylcholine at the muscle end plate
Pharmacology
67 / 68
B. directly stimulating the muscarinic cholinoceptors
C. activating the excitation-contraction coupling in the muscle fibers
D. facilitating the action of GABA in the CNS

Ans. A

149. The principal site of action of the potassium-sparing spironolactone is the:


A. proximal convoluted tubule C. distal convoluted tubule
B. thick ascending loop of Henle D. collecting ducts

Ans. B

151. The effect of this recycling process of reabsorption from the intestines is to prolong
drug action. What is this specific process?
A. continuous plasma protein binding
B. enterohepatic circulation
C. biliary excretion
D. hepatic metabolism

Ans. B

152. The most predominant CYP isoform involved in human drug metabolism is:
A. CYP1A1 C. CYP2D6
B. CYP2C9 D. CYP3A4

154. There is increasing interest in making available active enantionmers of the chiral
drugs because in racemic mixtures, the enantiomers may:

1. differ in their duration of action


2. vary in their pharmacologic activity
3. vary in their toxicity
4. equally fit to the receptor molecule

155. Which of the following is a proposed mechanism for theophylline-induced


bronchodilation?
a. potentiation of phosphodiesterase increasing the cAMP
b. potentiation of phosphodiesterase decreases cAMP
c. inhibition of phosphodiesterase increases cAMP
d. inhibition of phosphodiesterase decreases cAMP

Ans. C

156. Which of the following is a potent 5 lipoxygenase inhibitor


a. zafirlukast c. montelukast
b. zileuton d. all of the above

Ans. B

157. Which anticholinergic drug which selectively inhibits M3 muscarinic receptors


a. fluticasone c. ipratropium
b. formoterol d. tiotropium

Ans. D

158. A major adverse effect of cyclosporine that limits its use is:
A. ototoxicity C. nephrotoxicity
B. liver dysfunction D. bone marrow toxicity

Ans. C.

159. An 8-year old female had mumps recently. Her mother claimed that she got an
MMR vaccine at 15 months old. This can be explained by:
A. increasing prevalence of un-immunized children
B. waning immunity
C. lack of efficacy of initial immunization
D. virulent strain of virus

Ans. B.

160. Poor immunological response in infants is expect in the administration of which of


the following:
A. polysaccharide antigen C. toxoid
B. live vaccine D. recombinant Dna vaccine

Ans. A

Pharmacology
68 / 68
162. In renal failure, one of the following statements is NOT TRUE:
A. There is a reduced absorption due to gastroparesis.
B. There is reduced protein binding due to diminished affinity of albumin binding
sites for drugs.
C. There is decreased hepatic deacetylation, conjugation, and hydroxylation of
drugs
D. There is increased sensitivity to receptors.

Ans. D.

163. Thiazide diuretics can cause the following adverse effects EXCEPT:
A. hyperuricemia C. hypokalemia
B. hypoglycemia D. hypercholesterolemia

Ans. C.

164. The principal site of action of the potassium sparer spironolactone is at the:
A. proximal convoluted tubule C. distal convoluted tubule
B. thick ascending loop of Henle D. collecting ducts

Ans. C.

Pharmacology

Das könnte Ihnen auch gefallen